You are on page 1of 54

EXAMEN COMENTADO MIR 2005-2006

1 Para perfilar mejor el cuadro clnico le realizamos 5


Un paciente de 42 aos de edad consulta por molestias preguntas, que se refieren a continuacin. Cul de ellas
farngeas y cierta afona de dos meses de evolucin. NO es es la menos relevante para establecer la sospecha de
fumador ni tiene una profesin en que deba forzar la voz. enfermedad orgnica?
Una exploracin larngea revela un ligero edema de las 1. Ha perdido peso?.
cuerdas vocales y un ligero eritema de la regin 2. Tiene sensacin de saciarse con poca cantidad de alimente?.
interaritenoidea. Interrogado el paciente no refiere pirosis
ni regurgitacin cida. Cul de las siguientes 3. Las molestias le despiertan por la noche?.
afirmaciones es cierta?: 4. Tiene vmitos?.
1. La presencia de mnimas lesiones larngeas indican que muy 5. Mejora con anticidos?.
probablemente el paciente tenga tambin lesiones de esofagitis
y por tanto hay que indicar una endoscopia digestiva alta.
La respuesta correcta es: 5
2. En ausencia de sntomas de broncoaspiracin (tos y
sibilancias), las molestias larngeas no pueden ser atribuidas a
enfermedad por reflujo gastroesofgico. COMENTARIO:
3. La ausencia de sntomas de reflujo (pirosis y regurgitacin) La presencia de dispepsia de larga evolucin y debut a edad
no descarta la enfermedad por reflujo. avanzada obliga a excluir causa orgnica. La prdida de peso,
4. Puede averiguarse si la causa de los sntomas es una saciedad precoz sugieren neoplasia. Los vmitos neoplasia o
enfermedad por reflujo gastroesofgico con una prueba corta lesin estentica benigna. El ritmo del dolor de predominio
administrando ranitidina 150 mg al da durante dos semanas. nocturno sugiere ulcus pptico. Los anticidos alivian tanto los
La ausencia de mejora sintomtica descarta el reflujo como procesos benignos como malignos de forma transitoria.
causa de los sntomas larngeos.
5. Si un trnsito esfago-gstrico con bario muestra hernia
hiatal, es altamente probable quelos sntomas larngeos. 4
Un paciente diagnosticado de enfermedad celaca no
mejora clnicamente tras 5 meses de habrsele indicado
La respuesta correcta es: 3 una dieta carente de gluten. Los estudios analticos
muestran persistencia de ttulos altos de anticuerpos
antiendomisio tipo IgA. Cul es la causa ms probable de
COMENTARIO:
esa falta de respuesta a la dieta sin gluten?:
En la enfermedad por reflujo gastroesofgico, la ausencia de
1. Que no sigue correctamente la dieta.
clnica no puede descartar su existencia, especialmente en los
2. Que no es una enfermedad celaca lo que padece ese
casos sin esofagitis, hecho que ocurre en la mayora de las
enfermo.
ocasiones. La mayora de las ocasiones con manifestaciones
3. Que se trata de un esprue colgeno.
extraesofgicas cursan sin esofagitis y pueden presentarse
4. Que la enfermedad celaca se asocia con una
independientes. El tratamiento de la ERGE es con IBPs no con
inmunodeficiencia comn variable.
antiH2.
5. Que ha desarrollado un linfoma.

2
La respuesta correcta es: 1
Mujer de 52 aos que consulta por presentar desde hace
varios meses de dificultad para tragar tanto lquidos como
slidos, as como regurgitaciones de comida sin digerir. En COMENTARIO:
el proceso diagnstico se realiza una manometra La primera causa de remisin incompleta o persistencia de los
esofgica con los siguientes hallazgos: ausencia de sntomas y ttulos elevados de anticuerpos es el incumplimiento
peristaltismo en el cuerpo esofgico, hipertona y diettico. Slo excluyendo esta causa cabra plantearse en
relajacin incompleta del esfnter esofgico inferior tras la segundo lugar, un linfoma.
deglucin. Cul es el diagnstico?
1. Espasmo esofgico difuso.
2. Acalasia. 5
3. Peristalsis esofgica sintomtica. Cul de los siguientes hallazgos indica con mayor
4. Esofagitis por reflujo. precisin la gravedad de enfermedad heptica asociada a
5. Esclerodermia. infeccin por virus de la hepatitis C?:
1. Genotipo de virus infectante.
2. Concentracin de RNA-VHC srico.
La respuesta correcta es: 2 3. Estudio histolgico.
4. Edad de inicio temprana.
5. Niveles sricos de transaminasas.
COMENTARIO:
La presencia de disfagia a lquidos y slidos sugiere enfermedad
motora y el diagnstico de achalasia se obtiene al detectar La respuesta correcta es: 3
relajacin incompleta del EEI en la manometra.
COMENTARIO:
3 La hepatitis por virus C, genotipo, cuantificacin de RNA y edad
Acude a la consulta del Centro de Salud un hombre de 69 de inicio son factores con poder pronstico de respuesta al
aos de edad, no fumador ni bebedor, que refiere molestia tratamiento antiviral. La gravedad viene determinada por la
desde hace 4 meses. Ha tomado por su cuenta unas evolucin histolgica hacia la cirrosis o el grado de fibrosis
pastillas de Omeprazol que tena en casa, sin mejorar. heptica en general.

1
6 La respuesta correcta es: 5
Un hombre de 32 aos con el antecedente de una colitis
ulcerosa de 3 meses de evolucin, tratado con prednisona
COMENTARIO:
(40 mg/da) y mesalazina (4 g/da) acude al servicio de
La profilaxis primaria de sangrado por varices esofgicas es
urgencias por empeoramiento de su estado general
eficaz con la administracin de betabloqueantes no
acompaado de una aumento en el nmero de las
cardioselectivos cuando se observan varices esofgicas en la
deposiciones (hasta 10 diarias, todas con sangre) y dolor
endoscopia. Las tcnicas endoscpicas pueden emplearse
abdominal clico. Es ingresado instaurndose tratamiento
cuando est contraindicado el tratamiento farmacolgico.
con esteriodes por va intravenosa (1 mg/kg/da) y
sueroterapia. A las 36 horas del ingreso se incrementa el
dolor abdominal, aparece una distensin abdominal 9
importante y ausencia de deposiciones. La exploracin Mujer de 32 aos con antecedentes personales de
fsica muestra una temperatura corporal de 38,7 C; 124 consumo de drogas va parenteral hasta hace 4 aos, que
latidos por minuto; abdomen distendido, doloroso y algo en una analtica de control se objetiva serologa psitiva
timpnico junto a signos de irritacin peritoneal. En el par el virus C de la hepatitis y para el VIH negativas.
hemograma la cifra de leucocito alcanza 17.000/dl. Los Pareja estable en la actualidad. Cul de las siguientes
estudios radiolgicos realizados muestran un colon medidas NO aconsejaramos a esta paciente?:
transverso dilatado de 8 cm sin existencia de aire libre 1. Vacunacin de la hepatitis A.
peritoneal. El deterioro del paciente persiste tras 24 horas 2. La lactancia materna.
de observacin. Qu tratamiento le parece ms 3. No compartir objetos de aseo personal.
adecuado?: 4. Utilizar siempre mtodos de barrera en las relaciones
1. Infliximab. sexuales con su pareja.
2. Colectoma subtotal de urgencia. 5. No donacin de sangre.
3. Azatioprina.
4. Ciclosporina por va intravenosa.
5. Salazopirina a dosis altas por va oral. La respuesta correcta es: 4

La respuesta correcta es: 2 COMENTARIO:


A los enfermos con hepatitis C no se les prohibe la lactancia
(ausencia de riesgo de infeccin). No deben compartir objetos
COMENTARIO: de higiene personal ni donar sangre por riesgo de infeccin. Se
El caso clnico describe una complicacin de la colitis ulcerosa. recomienda vacunacin de VHA y VHB para prevenir
Tiene datos de brote grave y la distensin abdominal as como sobreinfeccin o hepatitis crnica. El riesgo de transmisin
la dilatacin del colon en la radiologa dan el diagnstico de sexual es bajo y en pacientes heterosexuales con pareja estable
megacolon txico. El tratamiento de megacolon txico es se les informa de ese bajo riesgo y queda a su eleccin, por
inicialmente mdico, pero dado que presenta mala evolucin en tanto no es obligado el empleo de mtodos de barrera en las
las primeras horas, est indicada la colectoma subtotal de relaciones sexuales.
urgencia.
10
7 En el diagnstico diferencial de la peritonitis bacteriana
En un paciente con colitis ulcerosa de 12 aos de espontnea del cirrtico con respecto a la peritonitis
evolucin, las biopsias seriadas demostraron la presencia secundaria, cul de los siguientes datos puede hacer
de displasia grave en las muestras del rea sigmoidea, sospechar una peritonitis secundaria?:
Qu actitud recomendara?: 1. Presencia de dolor abdominal a la palpacin.
1. Reseccin del sigma. 2. Presencia de >250 neutrfilos por mm3 de lquido asctico.
2. Colectoma total. 3. Flora monomicrobiana en la tincin de Gram.
3. Nueva toma de biopsias en 3 meses y reseccin de la lesin 4. Nivel de Glucosa en lquido asctico menor que en plasma.
en caso de confirmarse. 5. Baja concentracin de protenas en lquido asctico.
4. Ecoendoscopia para evaluar la invasin de la pared intestinal.

5. Quimioterapia con 5-fluorouracilo y reevaluacin en 6 meses. La respuesta correcta es: 4

La respuesta correcta es: 2 COMENTARIO:


Tanto la PBE como la PBS pueden cursar con dolor abominal y
neutrofilia. La etiologa monomicrobiana y una escasa
COMENTARIO: concentracin protica en el lquido asctico sugiere PBE. Los
En la colitis ulcerosa los factores de riesgo ms importante para niveles de glucosa bajos se observan con mayor frecuencia en
el desarrollo de un cncer de colon son la existencia de una la PBS.
pancolitis y la duracin de la enfermedad ms de 8 aos. La
presencia de displasia de alto grado o grave as como la de un
cncer son indicaciones de Proctocolectoma total. 11
Varn de 38 aos de edad que en los ltimos tras aos ha
presentado cifras moderadamente elevadas de
8 transaminasas. Afirma beber una o dos cervezas los fines
Paciente de 62 aos, diagnosticado de cirrosis por el virus de semana. La exploracin es normal, salvo peso 83 kg y
de la hepatitis C y sin antecedentes de descompensacin talla 174 cm con Hb 14,5 g/dl. VCM 86 fl, resto de la
de su hepatopata, en el que al efectuarse una hematimetra normal. ASAT 65 u.i., ALAT 87 u.i.,
fibrogastroscopia se detectan varices esofgicas de gran triglicridos 213 mg/dl, colesterol total 184 mg/dl,
tamao. Cul es la actitud teraputica que debera glucemia basal 114 mg/dl, ferritina 392 mg/ml.
adoptarse?: Bilirrubina, fosfatasa alcalina, cobre, ceruloplasmina, alfa-
1. Administrar somatostatina. 1-antitripsina, sideremia y CTF normales. Marcadores
2. Erradicar las varices con esclerosis endoscpica. virales A, B y C y autoanticuerpos no organoespecficos
3. Administrar nitratos se accin prolongada. (ANA, AMA, ASMA, anti LKM) negativos. Seale la opcin
4. Colocar una derivacin portosistmica percutnea ms razonable en este momento:
intraheptica. 1. El enfermo es homocigotico para la mutacin C282Y del gen
5. Administrar betabloqueantes no selectivos. HFE.

2
2. Es necesario iniciar tratamiento con estatinas. 14
3. Es obligado realizar una biopsia heptica de inmediato. La prueba de funcin pancretica de mayor sensibilidad
4. El diagnstico ms probable en este enfermo es hgado graso para valorar el grado de insuficiencia pancretica exocrina
no alcohlico. es:
5. El enfermo oculta que bebe alcohol en exceso de forma 1. Quimotripsina en las heces.
habitual. 2. Consumo de aminocidos tras la inyeccin de
secretina/pancreozimina.
3. Comida de prueba de Lundh.
La respuesta correcta es: 4
4. Secretina-pancreozimina.
5. Prueba de dilaurato de flurorescena.
COMENTARIO:
Sideremia y CTF normal excluyen hemocromatosis. No se
La respuesta correcta es: 4
describen macrocitosis ni marcadores bioqumicos como
predominio de GOT (ASAT) sobre GPT (ALAT), ni GGT que
sugiera etiologa alcohlica oculta. Presenta IMC elevado, COMENTARIO:
aumento de triglicridos por lo que el diagnstico ms probable Para el diagnstico de insuficiencia pancretica exocrina la
es enfermedad heptica por depsito graso no alcohlico. Es prueba ms sensible (detecta los casos ms leves) y especfica
coherente con este diagnstico la hiperglucemia. El tratamiento es el test secretina-pancreozimina. Usualmente en la prctica se
requiere reduccin de peso y si fuese necesario fibratos, no emplean pruebas de menor sensibilidad pero no invasivas como
estatinas. el dilaurato de fluorescena o la quimotripsina fecal.

12 15
Cul de las siguiente afirmaciones con relacin al Una paciente de 42 aos ingres hace 6 das con un
hemangioma heptico es cierta?: cuadro de pancreatitis aguda de origen biliar. La mala
1. Afecta al 1% de la poblacin. evolucin obliga a determinar si presenta necrosis
2. Se denomina gigante si sobrepasa los 10 cm de dimetro. pancretica. Qu prueba diagnstica solicitara?:
3. No es una neoplasia sino una malformacin vascular. 1. Ecografa abdominal con contraste endovenoso.
4. Suelen ser sintomticos. 2. Colangio-Resonancia magntica.
5. Cuando sobrepasan los 6 cm est indicada la ciruga 3. Radiografa simple de abdomen.
exertica aunque no produzcan sintomatologa. 4. Colangio pancreatografa retrograda endoscpica.
5. Tomografa computerizada con contraste endovenoso.
La respuesta correcta es: 3
La respuesta correcta es: 5
COMENTARIO:
El hemangioma heptico es una malformacin vascular; es muy COMENTARIO:
frecuente y asintomtica. Excepcionalmente requiere En las pancreatitis agudas con criterios de gravedad se precisa
tratamiento. evaluar la presencia de necrosis pancretica como forma de
correlacin. La prueba para demostrar necrosis es el TC
abdominal con contraste.
13
Al leer el resultado de una ecografa abdominal, que usted
orden por infecciones urinarias de repeticin, comprueba 16
que la mujer de 67 aos que tiene sentada en su consulta Mujer de 75 aos de edad, sin antecedentes de inters,
tiene clculos en la vescula biliar, son otros hallazgos que acude a urgencias por dolor abdominal clico difuso y
significativos. La paciente niega haber presentado clicos vmitos alimentarios y biliosos. Se diagnostica clico biliar
o ninguna otra complicacin relacionada con la colelitiasis. y se pauta tratamiento espasmoltico, pero la paciente
Su nica queja digestiva es la distensin abdominal empeora clnicamente, apareciendo febrcula, distensin
postprandrial. Cul de las siguiente recomendaciones le abdominal y disminucin del peristaltismo. En la
dar usted?: radiografa de abdomen se observa: asas de intestino
1. La distensin postprandrial indica que debe ser sometida a delgado dilatadas con distribucin en patrn "en escalera"
una colecistectoma de urgencia. y niveles hidroareos, pequea imagen de densidad calcio
2. No est indicado el tratamiento quirrgico porque se trata de en cuadrante inferior derecho y aire en la va biliar. Su
una colelitiasis asintomtica. sospecha diagnstica es:
3. Debe ser sometida a un colecistectoma abierta y no 1. Ileo biliar.
laparoscpica para prevenir un cncer de vescula. 2. Colangitis.
4. La colecistectoma profilctica est indicada por tratarse de 3. Colecistitis enfisematosa.
una mujer. 4. Trombosis de la vena mesentrica.
5. Debe someterse a un esfinterotoma endoscpica para 5. Apendicitis aguda.
prevenir un coledocolitiasis.
La respuesta correcta es: 1
La respuesta correcta es: 2
COMENTARIO:
COMENTARIO: Se trata de una paciente que asocia sntomas tpicos de clico
La colelitiasis asintomtica, en general no tiene indicacin de biliar con un cuadro posteriormente de obstruccin intestinal,
tratamiento quirrgico, slo se acepta y recomienda tanto clnico como radiolgico. La presencia de imagen clcica
colecistectoma profilctica en algunas situaciones especiales en la fosa iliaca derecha (ileon terminal) asociada a niveles
tales como: hidroareos por la obstruccin y la aerobilia hacen que el
- Clculos de gran tamao (>2,5 cm). cuadro ms probable sea el leo biliar, es decir, la obstruccin
- Anomalas congnitas de la va Biliar. intestinal por un clculo biliar impactado en ileon terminal, que
- Anemia Falciforme. ha conseguido pasar al tubo digestivo por una fstula
- Colelitiasis incidental: concominante con otras cirugas colecistoduodenal.
digestivas incluida la ciruga baritrica (de obesidad morbida).
- Vescula en porcelana.
17
La causa ms frecuente de abdomen agudo por oclusin

3
mecnica de intestino delgado es una de las siguientes: baos de asiento, laxantes u otras medidas para evitar el
1. Plastrn apendicular. estreimiento. Cuando este tratamiento fracasa y la fisura anal
2. Hernia inguinal incarcerada. es crnica se recomienda la esfinterotoma interna, ya sea
3. Hernia crural incarcerada. qumica o quirrgica.
4. Adherencias o bridas de intestino delgado consecuencia de
laparotomas previas.
20
5. Neoplasia de colon derecho.
En las oclusiones del colon izquierdo causadas por una
neoplasia en el recto-sigma es cierto que:
La respuesta correcta es: 4 1. El dolor suele ser intenso y de tipo claramente clico.
2. Los vmitos suelen ser poco abundantes y tardos.
3. La distensin abdominal suele ser poco importante.
COMENTARIO:
4. El cierre intestinal completo es poco habitual.
La causa ms frecuente de obstruccin intestinal de intestino
5. No resulta posible una oclusin de asa cerrada.
delgado son las adherencias o bridas como consecuencia de
laparotomas previas. En pacientes sin ciruga abdominal previa
la causa ms frecuente de oclusin de ID son las hernias. La respuesta correcta es: 2

COMENTARIO:
La causa ms frecuente de obstruccin a nivel de colon es la
presencia de un cncer colorrectal, con mayor frecuencia recto
18
y sigma. Suele instaurarse de forma lenta, por lo que un dolor
Paciente que acude a Urgencias por dolor abdominal
intenso y clico es poco frecuente. Los sntomas fundamentales
localizado en fosa ilaca izquierda. La historia clnica
son el dolor abdominal difuso, progresivo y no clico
sugiere una diverticulitis. Cul de las siguientes
acompaado de distensin abdominal, vmitos y estreimiento,
exploraciones NO debe utilizarse para estadificar la
con incapacidad para la expulsin de heces y gases. Los
gravedad de la enfermedad o evaluar la evolucin de la
vmitos suelen ser tardos, ms an si existe una obstruccin
misma?:
en asa cerrada por la competencia de la vlvula ileocecal.
1. Exploracin fsica.
2. Recuento de leucocitos.
3. Ecografa abdominal. 21
4. TAC con contraste. La literatura cientfica demuestra que el nico beneficio de
5. Colonoscopia. la profilaxis antibitica en ciruga electiva de aparato
digestivo es:
1. Reducir el riesgo de peritonitis difusa postoperatoria.
La respuesta correcta es: 5
2. Prevenir la formacin de abscesos intraabdominales.
3. Reducir la tasa de infecciones postoperatorias de las heridas
COMENTARIO: quirrgicas.
En fase aguda de diverticulitis aguda el diagnstico es 4. Prevenir las infecciones urinarias por enterobacterias.
fundamentalmente clnico siendo el TAC abdominal la mejor 5. Reducir la tasa de dehiscencia de anastomosis en ciruga
prueba de imagen para valorar la gravedad y la presencia de clica.
complicaciones. En fase aguda deben excluirse pruebas que
supongan un posible aumento de presin intraluminal en el
La respuesta correcta es: 3
colon por lo que deben evitarse tanto el enema opaco como la
colonoscopia que aumentaran el riesgo de perforacin.
COMENTARIO:
La profilaxis antibitica en ciruga electiva slo ha demostrado
19
reducir tasa de infeccin de la herida quirrgica sin haberse
Un paciente de 56 aos acude a la consulta por dolor anal
observado beneficios en la formacin de abscesos
de comienzo sbito tras un episodio de estreimiento. No
intraabdominales ni otras infecciones postoperatorias como las
tiene conductas sexuales de riesgo, no presenta fiebre y
infecciones urinarias o neumonas.
se queja de que cuando va a hacer deposicin siente un
dolor como "si le cortara". A la exploracin, no se
observan alteraciones externas pero la intentar hacer un 22
tacto rectal existe un aumento del tono del esfnter y es Una paciente de 34 aos de edad presenta un cuadro de
imposible hacer progresar el dedo por las quejas del cirrosis descompensada con ascitis y encefalopata. Se
paciente. De entre los siguientes, el diagnstico ms detecta virus de la hepatitis C y un hepatocarcinoma de 3
probable y la opcin teraputica es: cm en el lbulo heptico derecho. Cul es el tratamiento
1. Trombosis de una hemorroide externa - incisin y drenaje del de eleccin?:
coagulo. 1. Reseccin del lbulo heptico derecho.
2. Fisura anal aguda - tratamiento conservador con baos de 2. Reseccin limitada del tumor.
asiento, analgsicos y fibra. 3. Interfern.
3. Procitis aguda - metronizadol. 4. Trasplante heptico.
4. Proctalgia fugax - aminotriptilina. 5. Derivacin posto-sistmica.
5. Tumor del canal anal - radioterapia.
La respuesta correcta es: 4
La respuesta correcta es: 2
COMENTARIO:
COMENTARIO: El tratamiento del hepatocarcinoma en pacientes no cirrticos o
El dolor anal intenso se debe generalmente a fisura anal, con cirrosis estadio A de Child es la extirpacin quirrgica. En
absceso perianal generalmente asociado a fiebre y tumefaccin pacientes con cirrosis estadios B o C de Child, con
fluctuante perianal) o hemorroide complicada (prolapso hepatocarcinomas menores de 5 cm sin enfermedad
hemorroidal agudo o trombosis hemorroidal externa). En este extraheptica, est indicado el transplante heptico, dado que
caso, el factor desencadenante es el estreimiento, que produce la reserva funcional heptica en estos pacientes supondra una
un herida en el ano. La hipertona del esfnter y el dolor intenso insuficiencia heptica postoperatoria en caso de que hiciramos
que impide la exploracin son sugestivas de fisura anal aguda. una reseccin heptica.
El tratamiento de la fisura anal aguda es conservador, con

4
23 COMENTARIO:
Seale cul de las siguientes afirmaciones en relacin con La IC se caracteriza por presentar una marcada activacin
la presin central es FALSA: neurohormonal con elevacin, entre otros marcadores, del BNP.
1. La vena ms apropiada para realizar una estimacin correcta No es esperable en estos paciente que su valor sea normal ni
de la presin venosa central es la yugular interna. disminuido sino elevado en respuesta al aumento de presin
2. Los pacientes con presin venosa central elevada deben ser intraventricular.
examinados con el tronco en posicin horizontal para observar
mejor la onda de pulso venoso.
26
3. Se considera que la presin venosa est elevada cuando la
La etiologa ms frecuente e insuficiencia tricspide
distancia vertical entre el ngulo esternal y la parte superior de
orgnica es:
la columna venosa oscilante es mayor de 3 cm.
1. Infarto de miocardio.
4. La causa ms frecuente de presin venosa elevada es el
2. Carcinoide.
aumento de presin diastlica del ventrculo derecho.
3. Endocarditis.
5. En las fases precoces de la insuficiencia cardaca puede ser
4. Prolapso.
til la prueba del reflejo hepatoyugular para evidenciar un
5. Congnita.
aumento de la presin venosa.

La respuesta correcta es: 3


La respuesta correcta es: 2

COMENTARIO:
COMENTARIO:
La etiologa ms frecuene de IT orgnica es la endocarditis por
Si se examina en posicin horizontal al tener elevada la PVC por
afectacin de los velos o de incluso el aparato subvalvular por
la tera de vasos comunicantes no lograremos ver el punto de
los grmenes, tpicamente en adictos a drogas por va
mxima oscilacin de la columna venosa no pudiendo por tanto
parenteral. No obstante, la afectacin que con ms frecuencia
realizar la medida En las opciones 1,3, 4, 5 se hace referencia a
produce IT es la hipertensin pulmonar, que origina dilatacin
los principales espectos fisiopatolgicos de la presin venosa
del anillo tricuspideo y de las cavidades derechas, pero en ese
yugular.
caso es funcional y no por enfermedad orgnica valvular.

24
27
Un paciente de 65 aos, diagnsticado de insuficiencia
Se trata de un paciente de 78 aos, ingresado en la Unidad
cardaca de etiologa isqumica, en estadio avanzado
Coronaria, en el tercer da de evolucin de un infarto
(grado funcional III de la NYAH), consulta por
agudo de miocardio inferior que haba cursado sin
empeoramiento de su disnea. En el ltimo ao ha sufrido
complicaciones. De forma sbita el paciente pierde la
dos episodios de Edema Agudo de Pulmn y a raz del
conciencia y presenta severa hipotensin y falta de pulso,
ltimo fue dado de alta con dieta pobre en sal, inhibidores
con persistencia de complejos QRS en el monitor. En la
de la Enzima convertidora de la angiotensina (IE-CAs),
exploracin fsica aparecen cianosis e ingurgitacin
furosemida, espironolactona y aspirina (150mg/da). En el
yugular. Cul sera su sospecha diagnstica?:
momento de la exploracin es paciente no tiene disnea,
1. Shock hipovolmico agudo por hemorragia interna
est en ritmo sinusal, con una frecuencia cardaca normal
(seguramente gastrointestinal).
en reposo, tiene crepitantes en ambas bases y su tensin
2. Reinfarto de miocardio.
arterial es de 115/75. En la radiografa de trax hay
3. Ruptura de msculo papilar de la vlvula mitral.
signos de hipertensin postcapilar, sin imgenes de
4. Ruptura del tabique interventricular.
condensacin ni derrame pleural. Cul sera su
5. Ruptura de pared libre y taponamiento.
recomendacin teraputica?:
1. Iniciar tratamiento con antagonistas del calcio y nitritos
orales. La respuesta correcta es: 5
2. Sustituir la Aspirina por Clopidogrel.
3. Hacer una broncoaspiracin y cultivo del material aspirado.
4. Aadir Digoxina oral. COMENTARIO:
5. Iniciar tratamiento gradual con betabloqueantes. La clnica de la rotura cardaca aguda consiste en una
disociacin electromecnica brusca (prdida de pulso y
mantenimiento de la actividad elctrica).
La respuesta correcta es: 5
28
COMENTARIO: La actitud indicada en un sndrome coronario agudo con
El empleo de betabloqueanes mejora el pronstico de pacientes elevacin de ST es:
con severa disfuncin ventricular sistlica incluso en fases 1. Monitorizacin electrocardiogrfica.
avanzadas de la enfermedad CF III-IV con la condicin de que 2. Seguimiento de marcadores de dao miocrdico.
el paciente se encuentre euvolmico, por lo que est indicado 3. Test de esfuerzo.
su empleo en estos pacientes fuera de las descompensaciones. 4. Scan de perfusin.
5. Terapia de reperfusin.
25
Cul de los siguientes parmetros NO es de mal La respuesta correcta es: 5
pronstico en el paciente con insuficiencia cardaca
crnica?:
1. Consumo mximo de oxgeno <10 mk/kg por minuto. COMENTARIO:
2. Disfuncin ventricular izquierda <25%. En un SCA con ascenso del ST la actitud prioritaria, que puede
3. Hiponatremia. modificar la historia natural de la enfermedad es la reperfusin
4. Valores de BNP (pptido natriurtico tipo B) disminuidos (mediante angioplastia primaria o farmacolgica con
(<100 pg/ml). fibrinolticos.)
5. Extrasistolia ventriculares frecuentes.
29
La respuesta correcta es: 4 Paciente de 22 aos, fumador de 1/2 paquete al da y
diagnosticado de soplo cardiaco en la adolescencia. Acude
al hospital por dolor torcico y disnea. En la auscultacin

5
se detecta un soplo sistlico en mesocardio que aumenta La respuesta correcta es: 3
con la maniobra de valsalva. En el ECG est en fibrilacin
auricular rpida, tiene signos de crecimiento ventricular
COMENTARIO:
izquierdo y ondas q en DI y aVL. La Rx de trax demuestra
La estenosis renal sobre rin nico contraindica el empleo de
la existencia de insuficiencia cardaca y silueta normal.
IECA o ARA II, el asma el empleo de betabloqueantes y la crisis
Cul es el diagnstico ms probable?:
gotosa aconseja no emplear diurticos puesto que aumentan la
1. Estenosis artica congnita.
uricemia. Por la comorbilidad asociada de la paciente la
2. Infarto lateral alto.
repuesta razonable es un calcio antagonista.
3. Miocardiopata dilatada.
4. Angina inestable.
5. Miocardiopata hipertrfica obstructiva. Pregunta 33
La diseccin aguda de la aorta torcica tipo B de Stanford
se caracteriza por los siguientes hallazgos anatmicos:
La respuesta correcta es: 5
1. Diseccin que afecta la raz de la aorta y la vlvula artica,
pero preservando el resto de la aorta ascendente.
COMENTARIO: 2. La diseccin solamente afecta el cayado o argo artico.
La bipedestacin y la maniobra de Valsalva slo aumentan el 3. Diseccin que afecta a toda la aorta ascendente.
soplo en dos enfermedades (PVM y MHO). El resto de datos, 4. Diseccin de la aorta descendente distal a la arteria subclavia
especialmente la hipertrofia ventricular, facilita el diagnstico izquierda.
de esta entidad. 5. Diseccin de toda la aorta torcica.

30 La respuesta correcta es: 4


El tratamiento con sulfato magnsico en el infarto agudo
de miocardio se indica en situacin de:
COMENTARIO:
1. Insuficiencia renal.
La diseccin distal tipo B de Stanford se define como una
2. Hiperpotasemia.
diseccin de la aorta torcica descendente distal al origen de la
3. Hipercalcemia.
arteria subclavia izquierda. En principio, el manejo de esta es
4. Taquicardia ventricular con QT alargado.
conservadora, salvo en caso de que se complique (rotura,
5. Bloqueo A-V.
progresin), estando entonces indicada la ciruga.

La respuesta correcta es: 4


Pregunta 34
Los paciente en estado de choque presentan un marcado
COMENTARIO: descenso de la presin arterial sistmica. Segn la causa
En el contexto de un IAM solo se justifica el empleo de que motiva el estado de choque, la hipotensin arterial es
magnesio si aparece una TV en torsin de puntos con QT debida a alteraciones en el gasto cardiaco y/o alteraciones
prolongado. Este frmaco se utiliza adems en los episodios de las resistencias vasculares sistmicas. Entre las
torsadas secundarias a QT largo de otro origen (farmacolgico, siguientes, seale la respuesta correcta:
hipocalcemia, ...) con eficacia variable. 1. En el choque de origen sptico, el gasto cardiaco y las
resistencias vasculares se hallan incrementadas.
2. En el choque hemorrgico, el gasto cardiaaco est elevado y
Pregunta 31
las resistencias vasculares se hallan incrementadas.
Un joven de 18 aos acude a nuestra consulta por cifras
3. En el choque de origen cardiaco, el gasto cardiaco est
elevadas de la presin arterial. Est asintomtico desde el
reducido y las resistencias vasculares se hallan incrementadas.
punto de vista cardiovascular. En la exploracin fsica
4. En el choque causado por una crisis tireotxica, el gasto
destaca un clic de eyeccin en mesocardio y foco artico,
cardiaco est reducido y las resistencias vasculares se hallan
sin soplo y una marcada disminucin de la amplitud del
incrementadas.
pulso en extremidades inferiores. Cul de los siguiente
5. En el choque causado por una insuficiencia heptica, el gasto
diagnsticos es ms probable?:
cardiaco est reducido y las resistencias vasculares se hallan
1. Insuficienca artica por vlvula artica bicspide.
incrementadas.
2. Estenosis valvular artica de grado moderado.
3. Coartacin de aorta.
4. Miocardiopata hipertrfica. La respuesta correcta es: 3
5. Diseccin artica.
COMENTARIO:
La respuesta correcta es: 3 Los tipos de shock no son demasiado relevantes en el MIR, pero
en esta ocasin plantean una pregunta sobre su diagnstico
diferencia. El shock cardiognico implica una disfuncin cardaca
COMENTARIO:
en su tarea habitual de bombear sangre, por lo que habr un
La coartacin de aorta se caracteriza por una disminucin de la
descenso del gasto cardaco. Tambin habr aumento de las
amplitud del pulso en EEII con respecto a los superiores,
resistencias perifricas por la descarga catecolaminrgica que le
adems de un soplo en mesocardio. La HTA es un signo clave,
acompaa, de ah que la correcta sea la 3.
de forma que debe descartarse la coartacin ante todo
hipertenso con una exploracin simultanea de los pulsos en
extemidades superiores e inferiores. Pregunta 35
Hombre de 50 aos, infarto anterior extenso. En el
ecocardiograma se aprecia depresin severa de la funcin
Pregunta 32
ventricular, insuficiencia artica severa y no se aprecian
Cul es el antihipertensivo de eleccin en una mujer de
alteraciones en aorta ascendente. Presenta hipotensin
60 aos, asmtica, con crisis de gota, TA: 158 mmHg/90
importante y cuadro compatible con edema agudo de
mmHg, y estenosis de arteria renal sobre rin nico?:
pulmn, y ha presentado varias crisis de angor postinfarto
1. Diurtico.
durante el ingreso en la unidad coronaria. Cul de las
2. Betabloqueante.
siguientes actuaciones NO sera correcta?:
3. Calcioantagonista.
1. Iniciar tratamiento con dopamina.
4. Inhibidor de la enzima conversora de la angiotensina.
2. Realizar cateterismo cardiaco urgente.
5. Antagonista del receptor de la angiotensina.
3. Iniciar tratamiento con furosemida.

6
4. Introduccin de baln de contrapulsacin artico. una operacin de Bentall, con colocacin de tubo valvulado y
5. Introduccin de catter de termodilucin para monitorizacin reimplante de coronarias a la prtesis.
de gasto cardiaco y presiones endocavitarias.
Pregunta 38
La respuesta correcta es: 4 La cianosis central slo se debe a:
1. Aumento del volumen minuto cardiaco.
2. Aumento de la masa ertrocitaria.
COMENTARIO:
3. Aumento del valos hematocrito.
El BCIAo se contraindica en los sndromes articos agudos y en
4. Aumento de la hemoglobina reducida.
la insuficiencia artica severa. En esta ltima, al inflarse el
5. Aumento de la carboxihemoglobina.
baln en distole se aumentara la regurgitacin artica previa.
Las dems opciones son parte del amplio abanico teraputico
de un shock cardiognico. La respuesta correcta es: 4

Pregunta 36 COMENTARIO:
Un hombre de 50 aos de edad acude al servicio de La cianosis (coloracin azulada) de la piel puede ser central o
urgencias con dolor e hinchazn de la pierna derecha en perifrica. Las centrales se caracterizan por tener alto la Hb
los ltimos dos das. Fuma 2 paquetes de cigarrillos al da reducida como ocurre en enfermedad de origen pulmonar,
y est algo obeso. Recuerda que se dio un golpe en la cardaca o hipoventilacin.
pierna contra una mesa 3 das antes y se hizo una herida.
La temperatura es 38 C y la pierna derecha est
Pregunta 39
visiblemente hinchada hasta la ingle con moderado
Un da en que la presin atmosfrica es de 705 mmHg, un
eritema. Los pulsos son normales y el signo de Homans es
paciente de 40 aos se presenta en la sala de urgencias
negativo. Cul de las siguientes afirmaciones es
con una presin arterial de osgeno de 37 mmHg, una
correcta?:
presin de anhdrido carbnico de 82 mmHg y un Ph de
1. La ausencia de un cordn palpable y un signo de Homans
7.22. Calculamos la presin alveolar de oxgeno, que
negativo hacen el diagnstico de trombosis venosa profunda
resulta ser 39 mmHg. Entre las causas de insuficiencia
poco probable.
respiratoria mencionadas ms abajo. Cul ser la ms
2. La fiebre y eritema hacen el diagnstico de trombosis venosa
probable?:
poco profunda muy improbable.
1. Una embolia de pulmn.
3. El paciente debe comenzar con anticoagulantes con heparina
2. Es una insuficiencia respiratoria crnica reagudizada en un
inmediatamente.
paciente con enfermedad pulmonar obstructiva crnica (EPOC).
4. Dado que no hay evidencia de tromboembolismo pulmonar,
3. Tiene una crisis asmtica grave.
el paciente puede comenzar con anticoagulacin oral
4. Una neumona extensa.
(acenocumarol) sola.
5. Una sobredosis de morfina.
5. Debe realizarse una flebografa intravenosa en 24 horas.

La respuesta correcta es: 5


La respuesta correcta es: 3

COMENTARIO:
COMENTARIO:
Tpica pregunta del MIR sobre insuficiencia respiratoria.
Este paciente presenta una trombosis venosa profunda (signos
Recuerda que un aumento de pCO2 implica estado de
inflamatorios en miembro inferior y una serie de factores de
hipoventilacin y si estos se acompaan de D (A-a)o2 normal
riesgo trombtico, como son la obesidad, el tabaco y el
(PAO2 39 - Pa O2 37 = D (A-a)o2 = 2). El paciente presenta
antecedente de traumatismo). La ausencia de signo de Homans
una enfermedad no pulmonar.
positivo en absoluto descarta el diagnstico. En esta situacin,
debe iniciarse de inmediato tratamiento anticoagulante.
Pregunta 40
Un paciente de 58 aos de edad, no fumador, con disnea y
Pregunta 37
tos, es enviado al neumlogo. La historia clnica revela
Paciente varn de 29 aos, jugador activo de baloncesto.
catarros invernales prolongados, con tos y expectoracin,
Al exploracin fsica destaca pectus excavatum y
desde hace 10 aos. La situacin funcional ha sido muy
aranodactilia. Su padre fallecin por muerte sbita a la
buena hasta hace unos seis meses. En este perodo h
edad de 47 aos. En un estudio ecocardiogrfico se
aaparecido disnea pregresiva, que en la actualidad es de
detecta insuficiencia artica severa con dimetro
pequeos esfuerzos. La tos es improductiva. En la
telediastlico del VI de 75 mm y una fraccin de eyeccin
exploracin se aprecian cianosis central, acropaquias,
de 0.40. La aorta ascecente tiene un dimetro de 5 cm.
disminucin de la movilidad respiratoria, sibilancias
Qu actitud recomendara en dicho paciente?:
espiratorias y prolongacin del tiempo espiratorio. En la
1. Recambio valvular artico aislado en ese momento.
misma consulta se realiza una espirometra que muestra
2. Tratamiento mdico con calciantagonistas hasta que
los siguientes resultados: Capacidad vital 40% del valor
aparezcan los sntomas.
terico, volumen respiratorio mximo en 1 segundo
3. Tratamiento mdico con betabloqueantes hasta que
(FEVI) 70% del valor terico, el FEV 25-75 es 66% del
aparezcan los sntomas.
terico. Cul, entre las que se enumeran a continuacin
4. Control ecocardiogrfico anual hasta que aparezcan los
es la siguiente prueba a realizar que mejor podra aclara la
sntomas.
alteracin funcional encontrada?:
5. Recambio de la vlvula artica y la aorta ascendente con
1. Radiografa de trax, PA y lateral.
tubo valvulado (Operacin de Bentall).
2. Citologa de esputo.
3. Medicin de volmenes pulmonares por pletismografa.
La respuesta correcta es: 5 4. Espirometra con broncodilatadores.
5. Hemograma, bioqumica srica, estudio inmunolgico.
COMENTARIO:
Se trata de un paciente con sndrome de Marfan con La respuesta correcta es: 3
insuficiencia artica severa, FEVI moderadamente deprimida y
Ao torcica aneurismtica. La opcin quirrgica aconsejable es
COMENTARIO:
Nos presenta clnicamente a un paciente con enfermedad

7
pulmonar crnica con defectos de la espirometra. Pero los es de:
datos espiromtricos son contradictorios, ya que presenta un IT 1. Neumonitis por hipersensibilidad.
alto (FEV1/CVF), luego necesitan conocer el VR para diferenciar 2. Proteinosis alveolar.
obstruccin con atrapamiento areo frente a restriccin 3. Sacoidosis.
pulmonar. 4. Silicosis.
5. Silicatosis.
Pregunta 41
Enfermo diagnosticado de asma que no est recibiendo La respuesta correcta es: 1
tratamiento. En la historia clnica refiere episodios de
desnea sibilante en 4 ocasiones en el curso de una
COMENTARIO:
semana, ocasionalmente se despierta por la noche con
Pregunta bsica de enfermedades intersticiales. Los sntomas y
sntomas de asma y tiene un FEV1 que es el 80% del
la imagen del TAC apoyan el diagnstico de enfermedad
terico. Cmo debera ser tratado inicialmente este
intersticial, y el contacto con el ganado vacuno nos da la clave
enfermo?:
de neumonitis por hipersensibilidad (respuesta 1). No hay
1. Una dosis baja-moderada de corticoides inhalados ms un
sntomas de sarcoidosis, la proteinosis alveolar da un patrn
beta-antagonista.
perihiliar y no suele dar sntomas, y est expuesto a slice
2. Deber evitar alergenos y utilizar un corticoide inhalado a
(granito) pero es demasiado joven (30 aos) como para que
demanda.
haya dado tiempo a que se produzca. Tampoco aparecen las
3. Un beta agonista solo.
tpicas placas pleurales.
4. Una combinacin de tofilinas con corticoides inhalados y beta
agonistas.
5. Un ciclo inicial de corticoides orales combinado con beta Pregunta 44
agonistas. En el sndrome de apnea del sueo hay hipoxia,
hipercapnia y acidosis y puede asociarse con las
siguientes alteraciones EXCEPTO:
La respuesta correcta es: 1
1. Hipertensin arterial.
2. Hipertensin pulmonar.
COMENTARIO: 3. Hipoventilacin alveolar.
Esta pregunta plantea un problema: se trata de una errata, 4. Bradicardia.
puesto que la opcin 1 dice beta-antagonistas cuando debera 5. Aumento de las fases de ondas lentas del sueo profundo.
decir beta-agonistas, por lo que se tratara de una pregunta
equvoca. La opcin correcta sera la 1 si en vez de antagonistas
La respuesta correcta es: 5
dijese agonistas. Probablemente se trate de un error de
transcripcin por parte del Ministerio, puesto que las otras
cuatro opciones no son correctas. COMENTARIO:
La dificultad de esta pregunta deriva de no ser un tema habitual
del MIR y de que las preguntas previas se centran en el
Pregunta 42
tratamiento. Se puede sacar por descarte. El SAOS se asocia a
Un paciente de 68 aos con EPOC fue dado de alta en el
HTA (aunque no se ha demostrado asociacin a HTA crnica).
hospital en tratamiento con oxgeno domiciliario. Dos
Condiciona hipoventilacin alveolar e HTP secundaria a hipoxia
mese ms tarde acude a consulta refiriendo disnea con
alveolar. Se acompaa de bradicardia durante las pausas y
medianos esfuerzo. Ha dejado de fumar y sigue
habitualmente taquicardia posterior. La desestructuracin del
tratamiento correctamente con broncolilatadores
sueo por las pausas afecta principalmente al sueo de ondas
inhalados. Presenta saturacin de oxgeno de 85%
lentas, que es el reparador.
respirando aire ambiente y hematocrito de 49%. Cude
las siguiente afirmaciones respecto a la oxigenoterapia
domiciliaria es correcta?: Pregunta 45
1. Debe interrumpirse por tener hematocrito normal. Un paciente de 60 aos de edad acude a la consulta por
2. Debe continuar, pero no ms de 12 horas al da. habrsele descubierto un patrn de tipo intersticial en la
3. Debe administrarse mediante gafas nasales con flujo del, al radiografa de trax. Refiere ser fumador de 40
menos, 10 litros/minuto. paquetes/ao y haber trabajado como montador de
4. Debe mantenerse a largo plazo, pues se ha demostrado que tuberas. En la tomografa axial computerizada se observa
alarga la supervivencia en pacientes como ste. el mencionado patrn y un engrosamiento pleural. El
5. Debe mantenerse mientras el paciente permanezca diagnstico en que pensar es:
sintomtico. 1. Fibrosis pulmonar.
2. Neumoconiosis.
3. Bronquiolitis respiratoria.
La respuesta correcta es: 4
4. Asbestosis.
5. Silicosis.
COMENTARIO:
Se trata de un paciente que ya est en tratamiento con
La respuesta correcta es: 4
oxigenoterapia crnica domiciliaria, y la respuesta 4 que dice
que hay que mantenerla porque se ha demostrado que
aumenta la supervivencia es correcta porque cumple los COMENTARIO:
requisitos de estabilidad, medicacin broncodilatadora completa Pregunta fcil de un tema recurrente en el MIR. La exposicin a
y porque sigue con saturacin O2 por debajo del 88%, lo que asbesto per se, se asocia a placas pleurales (engrosamiento de
representa pO2 por debajo de 55 mmHg. la pleura parietal de contornos bien definidos y por lo general
irregulares), pero la presencia de enfermedad intersticial es
definitoria de asbestosis. En este caso, la exposicin al asbesto
Pregunta 43
est camuflada en las fibras de aislamiento de las tuberas.
Un paciente de 30 aos habitante de un pueblo de
Cantabria, no fumador, que trabaja por las maanas en
una cantera de granito y por las tardes ayuda en su casa al Pregunta 46
cuidado del ganado vacuno, acude al mdico por disnea de Una mujer de 55 aos, fumadora durante aos consulta
esfuerzo y un patrn intersticial. La TAC torcica confirma por sntomas derivados de un sndrome de vena cava
el patrn intersticial y adems objetiva imgenes de vidrio superior. En la Rx de trax hay marcado ensanchamiento
esmerilado de tipo mosaico. El diagnstico ms probable mediastnico. Cul es la patologa ms frecuente?:

8
1. Carcinoma microctico pulmonar. estadificacin en donde se confirma mediante estudio
2. Carcinoma qumico. anatomopatolgico una adnopata positiva en regin
3. Enfermedad de Hodgkin. paratraqueal baja derecha (4R). Segn la clasificacin
4. Linfoma folicular. TNM. Qu implica este hallazago en la estadificacin?:
5. Carcinoma tiroideo anaplsico. 1. T4.
2. N1.
3. N2.
La respuesta correcta es: 1
4. N3.
5. M1.
COMENTARIO:
Se describe una paciente con un sndrome de vena cava
La respuesta correcta es: 4
superior. La causa ms frecuente es el carcinoma broncognico,
y en concreto, el carcinoma microctico. Por lo tanto, la
respuesta correcta es la 1. COMENTARIO:
Una vez ms la estadificacin del cncer de pulmn ha cado en
el MIR. Es importante que recuerdes que N1 son ganglios
Pregunta 47
perihiliares y peribronquiales, N2 los subcarnicos y
Hombre de 65 aos de edad que presenta de forma aguda
mediastnicos ipsilaterales, y N3 los contralaterales y
disnea, dolor torcico pleurtico derecho y hemoptisis
supraclaviculares (opcin 4).
moderada. Una gammagrafa pulmonar de ventilacin-
perfusin se interpreta como de alta probabilidad de
embolismo pulmonar. Una radiografa de trax muestra un Pregunta 50
derrame pleural derecho que ocupa menos de un tercio del Mujer de 64 aos ingresada por neumona en lbulo
hemorrax derecho. La toracocentesis demuestra que se inferior derecho, en tratamiento con antibitico emprico.
trata de un derrame sero-hemorrgico. Cul de los Presenta una evolucin trpida los da posteriores al
siguientes sera el tratamiento correcto para este ingreso con persistencia de la fiebre, disnea y dolor de
paciente?: caractersticas pleurticas en hemitrax derecho. La
1. Colocar un tubo de drenaje torcico e iniciar anticoagulacin. radiografa de trax presenta veladura de la mitad inferior
del hemitrax derecho compatible con derrame pleural. Se
2. Iniciar anticoagulacin con heparina sdica y colocar un filtro realiza toracocentesis diagnstica obtenindose un lquido
de vena cava inferior. claro de aspecto seroso, con un pH 6,95, LDH 200, glucosa
3. Tratar con heparina sdica en bomba de infusin a dosis 30 y e Gram del lquido es negativo. Cul de las
plenas. siguientes sera la actitud correcta a seguir en este
4. Evitar la anticoagulacin y colocar un filtro en vena cava momento?:
inferior. 1. Mantener el mismo tratamiento antibitico y nueva
5. Realiza un arteriografa y despus iniciar anticoagulacin. toracocentesis diagnstica en 24 horas.
2. Cambiar el tratamiento antibitico empricamente a la espera
de los cultivos del lquido pleural.
La respuesta correcta es: 3
3. Colocacin de un tubo de drenaje pleural.
4. Solicitar una broncoscopia urgente para toma de muestras y
COMENTARIO: cultivo microbiolgico de las mismas.
Se describe un paciente con un TEP. El tratamiento de eleccin 5. Las caractersticas del lquido son compatibles con un
es la heparina (respuesta correcta la 3). La presencia de trasudado por lo que debera iniciarse tratamientos con
hemoptisis moderada NO es una contraindicacin para diurticos ante la sospecha de una insuficiencia cardiaca.
anticoagular, ya que adems est provocada por la patologa
que se quiere tratar con la heparina (la respuesta 4 no es
La respuesta correcta es: 3
correcta).

COMENTARIO:
Pregunta 48
Se trata de una pregunta asequible en la cual se debe conocer
Las radiografa de trax de un paciente de 62 aos, con
los datos bioqumicos que nos hacen sospechar la presencia de
dolor torcico de algunas semanas de evolucin,
un empiema, que en este caso es el pH<7, ya que el Gram es -.
demuestran una lesin focal, bien definida, perifrica, de
La presencia de empiema en lquido pleural es indicacin
contorno ntido, convexa hacia el pulmn, con dimetro
absoluta de drenaje pleural adems de los antibiticos que ya
crneo-caudal mayor que el transversal, con ngulos
tiene el paciente pautados.
obtusos con respecto a la pared torcica y que se
acompaa de una lesin osteoltica costal. Cul, entre los
siguientes, es el diagnstico ms probable?: Pregunta 51
1. Mesotelioma pleural. En el neumotrax espontneo primario, cul de las
2. Derrame pleural encapsulado. siguientes afirmaciones NO es cierta?:
3. Carcinoma de pulmn. 1. La causa ms importante es la rotura de "blebs" subpleural.
4. Tumor fibroso pleural. 2. Entre 1/3 y la mitad de los pacientes presentan recurrencias
5. Plasmocitoma. de la enfermedad.
3. Se da con ms frecuencia en pacientes fumadores.
4. El tratamiento depende del volumendel neumotrax, que sea
La respuesta correcta es: 5
primer episodio o haya tenido con anterioridad ms, y en
algunos casos de la profesin del paciente.
COMENTARIO: 5. Se da con ms frecuencia en pacientes mayores de 50 aos
Las preguntas sobre lesiones mediastnicas han aumentado de con enfermedad pulmonar obstructiva crnica.
frecuencia en este ltimo examen. Recuerda que pocas
enfermedades generan osteolisis en las costillas. Estas suelen
La respuesta correcta es: 5
ser de origen hematolgico (opcin 5). El mesotelioma se
observa como un engrosamiento pleural (opcin 1).
COMENTARIO:
Los neumotrax (aire en cavidad pleural) pueden tener
Pregunta 49
diferentes etiologas. Pueden ser espontneos, traumticos,
Un paciente con un carcinoma brocognico en lbulo
hipertensivos y catameniales. Los espontneos pueden ser
inferior izquierdo es sometido a una mediastinoscopia de

9
primarios si no tienen enfermedad pulmonar previa o Pregunta 54
secundarios cuando s la presentan (EPOC). Espontneo Un paciente de 62 aos presenta de forma brusca una
primario es tpico de paciente fumador joven, alto y flaco, que hemihipoestesia termo-algsica del hemicuerpo derecho,
presentan habitualmente burbuja o blebs pleurales (opcin 5 as como hipoestesia de la hemicara izquierda, hemiataxia
correcta). izquierda y debilidad de los msculos de la masticacin.
Donde localizaremos la lesin?:
1. Mesencfalo lateral derecho.
Pregunta 52
2. Mesencfalo medial izquierdo.
Mujer de 25 aos que acude al Centro de Salud muy
3. Prutuberancia lateral izquierda.
alarmada por que al mirarse en el espejo se ha visto la
4. Protuberancia medial derecha.
boca torcida. Ese mismo da durante la comida ha notado
5. Bulbo medial derecho.
que los lquidos le resbalaban por la comisura labial. De
entre las siguientes, seale la respuesta correcta:
1. El comienzo sbito nos orienta a una parlisis facial distinta a La respuesta correcta es: 3
la de Bell.
2. La ausencia o disminucin del reflejo palpebral es un dato
COMENTARIO:
diagnstico muy especfico para diferenciar la parlisis de origen
El paciente presenta un sndrome cruzado con afectacin de la
perifrico de las de origen central.
sensibilidad de hemicuerpo D y la hemicara I, el dado afectado
3. La parlisis facial perifrica idioptica no es frecuente que se
nos lo da el par craneal (V p.c. en este caso), as que estamos
acompae de ageusia y de hiperacusia.
en troncoencfalo izquierdo (Eso nos descarta las opciones 1, 4
4. La presencia de otros sntomas, como disfagia o diplopa, es
y 5). El paciente adems presenta hemiataxia ipsilateral, por
compatible con el diagnstico de parlisis facial perifrica
lesin de pednculos cerebelosos (situados entre bulbo y
idioptica.
protuberancia) y cerebelo, y debilidad de los msculos de la
5. En ste, como en todos los casos de parlisis facial, est
masticacin (lesin del V p.c. motor situado en la
indicada la realizacin de un EMG.
protuberancia). De manera que nos encontramos en
protuberancia lateral izquierda.
La respuesta correcta es: 2
Pregunta 55
COMENTARIO: Hombre de 32 aos que hace 2 aos fue diagnosticado de
La parlisis facial es perifrica cuando existe una afectacin de esclerosis mltiple. Acude urgente porque desde hace 4
los msculos faciales inervados por la rama superior de nervio das presenta paraparesia severa y progresiva que le
facial (asimetra de arrugas frontales y cierre incompleto del dificulta el caminar. Que actitud terapetica adoptara en
prpado con el signo de Bell) y pueden aadirse sntomas por ese momento?:
afectacin de fibras motoras del msculo del estribo 1. Inmunoglobulibas intravenosas.
(hiperacusia ) y de sensibilidad gustativa de los dos tercios 2. Pulsos intravenosos de 1 gramo de Metilprednisolona.
anteriores de la lengua (ageusia), que tambin van en el nervio 3. Plasmafresis.
facial. La EMG predice la posible regeneracin del nervio 4. Rehabilitacin motora intensa.
paralizado, por ello es til para establecer el pronstico, tras 5. Copolmero I.
unas semanas de instaurarse, en los casos de mala evolucin.
La asociacin de otros pares craneales afectos orientaran a
La respuesta correcta es: 2
origen central y no idioptica.

COMENTARIO:
Pregunta 53
El paciente del caso clnico presenta un brote de esclerosis
En la enfermedad oclusiva del Sistema Arterial Vrtebro-
mltiple. Dentro del tratamiento de esta enfermedad, el
Basilar cual de los siguientes signos NO es
tratamiento sintomtico del brote son corticoides i.v. a altas
caracterstico?:
dosis (metilprednisolona 1 gr./da durante 3-5 das).
1. Menenginismo.
2. Debilidad de Miembros.
3. Ataxia de la marcha. Pregunta 56
4. Parlisis culomotora. Mujer de 24 aos diagnosticada de esclerosis mltiple
5. Disfuncin orofarngea. haca 2 aos, despus de sufrir un brote inicial de neuritis
ptica retrobulbar. Seale cul es el sntoma o el signo
que NO esperara encontrar en la paciente:
La respuesta correcta es: 1
1. Disartra.
2. Incontinencia urinaria.
COMENTARIO: 3. Fatiga Crnnica.
La oclusin del sistema arterial vertebrobasilar produce signos 4. Afasia.
de disfuncin del tronco del encfalo (Manual CTO tema 4, 5. Importante sensibilidad anmala al calor.
(4.1), pag. 19).
Estos signos de disfuncin pueden ser:
La respuesta correcta es: 4
- Debilidad de miembros (opcin 2): paresia por lesin de la va
piramidal en su camino descendente desde corteza hacia COMENTARIO:
mdula (Manual CTO, tema 1 (1.4), pag. 7) En una esclerosis mltiple no esperaramos encontrar afasia
- Ataxia de la marcha: por lesin cerebelosa y de pednculos (opcin 4), que es un signo de afectacin cortical, y hay que
cerebelosos (opcin. 3), irrigados por las arterias cerebelosas recordar que en la EM se produce afectacin de la sustancia
que dependen de la circulacin posterior (vertebrobasilar). blanca, produciendo una variedad de sntomas:
- Parlisis oculomotora: por lesin de PPCC oculomotores
(opcin 4) en protuberancia y mesencfalo. - Disartria: aparece tpicamente en los brotes cerebelosos (es
- (Opcin 5). Disfuncin orofaringea: por lesin de los pares muy caracterstica de la EM la disartria escandida).
craneales bajos (disfagia, disfona...) que se encuentran el - Incontinencia urinaria: muy caracterstica de los brotes
bulbo. medulares.
- Fatiga crnica: frecuentemente asociada a la E.M., sobre todo
con el ejercicio.

10
- Intoleracia al calor, que suele ser causa de los llamados alucinaciones visuales y auditivas e ideas delirantes
pseudobrotes. paranoides. Asimismo, durante el ltimo ao los dficif
cognitios progresivos se han vuelto cada vez ms
evidentes para ella y para otros miembros de la familia.
Estos dficif, todava en una fase leve, implican la
memoria, las habilidades matemticas, la orientacin y la
Pregunta 57 capacida de aprender nuevas habilidades. Aunque se han
Hombre de 70 aos que consulta por un trastorno de la observado fluctuaciones en la capacidad cognitiva da a
marcha y un deterioro cognitivo sabagudo. Nos indican da o semana a semana, es aparente un curso declive
que el diagnstico de presuncin del paciente es definido. El paciente no ha estado tomando ningn
hidrocefalia a presin normal. En este caso, cul de los medicamento. El examen fsico revela un temblor en
siguientes datos NO esperara encontrar?: reposo de la rigidez en rueda dentada. La marcha del
1. Una hidrocefalia comunicante con acueducto de Silvio paciente se caracteriza por oasos cortos que los arrastra y
permeable en la resonancia cerebral. una disminucin del balanceo de los brazos. Que
2. Un trastorno de la marcha tipo aprxico. sndrome clnico es ms compatible con los sntomas de
3. Un LCR con leve elevacin de la presin de apertura y con un este hombre?:
aumento de clulas y protenas. 1. Enfermadad de Parkinson.
4. Larealizacin de una puncin lumbar evacuadora (30 ml de 2. Demencia de Creutzfeldt-Jacob.
LCR) puede mejorar la marcha del paciente. 3. Demencia con cuerpos de Lewy.
5. Ausencia de signos de atrofia cortical cerebral. 4. Demencia vscular.
5. Demencia del lbulo frontal.
La respuesta correcta es: 3
La respuesta correcta es: 3
COMENTARIO:
La hidrocefalia crnica del adulto, hidrocefalia normotensiva o COMENTARIO:
hidrocefalia a presin normal es un tipo de hidrocefalia El paciente presenta un cuadro clnico caracterizado por la
comunicante o no obstructiva que en la mayora de los casos es triada de deterioro cognitivo, alucinaciones visuales y
idioptica, si bien puede ser secundaria a otros trastornos. En el parkinsonismo, con fluctuacin importante. Este cuadro clnico
caso de la idioptica, como en el caso de esta pregunta, afecta es tpico de la demencia con cuerpos de Lewy.
a personas mayores, a partir de los 60 aos. Tiene una clnica
muy caracterstica de apraxia de la marcha (sntoma ms
Pregunta 60
frecuente y el primero que suele aparecer), demencia y
Un hombre de 30 aos acude a su consulta por presentar
trastornos esfinterianos. En las pruebas de imagen se visualiza
debilidad muscular y diplopia fluctuantes, de un mes de
un aumento del tamao de todas las cavidades ventriculares; a
evolucin, que usted objetiva con la exploracin fsica.
diferencia de la hidrocefalia `ex vacuo o secundaria a atrofia
Cul de las siguientes pruebas NO le parecera oportuno
cerebral, este tipo de hidrocefalia suele presentar hipodensidad
solicitar:
periventricular, abombamiento de astas frontales y compresin
1. Electromiografa de fibra muscular.
de los surcos de la convexidad. El diagnstico se complementa
2. Electromiografa con estimulacin repetitiva.
con un registro continuo de presin intracraneal, en el que se
3. Estudio de funcin autonmica.
observa un aumento de la presin y/o la presencia de ondas
4. Determinacin de anticuerpos anti-receptor de aceticolina.
patolgicas de Lundberg, un test de infusin, en el que se
5. TAC torcico.
evidencia un aumento de la resistencia a la salida de lquido
cefalorraqudeo, y una prueba de evacuacin de lquido
cefalorraqudeo, en la se comprueba una mejora clnica del La respuesta correcta es: 3
paciente ante la extraccin de una pequea cantidad del
mismo. El tratamiento suele consistir en la colocacin de una
COMENTARIO:
derivacin ventriculoperitoneal.
El paciente tiene una sintomatologa que nos debe hacer pensar
en miastenia gravis, como 1 posibilidad, incluyendo en su
Pregunta 58 estudio:
Debe dudar de un diagnstico de enfermedad de
Parkinson si en la esploracin se encuentra: - EMG de fibra muscular aislada ms estimulacin repetitiva,
1. Temblor de reposo. buscando datos de fatigabilidad.
2. Temblor postural. - Determinacin de Ac anti-R de acetilcolina (es una enf.
3. Parpadeo aumentado. Autoinmune, y podemos determinar los Ac. en laboratorio)
4. Micrografa. - TC torcico: buscando timoma, frecuentemente asociado a M.
5. Reflejos de estiramiento muscular hiperactivos. Gravis. No sera necesario realizar estudio de la funcin
autonmica, que no se altera en la M. Gravis.
La respuesta correcta es:
Pregunta 61
Un episodio caracterizado por sensacin epigstrica que
COMENTARIO:
asciende hacia el trax, seguido por dificultad para
Nos hara dudar una enfermedad de Parkinson la presencia de
conectar con el entorno, movimientos de masticacin,
parpadeo aumentado, ya que la bradicinesia se manifiesta de
distonia de una mano y falta de respuesta, de un minuto
forma muy frecuente con disminucin franca de la velocidad de
de duracin, con amnesia postcrtica, es una crisis:
parpadeo, contribuyendo a la tpica expresin del parkinsonismo
1. Parcial simple.
llamada cara de poker. Tambin nos hara dudar la presencia
2. Parcial secundariamente generalizada.
de reflejos miotticos hiperactivos, que sugieren piramidalismo,
3. Parcial compleja.
y por tanto otras formas de parkinsonismo (atrofia sistmica
4. Ausencia tpica.
mltiple, parkinsonismo vascular...)
5. Ausencia atpica.

Pregunta 59
La respuesta correcta es: 3
Un hombre de 77 aos es trado a la consulta por su
esposa para evaluacin. Ella refiere que durante los
ltimos seis meses su marido ha experimentado fuertes

11
COMENTARIO: inicia de forma progresiva alteracin de funciones
Se trata de una crisis parcial compleja: la clnica caracterstica superiores, incontinencia urinaria ocasional y su caminar
es: desconexin con el medio que puede estar precedido por un es torpe. Probablemente presenta:
aura (en este caso malestar epigstrico ascendente ), 1. Hematoma intraparenquimatoso cerebral tardo.
movimientos automticos (chupeteo, masticacin y breve 2. Hemorragia subaracnoidea.
periodo post-crtico). 3. Hidrocefalia arreabasortiva.
4. Atrofia cerebral postraumtica.
5. Tumor cerebral.
Pregunta 62
Un paciente de 62 aos presenta una historia de
cervicalgia irradiada a hombros. Desde hace 1 ao La respuesta correcta es: 3
presenta dificultad progresiva para caminar aadindose
dolor en brazo derecho. A la exploracin presenta u reflejo
COMENTARIO:
bicipital abolido y unos reflejos osteotendinosos
La hidrocefalia postraumtica se trata de una de las
policinticos en piernas.
complicaciones tarda de un traumatismo craneoenceflico. Un
1. Creo que tiene un tumor medular y le solicitara una
4% de los traumatismos craneoenceflicos severos presenta
RNM cervical.
este trastorno. No es ms que uno de los tipos secundarios de
2. Creo que tiene una hernia discal con espondilosis y le
hidrocefalia crnica del adulto que veamos en la pregunta 57.
solicitara RNM cervical.
La clnica, el diagnstico y el tratamiento es similar a lo
3. Probablemente tiene una siringomielia y le solicitara
comentado en la pregunta 57. En la pregunta se hace referencia
una RNM.
a la hidrocefalia arreabsortiva, ya que en el test de infusin se
4. Creo que tiene espondilosis cervical y le solicitara un
evidencia una disminucin de la reabsorcin o un aumento de la
TAC de columna cervical.
resistencia a la salida de lquido cefalorraqudeo.
5. Creo que un estudio radiolgico simple de columna y un
tratamiento con AINES es lo adecuado.
Pregunta 65
Una de las siguientes afirmaciones NO es correcta con
La respuesta correcta es: 2
respecto al cncer diferenciado de tiroides:
1. El carcinoma papilar es un tumor derivado del epitelio
COMENTARIO: folicular generalmente encapsulado, y con alta frecuencia de
En el tratamiento de la migraa, diferenciamos: Tratamiento diseminacin hematgena.
sintomtico durante el episodio: 2. El carcinoma folicular es un tumor diferenciado de tiroides,
habitualmente unicntrico y en el que la afectacin de los
- Triptanes: en caso de dolor moderado severo. ganglios linfticos es poco frecuente.
- Ergotamina: como alternativa a los triptanes. 3. El carcinoma papilar es el cncer ms frecuente del tiroides y
- AINES (naproxeno, por ej) en caso de dolor leve moderado. su pronstico es mejor que el de carcinoma folicular.
4. La PAAF permite diagnosticar el carcinoma papilar de tiroides
Tratamiento preventivo si hay > 2 episodios/mes, con distintos pero no suele ser de utilidad en el diagnstico del carcinoma
frmacos, entre ellos la flunaricina (antagonista del calcio), y el folicular de tiroides.
propranolol (beta-bloqueante). La ergotamina no es un 5. El carcinoma papilar de tiroides es ms frecuente en zonas
tratamiento profilctico, de manera que no est indicado su uso ricas en yodo mientras que el carcinoma folicular de tiroides es
diario como tal. La opcin 1 en la que pone tomar triptanes ms frecuente en zonas con dficit del mismo.
durante todos los ataques, sera correcta si todos los ataques
son de intensidad leve moderada.
La respuesta correcta es: 1

Pregunta 63
COMENTARIO:
Mujer de 34 aos diagnosticada de migraa sin aura que
Esta pregunta resulta sencilla conociendo las caractersticas
consulta por episodios de sus cefaleas habituales en
generales de los tumores malignos de tiroides. El carcinoma
nmero de 4-5 al mes. Cul de estos tratamientos NO
papilar es el ms frecuente y benigno. Deriva del epitelio
estara indicado?:
folicular, suele estar encapsulado y en raras ocasiones produce
1. Tomar triptantes durante todos los ataques.
metstasis a distancia. La diseminacin hematgena es poco
2. Utilizar dosis bajas diarias de ergotamina.
frecuente y aparece con ms incidencia en zonas ricas en yodo.
3. Administrar como profilaxis propanolol. 4 Tratar todos los
El carcinoma folicular presenta peor pronstico, aunque suele
ataques agudos con naproxeno.
ser unicntrico con escasa afectacin linftica, siendo ms
5. Utilizar como profilaxis flunaricina.
frecuente su aparicin en zonas deficitarias en yodo. Recordad
que la PAAF es poco til para el diagnstico del carcinoma
La respuesta correcta es: 2 folicular, siendo precisa la realizacin de una biopsia.

COMENTARIO: Pregunta 66
En el tratamiento de la migraa, diferenciamos: Seale la respuesta FALSA referido a la Tiroiditis
Tratamiento sintomtico durante el episodio: subaguada (de Quervain):
- Triptanes: en caso de dolor moderado severo. 1. Los Antiinflamatorios no esteroideos son un pilar del
- Ergotamina: como alternativa a los triptanes. tratamiento.
- AINES (naproxeno, por ej) en caso de dolor leve moderado. 2. Los B-bloqueantes son tiles si existe tireotoxicosis.
Tratamiento preventivo si hay > 2 episodios/mes, con distintos 3. Cuando existen sntomas importantes, locales o sistmicos,
frmacos, entre ellos la flunaricina (antagonista del calcio), y el es til aadir corticoides.
propranolol (beta-bloqueante). La ergotamina no es un 4. El Propiltiouracilio y otros antitiroideos deben usarse si existe
tratamiento profilctico, de manera que no est indicado su uso tireotoxicosis.
diario como tal. La opcin 1 en la que pone tomar triptanes 5. La velocidad de sedimentacin globular es
durante todos los ataques, sera correcta si todos los ataques caractersticamente normal.
son de intensidad leve moderada.
La respuesta correcta es: 4
Pregunta 64
Un paciente de 73 aos sufri un accidente de trfico con
traumatismo craneal del que se recuper. A los tres meses

12
COMENTARIO: 3. El cortisol mantiene su ritmo circadiano pero a un nivel ms
En esta pregunta existen 2 opciones incorrectas. La tiroiditis de alto que el normal.
De QUERVAIN, se caracteriza clnicamente por sntomas de 4. El cortisos libre en orina es menos til para el diagnstico
hipertiroidismo con dolor a la palpacin en la glndula tiroidea, que el cortisol basal en sangre.
asociando fiebre habitualmente. Analticamente, se confirma el 5. El tamao normal de las glndulas suprarrenales en una TAC
hipertiroidismo primario, y es tpica la elevacin de la VSG abdominal descarta el diagnstico.
(opcin 5 falsa). Sin embargo, dado que el hipertiroidismo de
este cuadro se produce por destruccin glandular, los
La respuesta correcta es: 2
antitiroideos, que inhiben la sntesis de hormonas tiroideas, no
son de utilidad. Por ese motivo, la opcin 4 es tambin falsa. El
resto de opciones no ofrecen ninguna duda. COMENTARIO:
Pregunta muy sencilla en relacin al SD. CUSHING endgeno.
La respuesta 2 es correcta, ya que sabis que la confirmacin
Pregunta 67
del diagnstico requiere la ausencia de supresin del cortisol
Respecto a la insuficiencia suprarenal primaria NO es
tras administrar dexametasana (Test supresin 2 mgr). En el
cierto que:
Cushing endgeno, la ACTH puede estar elevada (origen central
1. La infeccin por citomegalovirus afecta habitualmente a las
o ectpico) o suprimida (origen suprarrenal). Es caracterstica la
glndulas suprarrenales produciendo la llama adrenalitis
ausencia del ritmo de cortisol y solemos emplear el CLU 24h
"necrosante".
como test de despistaje. Por ltimo, aunque en el Cushing
2. La insuficiencia suprarenal de los pacientes con SIDA se
ACTH dependiente podemos encontrar aumento de tamao de
manifiesta frecuentemente en forma de insuficiencia
las suprarrenales, la no existencia del mismo no descarta en
suprarrenal aguda grave.
absoluto la existencia de este sndrome.
3. La astenia es el sntoma principal.
4. Afecta igualmente a ambos sexos.
5. Cuando la destruccin de las glndulas es rpida, como en la Pregunta 70
hemorragia suprarrenal bilateral, no suele haber Cul de los siguientes enunciados NO corresponde al
hiperpigmentacin. feocromocitoma?:
1. Tratamiento de la crisis hipertensa con nitroprusiato.
2. Fallo cardiaco sin causa aparente durante una gestacin.
La respuesta correcta es:
3. Crisis hipertensa desencadenada por frmacos hipotensores.
4. Flushing, diarreas y 5HIAA elevado en orina.
COMENTARIO: 5. Metanefrinas en orina de 24 h. elevadas.
Esta pregunta ofrece varias respuestas que son falsas. Por un
lado, es cierto que el CMV puede producir adrenalitis
La respuesta correcta es: 4
necrotizante, pero esta afectacin es muy infrecuente (opcin 1
falsa) como causa de insuficiencia suprarrenal primaria. Por
otro lado, la crisis suprarrenal aguda en pacientes con SIDA es COMENTARIO:
una forma de presentacin bastante infrecuente, luego la Esta pregunta hace referencia a un tema muy explicado en
opcin 2 tambin es falsa. Finalmente, aunque en la ltima clase como es el feocromocitoma. Esta patologa se caracteriza
edicin del Harrison aparece que la incidencia de esta por la existencia de un tumor productor de catecolaminas.
enfermedad es similar en ambos sexos, en cualquier tratado de Clnicamente se caracteriza por la existencia de crisis
Endocrinologa figura que es ms frecuente en mujeres (opcin hipertensivas con sntomas adrenrgicos asociados; que pueden
4 falsa). ser desencadenadas por frmacos hipotensores (alfa-
metildopa). En gestantes con fallo cardaco se debe incluir en el
diagnstico diferencial, aunque esta forma de presentacin sea
Pregunta 68
extraordinaria. Su diagnstico se realiza confirmando la
En relacin con el hiperaldosteronismo primario cul de
elevacin de catecolaminas y metanefrinas en orina de 24 h. La
estas afirnmaciones NO es correcta?:
elevacin del 5-HIAA junto a la aparicin de flushing y diarreas
1. Ocasionalmente son tumores bilaterales.
es tpico del Sd. Carcinoide (opcin 4 falsa).
2. Es caracterstica la presencia de hipertensin e hipokalemia.
3. El tratamiento mdico se realiza con espironolactona.
4. A pesar de su condicin benigna, el tratamiento ideal es Pregunta 71
quirrgico independientemente del tamao o de que la lesin La retinopata diabtica proliferactiva:
sea unilateral o bilateral. 1. Solo se presenta en pacientes con diabetes tipo 1.
5. la relacin entre la aldosterona y la reinba est alterada. 2. Es la forma de la retinopata en pacientes diabticos mal
controlados.
3. Es frecuente que provoque hemorragias de vitreo.
La respuesta correcta es: 4
4. Se caracteriza por la presencia de microaneurismas.
5. Suele aparecer despus de los 60 aos de edad.
COMENTARIO:
Pregunta sencilla en relacin al hiperaldosteronismo primario.
La respuesta correcta es: 3
Este cuadro se caracteriza clnicamente por la existencia de
hipertensin con hipopotasemia, sin edemas. La causa ms
frecuente es la existencia de un tumor productor de aldosterona COMENTARIO:
(SD. CONN), aunque puede deberse a hiperplasias suparrenales La retinopata diabtica proliferativa constituye la fase final de
uni o bilaterales. Para el diagnstico se emplea el cociente esta enfermedad, y aparece tanto en la diabetes tipo 1 como en
ALD/ARP, que se encuentra aumentado. En relacin al la tipo 2. Ante la isquemia la retina secreta algunos factores de
tratamiento, la ciruga es de eleccin en el caso del adenoma, crecimiento que favorecen la aparicin de los neovasos. Por lo
mientras que en la hiperplasia bilateral se prefiere el tanto es la presencia de neovasos, y no de microaneurismas lo
tratamiento farmacolgico, con espironolactona (opcin 4 que define esta fase de la enfermedad. La presencia de
falsa). neovasos da lugar a muchas complicaciones. Recuerda que las
ms importantes son el hemovtreo, el desprendimiento de
retina traccional y el glaucoma neovascular. El factor ms
Pregunta 69
importante que determina su aparicin, es el tiempo de
En el diagnstico del sndrome de Cushing endgeno:
evolucin de la enfermedad, pero no puede establecerse una
1. La ACTH debe estar elevada para confirmar el diagnstico.
determinada edad frontera, entre otras cosas, porque
2. Debe confirmarse una falta de supresin del cortisol tras
depender del momento de aparicin de la diabetes.
dexametasona.

13
Pregunta 72 aparece la prealbmima (2 das) y la protena ligada al retinol
En la relacin a la diabetes tipo 2: (10 horas).
1. Los anticuerpos anti-decarboxilasa del cido glumtico son
un marcador precoz de la enfermadad.
Pregunta 75
2. Los virus son capaces de iniciar el proceso de destruccin de
Un paciente de 76 aos es ingresado con una masa
la clula beta propio de la enfermedad.
pulmonar y una cifra de calcio srico de 16mg/dl. Cul es
3. No existe alteracin en la secrecin de la insulina.
la primera media teraputica a adoptar?:
4. Existe un aumento en la produccin heptica de glucosa en
1. Pamidronato intravenoso
ayunas y postingesta.
2. Alendronato oral.
5. No existe resistencia a la insulina.
3. Furosemida.
4. Suero salino.
La respuesta correcta es: 4 5. Cacitonina.

COMENTARIO: La respuesta correcta es: 4


Pregunta muy sencilla en relacin con la patogenia de la
diabetes mellitus tipo 2. La posible relacin entre las infecciones
COMENTARIO:
virales y el desencadenamiento de la destruccin de las clulas-
Pregunta muy sencilla en relacin al tratamiento de la
beta, junto a la positividad de los anticuerpos anti-GAD
hipercalcemia grave. La primera medida a tomar en estos
caracteriza a la diabetes mellitus tipo 1. En la diabetes tipo 2
pacientes, muy recalcado en las clases, es la hidratacin
existe habitualmente resistencia insulnica y un defecto en la
abundante con suero salino. Despus de la hidratacin,
secrecin de la misma. Es tpica en esta patologa el aumento
emplearemos frmacos que resultan de utilidad como la
de la neoglucognesis heptica, como figura en la opcin 4.
furosemida y los bifosfonatos intravenosos. Los glucocorticoides
tardan en actuar y slo son de utilidad en las hipercalcemias
Pregunta 73 tumorales y las producidas por exceso de vitamina D.
Varn de 70 aos que acude al Hospital de Da de
Endocrinologa tras hallazgo en analtica de control de una
Pregunta 76
glicemia de 456. El paciente refiere polidipsia y poliuria as
En un hombre de 28 aos operado y curado hace 3 aos de
como prdida de 15 kg de peso en los ltimos 3 meses.
acromegalia, en el que se detecta hipercalcemia, con
Presenta un IMC de 22kg/m2. La actitud inicial que
fosforemia baja y PTH elevada y la gammagrafa con
deberamos tomar es:
Setamibi a los 150 minutos (2 fase de la prueba, tras el
1. Repetir la glucemia en ayunas o realizar test tolerancia oral o
lavado de la actividad tiroidea) demuestra captacin
la glucosa para confirmar el diagnstico.
positiva en 3 puntos de la regin anterocervical, el
2. Iniciar tratamiento con sulfonilureas y revaluar en 6 meses.
diagnstico ms probable es:
3. Lo ideal es empezar un tratamiento combinado con acarbosa
1. Carcinoma medular de tiroides.
y metformina a dosis plena.
2. Osteomalacia.
4. Inicialmente debemos pautar tratamiento diettico y ejercicio
3. Insuficiencia renal crnica.
fsico, valorando aadir algn frmaco segn la evaluacin
4. Neoplasia endocrina mltiple.
posterior.
5. Bocio multinodular txico.
5. En este momento lo ms adecuado es iniciar insulinoterapia
junto con tratamiento diettico y modificacin en el estilo de
vida. La respuesta correcta es: 4

La respuesta correcta es: 5 COMENTARIO:


En esta pregunta nos presentan un caso clnico de un varn
joven diagnosticado de acromegalia y de hiperparatiroidismo
COMENTARIO:
primario por una hiperplasia paratiroidea. Esta asociacin,
Caso clnico relacionado con el tratamiento de la diabetes
claramente descrita en el manual y muy remarcada en clase, se
mellitus. Los datos que deben orientar a la respuesta correcta
presenta en la neoplasia endocrina mltiple tipo 1. Adems
son la existencia de hipoglucemia severa junto a sntomas
estos pacientes suelen asociar tumores pancreticos (respuesta
cardinales importantes, en especial la prdida de peso, que
correcta 4).
sugieren que el paciente presenta insulinopenia. Aunque el
diagnstico ms probable sea el de diabetes mellitus tipo 2,
tanto la hiperglucemia como la clnica severa, hacen Pregunta 77
recomendable iniciar de momento un tratamiento con dieta, Cul de las siguientes alteraciones NO es un criterio para
modificacin del estilo de vida e insulinoterapia (opcin correcta el diagnstico del Sndrome metablico?:
5). 1. Nivel de triglicridos mayor de 149mg/dl
2. Nivel de HDL-Colesterol menor de 40 mg/dl en varones.
3. Tensin arterial de 130/85 mm Hg o superior.
Pregunta 74
4. Glucosa en ayunas mayor de 109mg/dl.
La vida media de la albmina es de:
5. Nivel de LDL-Colesterol mayor de 130mg/dl.
1. 8 das.
2. 2 das.
3. 20 das. La respuesta correcta es: 5
4. 10 dis.
5. 1 mes.
COMENTARIO:
Pregunta muy difcil en relacin al concepto de Sd. Metablico.
La respuesta correcta es: 3 Lo primero a sealar, es que el concepto del sndrome es
distinto en algunos detalles segn los criterios que empleemos
(NECP u OMS), y al no especificarse en la pregunta, creemos
COMENTARIO:
que puede ser impugnada. Es cierto que las cifras LDL no se
Pregunta difcil, dado que puede pasarse por alto, aunque viene
emplean en la definicin en ninguna de estas 2 clasificaciones, y
reflejada claramente en el manual. Para la valoracin del estado
sera pues la opcin ms incorrecta. Si empleramos los
nutricional empleamos diversas protenas. Dentro de las
criterios de la OMS, las opciones 3, 4 y 5 seran falsas. Segn
protenas de vida media larga encontramos la albmina (20
los de la NECP, no habra dudas y la opcin 5 sera la falsa.
das) y transferrina (10 das). Entre las de vida media corta

14
Pregunta 78 Pregunta 81
Cul de las siguientes substancias produce mayor Todas las siguientes son manifestaciones de la
elevacin de las lipoprotenas de alta densidad espondilitis anquilosante (anquilopoytica) EXCEPTO:
transportadoras de colesterol (HDL-col): 1. Dolor lumbar y rigidez de ms de 3 meses que mejora con el
1. cido nicotnico. ejercicio pero no con el reposo.
2. Estatinas. 2. Sacroileitis radiolgica bilateral.
3. Fibratos. 3. Uveitis aguda anterior.
4. Inhibidores de la reabsorcin. 4. Insuficiencia artica.
5. Resinas. 5. Vasculitis de pequeos vasos.

La respuesta correcta es: 1 La respuesta correcta es: 5

COMENTARIO: COMENTARIO:
Pregunta muy difcil en relacin a los frmacos hipolipemiantes. La pregunta describe las principales caractersticas de la
El cido nicotnico, que acta inhibiendo la movilizacin de los enfermedad. Su principal manifestacin clnica (dolor y rigidez
cidos grasos y la sntesis de VLDL, es rara vez empleado en la lumbar de carcter inflamatorio), su ineludible hallazgo
clnica habitual por sus efectos secundarios. Su principal radiolgico (sacroileitis, habitualmente bilateral), y dos de sus
indicacin es la hipertrigliceridemia y caractersticamente, es el manifestaciones extrarticulares, la ms frecuente (uveitis
frmaco que ms aumenta la concentracin de HDL. Los anterior aguda) y la insuficiencia artica. Es sustrato
fibratos, que se emplean con mucha frecuencia en la anatomopatolgico de la enfermedad no es la inflamacin
hipertrigliceridemia, tambin elevan las HDL, aunque en menor vascular.
medida que el cido nicotnico. Las estatinas, resinas y los
inhibidores de la absorcin, tienen escaso efecto sobre las HDL.
Pregunta 82
Mujer de 42 aos de edad que presenta desde hace 6
Pregunta 79 meses, en relacin con el fro, episodios de palidez en los
Cul de los siguientes frmacos se considera hoy en da dedos seguida de enrojecimientos, refiere artralgias en los
de eleccin en el tratamiento de la artritis reumatoide?: dedos de las manos, tirantez de la piel en brazos y cara
1. Sales de oro. con disfagia frecuente con los alimentos slidos. As
2. Metrotexate. mismo refiere tos seca, disnea y crepitantes en bases
3. D-panicilamina. pulmonares. Respecto al cuadro clnico de la paciente es
4. Ciclosporina. FALSO que:
5. Glucocorticoides. 1. El fenmeno de Raynaud es la manifestacin inicial en el
100% de las formas limitadas y en ms del 70% de las difusas.
2. La aparicin de anticuerpos antitopoisomerasa I (anti-Scl-70)
La respuesta correcta es: 2
predispone a un aumento de la fibrosis drmica.
3. Los bloqueantes del calcio son eficaces en el tratamiento del
COMENTARIO: Fenmeno de Raynaud.
La medida ms trascendente en el manejo de la artritis 4. Las pruebas de funcin respiratoria muestran un patrn
reumatoide es el tratamiento de fondo y entre los restrictivo con disminucin de la difusin de CO.
mencionados en la pregunta el de eleccin por eficacia, 5. El esfago se afecta en el 75-90% de los pacientes, con
comodidad y seguridad es hoy en da sin duda el metotrexate. hipomitilidad del esfinter y de los dos tercios inferiores del
Las sales de oro, D-penicilamina y la ciclosporina son frmacos esfago.
cuyo uso actualmente se ha abandonado en la AR. Los
corticoides no se consideran modificadores de la enfermedad,
La respuesta correcta es: 2
se utilizan para obtener control de los sntomas hasta que los
tratamientos de fondo ejercen su efecto y lo deseable es
utilizarlos el menor tiempo posible y a la menor dosis eficaz. COMENTARIO:
La descripcin permite identificar una esclerosis sistmica
progresiva y el hecho de que se afecte la piel de los antebrazos
Pregunta 80
(proximal a los dedos por lo tanto) y la presencia de sntomas
Hombre de 80 aos que en una anltica se le detectan
respiratorios sugestivos de fibrosis (crepitantes) nos indica que
unas fosfatasas alcalinas aumentadas. Vd. piensa en una
se trata de una forma con afectacin cutnea difusa. El
enfermedad sea de Paget. Que prueba es la ms ptima
fenmeno de Raynaud es prcticamente constante en la forma
para darnos la idea de la extensin de la enfermedad?:
limitada y suele estar presente desde aos antes del comienzo
1. Resonancia holocorporal.
de la clnica cutnea. En cambio en la forma difusa el Raynaud
2. Gammagrafa sea con Tc 99.
es algo menos frecuente y de reciente aparicin por lo que en
3. Rx seriada sea.
estas formas es menos comn que el fenmeno de Raynaud sea
4. Gammagrafa con emisin de positrones.
la manifestacin inicial de la enfermedad. Los anticuerpos
5. Hidroxiprolinuria en orina de 24 horas.
antitopoisomerasa se asocian a las formas con afectacin
cutnea difusa (y por lo tanto a las formas con afectacin
La respuesta correcta es: 2 cutnea mas extensa) sin embargo la manifestacin visceral a
la que se asocian es a la fibrosis pulmonar. Las dems opciones
no ofrecen dudas ya que los calcioantagonistas son frmacos
COMENTARIO: tiles frente al fenmeno de Raynaud, la afectacin pulmonar
Para el diagnstico de la enfermedad de Paget la radiologa provoca alteraciones de las pruebas de funcin respiratoria
simple es suficiente ya que identifica los cambios caractersticos como la difusin del CO, y tras la piel el rgano ms
(aumento del tamao del hueso, engrosamiento de la cortical, frecuentemente afectado es el esfago.
trabculas groseras y coexistencia de lesiones lticas y
blsticas). Sin embargo,dado que la enfermedad a menudo es
asintomtica para conocer la extensin d la enfermedad Pregunta 83
debemos realizar una tcnica ms sensible (aunque menos Una mujer de 70 aos, previamente bien, consulta al
especfica) como la Gammagrafia osea. La hidroxiprolina mdico por una prdida aguda de visin del ojo derecho,
urinaria refleja la presencia de reabsorcin sea pero no es til siendo diagnosticada de neuritis ptica isqumica
par cuantificar la extensin de la enfermedad. anterior. Se recoge en su historia la existencia de una
hipertensin bien controlada y un cuadro de cefalea

15
parietal en las ltimas semanas. Una semana antes, un intrarticulares . La regla de oro en el manejo de la artritis
estudio analtico mostraba una VSG de 100 mm a la 1 gotosa aguda es no modificar la uricemia, de forma que si se
hora. Indique la conducta inmediata ms correcta: trata del primer episodio no hay que iniciar tratamiento con
1. Solicitar una tomografa axial computarizada cerebral. alopurinol y si el paciente ya estaba tomando alopurinol hay
2. Comenzar con tratamiento con Clopidogreal y controlarlo a que mantenerlo sin modificar la dosis.
los 3 meses.
3. Administrar heparina de bajo peso molecular, mientras se
Pregunta 86
llega a un diagnstico definitivo.
Paciente de 45 aos de edad acude a consulta por un
4. Administrar prednisona, al menos 1 mg/kg de peso, hasta
cuadro de 5 meses de evolucin de secrecin nasal
concluir el diagnstico.
purulenta, tos con expectoracin hemoptoica y lesiones
5. Realizar una retinografa de urgencia.
ulceradas en encas. En el control analtico destaca un
creatinina de 2,3 mg/dL, y ANCAs positivos. Que
La respuesta correcta es: 4 tratamiento sera el ms adecuado para nuestro
paciente?:
1. Metilprednisola endovenosa a dosis de 1mg/kg de peso hasta
COMENTARIO:
mejora clnica.
La cefalea es la manifestacin ms frecuente y la neuritis ptica
2. Mantener en observacin sin tratamiento.
isqumica es la manifestacin ms grave de la Arteritis de la
3. Ciclofosfamida y prednisona.
Temporal. La edad avanzada de la paciente y la marcada
4. Azatioprina y prednisona.
elevacin de la VSG permiten identificar el cuadro clnico sin
5. Clasmafresis y ciclofosfamida.
problemas. Dada la presencia de afectacin ocular es necesario
instaurar tratamiento lo antes posible (corticoide a dosis altas-
prednisona 1 mg/kg/dia). Posteriormente habr que confirmar La respuesta correcta es: 3
el diagnstico mediante una biopsia de arteria temporal, que
habra que realizar lo antes posible, ya que el tratamiento con
COMENTARIO:
corticoides negativizar los hallazgos anatomopatolgicos en 1-
El enunciado menciona todos los hallazgos tpicos del Wegener
2 semanas.
(afectacin de la va area superior con secrecin nasal
purulenta, afectacin pulmonar con hemoptisis, y deterioro de
Pregunta 84 la funcin renal. Adems la presencia de c-ANCA apoya an
Un hombre de 30 aos, sufre un cuadro gripal, mas el diagnstico de la enfermedad. La medida terapetica
acompaado de lesiones cutneas, nodulares de que ms ha modificado favorablemente el pronstico del
coloracin rojo-violcea, en cara anterior de ambos Wegener el la administracin de Ciclofosfamida que se
miembros inferiores. Tratado con paracetamol, las complemente con corticoides (p. ej prednisona) a dosis altas.
lesiones ceden parcialmente. A los tres meses vuelve a la
consulta con un cuadro de poliartrargias, junto con fiebre
Pregunta 87
de hasta 38C y empeoramiento de sus ndulos cutneos.
Un paciente de 7 aos de edad, ingresado en
Ante la existencia de una elevacin de la velocidad de
traumatologa desde hace 6 horas tras haber reducido
sedimentacin y de la Protena C reactica, inicia
ortopdicamente una fractura supracondlea de hmero,
tratamiento con aspirina y a la semana sufre una parlisis
avisa quejndose de dolor intenso en el miembro,
facial derecha, perifrica. Cul entre los siguientes, sera
parestesias en la mano y dificultad para movilizar los
el diagnstico ms probable?:
dedos. Ante este cuadro, lo primero que debemos realizar
1. Fiebre botonosa mediterrnea.
es:
2. Brucelosis.
1. Una radiografa de codo.
3. Fiebre recurrente.
2. Retirar la escayola.
4. Enfermedad de Lyme.
3. Una analtica con iones.
5. Sarcoidosis.
4. Administrar un analgsico.
5. Sujetar el brazo con una charpa.
La respuesta correcta es: 5
La respuesta correcta es: 2
COMENTARIO:
La presencia de eritema nodoso, fiebre y poliartralgias se
COMENTARIO:
vuelve muy sugestiva de sarcoidosis al aadirse a la clnica la
El aumento de presin en un compartimento osteofascial
parlisis facial. No obstante la duda podra presentarse con la
condiciona una clnica de dolor intenso, parestesias distales y
enfermedad de Lyme en la que aparecen fiebre, lesiones
dificultad para movilizar los dedos de la extremidad. En nios,
cutneas y parlisis facial. La patocronia y el tipo de lesiones
la causa ms frecuente para el desarrollo de un sndrome
cutneas no son propias de la enfermedad de Lyme.
compartimental es la fractura supracondlea de hmero. El
primer gesto teraputico a realizar ante la sospecha clnica en
Pregunta 85 estos casos es la retirada de vendajes o yesos, manteniendo la
Para el tratamiento de un ataque de gota agudo se extremidad afecta elevada para evitar el aumento del edema. Si
pueden utilizar todos los siguientes frmacos EXCEPTO: el cuadro no cede, es necesaria la apertura quirrgica urgente
1. La indometacina. del compartimento o compartimentos afectos mediante
2. Los glucocorticoides. fasciotomas.
3. El naproxeno.
4. El alopurinol.
Pregunta 88
5. La colchicina.
Si un paciente presenta imposibilidad para la flexin
aislada del cuarto dedo, imposibilidad para la abduccin
La respuesta correcta es: 4 adducin de los dedos segundo a quinto e hipotesia de
cara volar de 4 y 5 dedos, siendo el resto normal. Que
estructuras anatmicas sospecharemos que estarn
COMENTARIO:
lesionadas?:
En la artritis gotosa aguda el tratamiento se realiza con reposo
1. N. mediano y flexor profundo de 4 dedo.
de la articulacin afectada, AINEs a dosis altas y lo antes
2. N. mediano y flexor superficial de 4 dedo.
posible en el curso de la enfermedad, Colchicina y si la
3. N. cubital y flexor de 4 dedo. 4 N. radial y flexor superficial
articulacin es lo suficientemente grande corticoides

16
de 4 dedo. COMENTARIO:
5. N. radial y flexor profundo de 4 dedo. Presentan el caso clnico de un traumatismo indirecto de la
rodilla. Para responder este tipo de caso clnico hay que prestar
atencin siempre a tres puntos. El mecanismo de produccin, el
La respuesta correcta es: 3
derrame articular y la exploracin clnica. En este caso, slo nos
indican el mecanismo de produccin (un giro brusco) y el
COMENTARIO: derrame articular (intenso y de rpida aparicin). La lesin del
Nos presentan el caso clnico de una lesin nerviosa y ligamento cruzado anterior puede producirse tanto por un
tendinosa. Para responder correctamente cul es la lesin mecanismo de hiperextensin de la rodilla como por un giro
nerviosa hay que atender a dos puntos: la alteracin sensitiva y forzado, normalmente asociado a valgo. El giro forzado de la
la alteracin motora. El nervio cubital inerva sensitivamente al rodilla tambin puede producir una lesin meniscal. El derrame
5 dedo y a la mitad cubital del 4, motoramente a nivel de la en la lesin del ligamento cruzado anterior es de aparicin
mano inerva la mayor parte de la musculatura intrnseca, sta rpida (normalmente una o dos horas) y abundante, a
es la encargada de realizar la abduccin y adduccin de los diferencia del derrame producido por patologa meniscal que es
dedos. La lesin tendinosa que presenta el paciente evita que escaso y de aparicin tarda (18-24 horas). Las lesiones de los
haga una flexin aislada del 4 dedo. Este movimiento lo realiza ligamentos colaterales no cursan con derrame articular.
el flexor superficial del 4 dedo, que se explora manteniendo los
dedos adyacentes en extensin y pidiendo al paciente que
Pregunta 91
realice una flexin del dedo libre. Sin el bloqueo de los dedos
A la consulta nos traen una nia de 7 aos de edad porque
adyacentes la flexin puede realizarse a expensas del flexor
su abuewla le ha apreciado un arco plantar excesivo. Al
profundo de los dedos.
explorarle apreciamos en ambos pies, efectivamente, un
arco plantar longitudinal excesivo con apoyo exclusivo en
Pregunta 89 taln y cabezas de metatarsianos, as como un leve varo
Un hombre de 45 aos, cuyos nicos antecedentes son de retropi. Cul debera ser nuestra siguiente actitud?:
una hipercolesterolemia sin control y una ingesta de 1. Exploracin neurolgica detallada.
aproximadamente 100 gramos de alcohol diarios, consulta 2. Es la actitud normal del pie a esa edad.
por dolor muy intenso en el muslo de inicio brusco hace 2 3. Resonancia magntica y/o TAC para descartar coaliciones
semanas, sin antecedente traumtico. El paciente camina tarsales.
a duras penas con marcada cojera y ayudndose de 2 4. Yesos progresivos para corregir la deformidad.
muletas. El diagnstico ms probable es: 5. Plantillas con cua de base medial para el varo de retropi y
1. Osteoporosis transitoria. soporte del arco londitudinal.
2. Coxartrosis.
3. Fractura por estrs del cuello femoral.
La respuesta correcta es: 1
4. Espondilitis anquilosante con afectacin monoarticular.
5. Necrosis isqumica de la cabeza femoral.
COMENTARIO:
El caso clnico es el de un pie cavo infantil que se caracteriza
La respuesta correcta es: 5
por una elevacin de la bveda plantar. El punto ms
importante a tener en cuenta en el diagnstico de un pie cavo
COMENTARIO: infantil es que ms de un 50% de los casos estn asociados a
La necrosis avascular (no traumtica) de la cabeza femoral en alteraciones neurolgicas, siendo la ms frecuente la
el adulto es un cuadro de presentacin tpica en varones de enfermedad de Charcot-Marie-Tooth. En cualquier nio con un
edad entre los 35 y los 50 aos. La presentacin clnica pie cavo debera realizarse un electromiograma como estudio
combina el dolor intenso en el muslo (irradiado desde la regin inicial, considerando que un pie cavo tiene una etiologa
inguinal) con cojera y limitacin dolorosa de la movilidad de la neurolgica mientras no se demuestre lo contrario.
cadera (especialmente la rotacin interna). Los factores de
riesgo ms frecuentes a investigar en la historia clnica son el
Pregunta 92
consumo de alcohol, el tratamiento con corticoides,
Una de las siguientes afirmaciones referidad a las
hemoglobinopatas, pancreatitis, colagenopatas, gota e
osteomielitis y artritis spticas causadas por
hiperuricemia, enfermedades metablicas, hipercolesterolemia,
Staphylococus aureus NO es correcta:
etc. Hasta en un 20% de los casos la necrosis es idiomtica. El
1. La osteomielitispor S aureus puede ser resultado de
tratamiento es quirrgico realizndose perforaciones (forages)
diseminacin hematgena, traumatismos o infeccin
en estadios iniciales (cabeza femoral no deformada) o prtesis
estafiloccica sobreyacente.
en estadios ms avanzados (cabeza femoral deformada). El
2. En los nios, la diseminacin hematfena suele afectar las
diagnstico diferencial debe incluir la osteoporosis transitoria de
metfisis de los huesos largos, un area de crecimiento seo
cadera, casi siempre en mujeres y asociada al embarazo, que
muy vascularizada.
se resuelve sin necesidad de ciruga.
3. En los adultos la osteomielitis hematgena suele afectar a las
vrtebras, y rara vez aparece en los huesos largos.
4. La evidencia radiogrfica de osteomielitis es previa al inicio
de los sntomas clnicos.
5. El absceso de Brodie es un foco aislado de osteomielitis
Pregunta 90 estafiloccica en el rea metafisaria de los huesos largos.
Un paciente de 25 aos sufre un giro brusco en su rodilla
durante un partido de futbol, forzando la rotacin interna
de la pierna. Nota un crujido articular y presenta dolor La respuesta correcta es: 4
inmediato as como un derrame abundante a los pocos
minutos del accidente. El diagnstico ms probable ser:
COMENTARIO:
1. Rotura del ligamento lateral interno de la rodilla.
Las osteomielitis y artritis spticas causadas por Staphylococcus
2. Rotura del menisco externo.
aureus pueden tener un origen diverso. En ocasiones son el
3. Rotura del cuerno posterior del menisco interno.
resultado de una diseminacin hematgena desde un foco
4. Rotura del ligamento cruxzado anterior.
distinto al hueso, de la infeccin de una herida, o de una
5. Rotura del ligamento cruzado posterior.
fractura abierta. Respecto de la localizacin de la afectacin, es
tpica en el nio la afectacin metafisaria por ser las metfisis
La respuesta correcta es: 4 las regiones ms vascularizadas del hueso en crecimiento y la
afectacin vertebral en el adulto por la abundante

17
vascularizacin vertebral. El casos de osteomielitis metafisaria 2. Cilindros leucocitarios.
puede desarrollarse un foco aislado de osteomielitis que 3. Cilindros hemticos.
conocemos con el nombre de absceso de Brodie. Lo que es 4. Cilindros granulosos.
evidente en esta pregunta es que la evidencia radiogrfica de la 5. Lipiduria.
osteomielitis NUNCA es previa al incio de los sntomas clnicos.
Al contrario, los cambios radiolgicos aparecen como mnimo
La respuesta correcta es: 3
unas dos semanas despus del inicio de la clnica. Este hecho
de hallazgos radiolgicos un tiempo despus de la clnica
tambin lo recordamos caracterstico de las fracturas de fatiga o COMENTARIO:
de estrs, y de algunas patologas como la enfermedad de El sndrome nefrtico se define por la presencia de hematuria,
Perthes. proteinuria y reduccin del filtrado glomerular con oliguria,
insuficiencia renal y retencin de agua y sal. El sedimento
puede contener cilindros hemticos (hallazgo exclusivo en las
Pregunta 93
hematurias de origen en la nefrona).
Una espondilolistesis grave tipo IV de L5 en un enfermo
de 16 aos, Que expresin clnica puede producir, entre
las siguientes, con mayor probabilidad?: Pregunta 96
1. Parapleja. Un hipertenso de 45 aos, hasta entonces con buen
2. Deformidad ciftica lumbo-sacra. control, experimenta cefalea intensa, disminucin de la
3. Lordosis torcica. visin, malestar profundo y marcado ascenso de las cifras
4. Parlisis Cuadriceps. tensionales a 240/140. En la exploracin fsica presenta
5. Prdida fuerza del pie. edema de papila, hemorragias en llama y estertores
hmedos en las bases pulmonares. En el curso de una
semana la urea ha aumentado a 150 mg/dl y en la orina
La respuesta correcta es: 5
se detecta microhematuria. Este cuadro tiene una lesin
histolgica vascular caracterstica. Sealela:
COMENTARIO: 1. Hipertrofia de la capa media de arterias y arteriolas.
La espondilolistesis es el desplazamiento de una vrtebra 2. Necrosis fibrinoide.
respecto a la adyacente. Existen varios tipos: displsica, 3. Panarteritis exudativa.
stmica, traumtica, degenerativa, y patolgica. El caso que nos 4. Hialinosis de la media.
ocupa es una espondilolistesis stmica que afecta generalmente 5. Fibrosis de la ntima.
a varones jvenes y se debe a una espondilolisis (fractura de la
pars articular). El grado de desplazamiento se valora segn la
La respuesta correcta es: 2
escala de Meyerding: grado I (<25%), II (25-50%), III (50-
75%), y IV (>75%). La presentacin clnica depende del grado.
El grado I y II suele cursar con dolor episdico local, y el grado COMENTARIO:
III y el IV cursa con un dolor duradero y mantenido La necrosis fibrinoide de las arteriolas, con obliteracin de la luz
acompaado de deformidad del tronco, con prdida o inversin por endarteritis (infiltracin leucocitaria), es caracterstica de la
de la lordosis lumbar (respuesta 2 correcta), alteracin de la HTA maligna o una HTA acelerada cuando afecta a los nios. En
marcha y contractura de isquiotibiales. En los grados III y IV, el la HTA maligna hay edema de papila.
dolor radicular y el compromiso neurolgico son muy
infrecuentes, si no coexisten con otras patologas como una
hernia discal concomitante. Pregunta 97
Le consultan por un paciente de 60 aos con trastornos de
la conciencia de pocos das de evolucin que tiene una
Pregunta 94 hiponatrema de 120mEq/I. No vmitos ni edemas. No ha
La evaluacin neurolgica inicial de un paciente con realizado tratamiento alguno. Cul es la posibilidad
traumatismo craneoenceflico revela que solo abre los diagnstica ms frecuente?:
ojos ante un estmulo doloroso y emite sonidos 1. Insuficiencia cardiaca.
incomprensibles, pero localiza el dolor. Seale su 2. Sndrome de secrecin inadecuada de ADH.
puntuacin en la Escala de Coma de Glasgow: 3. Sndrome nefrtico.
1. 6 puntos. 4. Hipertensin portal.
2. 4 puntos. 5. Insuficiencia renal aguda.
3. 3 puntos.
4. 9 puntos.
5. 12 puntos. La respuesta correcta es: 2

La respuesta correcta es: 4 COMENTARIO:


Sin otros datos que orienten a otras causas de hiponatremia
(vmitos, diurticos, etc.) y en ausencia de edemas (que
COMENTARIO: excluyen una hiponatremia dilucional), lo razonable es pensar
La escala de Glasgow se utiliza para valorar el nivel de en un SIADH - neoplsico? idioptica? u otras causas?...
conciencia de un paciente que ha sufrido un traumatismo
craneoenceflico, si bien se ha hecho extensiva para graduar
todo trastorno de conciencia provocado por cualquier causa. Pregunta 98
Valora la respuesta verbal, ocular y motora del paciente. En Un hombre de 38 aos de edad con funcin renal previa
funcin de la puntuacin en la escala de Glasgow, los normal presenta sntomas gripales e inicia tratamiento
traumatismos craneoenceflicos se clasifican en leves (14-15 con Ibuprofeno 600mg cada 12 horas. Pasados 5 das
puntos), moderados(9-13 puntos) y graves(3-8 puntos). Por consulta por persistencia de fiebre, artralgias y aparicin
definicin, el coma es tener una puntuacin, segn esta escala, de una erupcin cutnea eritematosa pruriginosa. En la
entre 3 y 8 puntos. El paciente de la pregunta presenta un nivel analtica presenta creatinina de 3,2 mg/dl, urea
de conciencia de 9 puntos (motor: 5; ocular: 2; verbal:2) 126mg/dl, eosinofilia, proteinuria 0,5g/24h, sedimento
urinario con 8-12 leucocitos por campo y cilindros
hialinos. Cul es la etiologa ms probable de la
Pregunta 95 insuficiencia renal aguda?:
Indique el hallazgo ms indicativo de sndrome nefrtico 1. Necrosis tubular aguda.
agudo, en el anlisis de orina: 2. Necrosis cortical por antiinflamatorios no esteroideos.
1. Cilindros hialinos. 3. Nefritis intersticial aguda.

18
4. origen prerrenal por vasoconstriccin de la arteriola aferente 4. Poliuria.
glomerular por antiinflamatorios no esteroideos. 5. Eosinofilia.
5. Glomerulonefritis aguda postinfecciosa.
La respuesta correcta es: 4
La respuesta correcta es: 3
COMENTARIO:
COMENTARIO: En el rechazo agudo del trasplante renal puede aparecer fiebre,
Un cuadro de insuficiencia renal aguda asociada a clnica de dolor en el injerto, aumento del tamao por el edema y
alergia y eosinofilia y con el antecedente de la toma de un eosinofilia. No aparece poliuria, s puede aparecer oliguria.
frmaco, obliga a pensar en un cuadro de nefropata intersticial
aguda por frmacos, tambin llamada por hipersensibilidad.

Pregunta 99
102
Un paciente con insuficiencia renal crnica (filtrado
glomerular 20 ml/min/1.73 m2) con anemia en
tratamiento con dosis estables de eritropoyetina sin
suplementos de hierro, presenta en la ltima revisin En relacin a la bacteriuria asintomtica durante el
hemoglobina 10.7g/dl, hematocrito 32.4%, ferritina 56 embarazo, seale la afirmacin INCORRECTA:
ng/ml, ndice de saturacin de la tranferrina del 12%, sin 1. La pielonefritis aguda durante el embarazo es ms
evidencia se sangrado. Que actitud teraputica es frecuente en el tercer trimestre.
aconsejable?: 2. La incidencia de pielonefritis clnica aguda en las
1. Aumentar solo la dosis de eitropoyetina. mujeres embarazadas con bacteriuria aumenta
2. Administrar hierro y mantener la misma dosis de significativamente respecto a las mujeresno embarazadas.
eritropoyetina.
3. Suspender la eritropoyetina y administrar hierro. 3. La prevalencia de bacteriuria hallada en screening de
4. Aumentar la dosis de eritropoyetina y administrar hierro. mujeres embarazadas es significativamente superior a la
5. No modificar el tratamiento. hallada en mujeres no embarazadas.
4. El desarrollo de pielonefritis aguda durante el embarazo
aumenta el riesgo de prematuridad.
La respuesta correcta es: 2
5. Las mujeres embarazadas con bacteriuria tienen riesgo
alto de sufrir bacteriuria recurrente.
COMENTARIO:
La eritropoyetina necesita un buen depsito de hierro para
La respuesta correcta es: 3
poder corregir la anemia. Antes de darla al enfermo hay que
demostrar que el enfermo tiene buena ferritina.
COMENTARIO:
La PNA durante el embarazo es ms frecuente en el 3er.
100
Trimestre debido a una mayor dilatacin fisiolgica de la va
Un enfermo de 50 aos, diabtico conocido desde hace 30
urinaria durante este perodo, debido al efecto compresivo del
aos, sin retinopata proliferactiva ni alteraciones
propio feto. La incidencia de la PNA clnica durante el embarazo
urinarias conocidas, presenta edemas de instauracin
es superior en la mujer no gestante por lo descrito
rpida, proteinuria nefrtica y microhematuria. El
anteriormente en la respuesta n 1. La prevalencia de
aclaramiento de creatina es de 120 ml/min. El
bacteriuria en mujer embarazada es similar a la mujer no
complemento es normal y no se detectan anticuerpos. el
embarazada, sin embargo existe un potencial riesgo de que
cuadro se ha mantenido estable en los ltimos seis meses.
esta origine cuadros clnicos con afectacin del tracto urinario
El diagnstico sera:
superior (Pielonefritis). La PNA en embarazada se asocia con un
1. Nefropata diabtica.
aumento del riesgo de prematuridad y mortalidad perinatal. Las
2. Nefropata mesangial.
mujeres embarazadas con bacteriuria tienen alto riesgo de
3. Glomerulonefritis membrano-proliferativa.
sufrir bacteriuria recurrente por existir factores predisponentes
4. Nefropata membranosa.
a la aparicin de esta. No obstante, este hecho ocurre de igual
5. Glomerulonefritis proleferativa extracapilar.
forma en la bacteriuria que se produce en mujer no gestante.

La respuesta correcta es: 4

COMENTARIO:
103
En este paciente que ha permanecido estable sin retinopata ni
alteraciones urinarias previas, la aparicin de un cuadro de
instauracin rpida de proteinuria nefrtica y microhematuria (y
a la edad de cincuenta aos) nos tiene que hacer pensar en una Varn de 57 aos de edad, fumador, que consulta
nefropata membranosa. por hematuria terminal, polaquiuria, urgencia y dolor
miccional. Presenta citologas urinarias positivas de
carcinoma urotelial y el estudio anatomo patolgico tras la
Reseccin Transuretral es de carcinoma "in situ" difuso,
con intensa inflamacin crnica. El tratamiento estandar
101 ser:
1. Instalacin del bacilo de Calmette y Guerin.
2. Cistectoma radical.
3. Instilaciones con Mytomicina.
Cul de los siguientes sntomas o signos NO son
4. Quimioterapia con Cisplatino.
propios de la presencia de un rechazo agudo en el
5. Antiinflamatorios no esteroideos ms quinolonas
trasplante renal?:
durante seis meses.
1. Fiebre.
2. Dolor en el rea del injerto.
3. Aumento tamao injerto renal. La respuesta correcta es: 1

19
COMENTARIO:
El carcinoma in situ urotelial se caracteriza por ser una
enfermedad difusa y con alta tendencia a la recidiva. Este
106
hecho, se ve favorecido por la alta indiferenciacin celular que
presentan estos tumores. Se considera por tanto, un tumor de
alto grado de malignidad. Su diagnstico se ve dificultado al
afectar difusamente la mucosa urotelial sin crecer hacia la luz Paciente de 66 aos intervenido de Prostatectoma
de la va urinaria. El tratamiento aceptado son las instilaciones Radical hace 3 aos por Adenocarcinoma de Prstata
endovesicales con bacilo de Calmette Guerin que induce una Gleason 8 (pT2b N0M0). Presenta en el momento actual
reaccin inmunolgica sobre el urotelio afectado. El fracaso de una cifra de PSA srico de 12 ng/ml. Seale cul de las
la terapia con BCG en estos tumores con afectacin vesical siguientes afirmaciones le parece correcta:
obligaria a la ciruga radical (cistectoma). 1. La supervivencia media en el momento acutal es menor
de 1 aos.
2. la cifra de PSA est en rango normal ya que existen
otras fuentes de produccin del mismo.
3. El paciente puede tener una recidiva local o bien
104 metstasis a distancia.
4. La utilizacin de bloqueo hormonal en este caso no es
una opcin de tratamiento posible.
5. En caso de tratarse de una recidiva local estara
Los pacientes con enfermedades intestinales
indicado realizar ciruga de rescate para extirpar dicha
inflamatorias que requieren amplias resecciones
masa.
intestinales, con relativa frecuencia presentan litiasis
renal recidivante. A que trastorno metablico se suele
asociar este tipo de litiasis?: La respuesta correcta es: 3
1. Hiperuricosuria.
2. Hiperoxaluria.
COMENTARIO:
3. Cistinuria.
El PSA es una protena especfica de tejido prosttico. En clnica
4. Hiperparatiroidismo.
se utiliza para el diagnstico del adenocarcinoma prosttico, as
5. Hipercalciuria.
como para el seguimiento de los enfermos diagnosticados de
este tipo de tumores. Tras la prostatectoma radical (PR) el PSA
La respuesta correcta es: 2 deber ser indetectable, dado que hemos extirpado todo el tejido
prosttico. En el seguimiento de estos enfermos el PSA debe
permanecer indetectable y su aumento nos obliga a investigar
COMENTARIO:
si se produce por recidiva local o por metstasis a distancia.
Todos aquellos factores que induzcan una malabsorcin de
Este enfermo presenta a los 3 aos de PR un PSA claramente
cidos grasos ocasionan un aumento en la absorcin del oxalato
patolgico. La supervivencia de este enfermo en principio es
y secundariamente hiperoxaluria. La causa ms frecuente de
muy superior a 1 ao de vida. La utilizacin de bloqueo
hiperoxaluria es precisamente la malabsorcin de AG. Debido al
hormonal es una de las mejores opciones teraputicas para este
alto poder de precipitacin que presenta el oxalato, finalmente
enfermo. Si se tratase de recidiva local no estara indicado
se producen litiasis de oxalato clcico en estas pacientes. Son
ciruga de rescate y s se podra valorar la irradiacin del lecho
causas de malabsorcin de AG: dficit de enzimas pancreticas
quirrgico.
exocrinas, afectacin ileal como por ejemplo Enf. de Crohn,
resecciones ileales que produzcan un sndrome de intestino
corto....

107

105
Un hombre de 31 aos de edad consulta por la
presencia de una masa palpable en el teste derecho, de un
mes de evolucin, no dolorosa. Su urlogo le realiza una
Un hombre de 45 aos, tiene un carcinoma de
ecografa testicular, en la que se evidencia una lesin
clulas renales extendido. Los niveles de GOT, fosfatasa
hipoecoica, bien delimitada, intratesticular. Los
alcalina, LDH y alfa-2globulina son elevados y el tiempo de
marcadores tumorales alfafetoprotena y beta HCG son
protombina alargado. El hgado aparece difusamente
negativos. La actitud ms correcta de entre las siguientes
agrandado pero no existen defectos focales de infiltracin
sera:
intrahpatica. La explicacin etiolgica ms probable para
1. Dado que los marcadores tumorales son negativos, se
estos hallazgos ser:
descarta neoplasia testicular y requiere observacin.
1. Los efectos hepato-txicos de tumor.
2. Repetir la ecografa testicular en un plazo de tres
2. Metstasis heptica.
meses.
3. Amiloidosis.
3. Realizacin de una tomografa axial computorizada
4. Trombosis tumorales que obstruyen la vena heptica.
traco-abdmino-plvica.
5. Hepatitis vrica aguda.
4. Biopsia transescrotal del testculo.
5. Orquiectoma radical y esperar resultado del patlogo.
La respuesta correcta es: 1
La respuesta correcta es: 5
COMENTARIO:
El adenocarcinoma renal tambin llamado tumor del internista
COMENTARIO:
se caracteriza por la potencialidad para producir numerosos
La aparicin de una masa palpable en un testculo de un
cuadro clnicos. Entre ellos, es capaz de inducir un efecto txico
paciente adulto joven, debe de hacernos sospechar de
a nivel heptico con aparicin de alteraciones en el perfil
existencia de tumor testicular. Se hace mandatoria en estos
heptico, sin la necesidad de que existan metstasis a este
casos la realizacin de una ecografa escrotal. De confirmarse
nivel (Sind. De Stauffer).
ecogrficamente estos hallazgos y pese a ser los marcadores
tumorales testiculares negativos (un 30% de los tumores de

20
testculo no elevan marcadores), la 1 actitud teraputica debe
ser la extirpacin de ese testculo. La realizacin de biopsia Los niveles de B12 por debajo de los 300-350
transescrotal se encuentra proscrita por la posibilidad de pg/ml identifican mal los enfermos con carencia de B12.
diseminacin tumoral y por la dificultad que entraa el obtener Para paliar esta coyuntura se debe cuantificar sus
una correcta cantidad de tejido procedente de la pulpa metabolitos (homocistena y cido metil-malnico), pero
testicular. para su valoracin es importante conocer su relacin.
Cal de las siguientes relaciones es la correcta?:
1. Ambos metabolitos se encuentran elevados en el dficit
de folatos y ambos normales en el dficit de cobalaminas.
2. Ambos metabolitos se encuentran elevados en el dficit
108 de cobalaminas y ambos normales en el dficit de folatos.
3. Homocistena y metil-malnico se encuentran elevados
en el dficit de cobalaminas y homocistena elevada en el
dficit de folatos.
Paciente de 63 aos en tratamiento a demanda con 4. Homocistena elevada en el dficit de cobalaminas y
Citrato de Sildenafilo por presentar disfuncin erctil de homocistena y metil-malnico elevado en el dficit de
aos de evolucin. Seale cul de los siguientes frmacos folatos.
NO asociara en ningn caso a su tratamiento: 5. Homocistena elevada, metilmalnico normal en dficit
1. Amiodarona. de cobalaminas y metilmalnico elevado y homocistena
2. Verapamilo. normal en dficit de folatos.
3. Digoxina.
4. Indapamida.
5. Mononitrato de Isosorbide. La respuesta correcta es: 3

La respuesta correcta es: 5 COMENTARIO:


Los niveles sricos de B12 y folato pueden encontrarse dentro
de limites normales en paciente con deficiencia orgnica de
COMENTARIO: estas sustancias. La medicin de niveles plasmticos de
Para tratamiento de la disfuncin erctil existen varias opciones homocisteina y metilmalnico son ms fiables para identificar
teraputicas . Hoy en da el uso de los inhibidores de la S- carencia en el organismo de B12 y folato, pero mientras ambas
fosfodiesterasa va oral es la ms extendida. Para la sustancias estn elevadas en la deficiencia de B12, en la de
prescripcin de este tipo de frmacos debemos descartar el uso folato slo se incrementa la homocisteina.
concomitante de nitratos, pues el mecanismo de accin de
ambos es vasodilatacin. Los antecedentes personales del
enfermo deben ser tambin valorados pues haber padecido
IAM, I. Cardaca y otras patologas cardiolgicas pueden ser
causa de la no utilizacin de este tipo de terapia. Estas 111
patologas deben ser valoradas por un cardilogo pues no
siempre contraindica el uso de este grupo de frmacos.

Nio de 8 aos diagnosticado de leucemia


mieloblstica aguda, sometido a trasplante alognico de
una hermana HLA idntica tras acondicionamiento con
109 Ciclofosfamida e irradiacin corporal total. El da+26
postraplante comienza con diarreas acuosas frecuentes
acompaadas de dolor abdominal, presenta un eritema en
palmas, antebrazos y trax y en la analtica se detecta un
Un sujeto de 70 aos diagnosticado de artritis aumento de transaminasas discreto con elevacin
reumatoide consulta por clnica de sndrome anmico importante de la bilirrubina. cul es el diagnstico ms
desde hace 6 meses. Se realiza un hemograma que probable?:
muestra: Hb:8,5g/dl; VCM:69 fl; leucocitos 7,5.109/l y 1. Enfermedad injerto contra husped aguda.
plaquetas 220.109/l, sideremia 30 y/dl (N: 50-150), 2. Enfermedad Venooclusiva heptica.
frerritina: 520 ng/ml (N<400). Seale cual es el 3. Enfermedad por Citomegalovirus.
diagnstico ms probable: 4. Enfermedad por Citomegalovirus.
1. Anemia de enfermedad crnica. 5. Enfermedad injerto contra husped crnica.
2. Anemia ferropnica.
3. Anemia por dficit de Ac. Flico.
4. Anemia por dficit de B 12. La respuesta correcta es: 1
5. Anemia perniciosa.
COMENTARIO:
La respuesta correcta es: 1 La EICH aguda aparece en los primeros 100 das tras el
trasplante alognico y se caracteriza por afectacin preferente
de piel, tracto gastrointestinal con diarrea, naseas y vmitos y
COMENTARIO: lesin heptica con patrn bioqumico de citolisis y colestasis.
La anemia de enfermedad crnica puede ser normoctica o, La infeccin por CMV suele presentarse ms tardamente (1-
como en este caso clnico, microctica. En el paciente existe 3 ,meses) y se presenta fundamentalmente como neumona .
adems antecedente de una enf. Crnica como la A.R. La enf. Venooclusiva heptica se presenta con hepatomegala
Bioqumicamente se caracteriza por hiposideremia, como la dolorosa , ascitis e ictericia. La EICH crnica produce
ferropenia, pero a diferencia de esta ltima la ferritina srica se manifestaciones mas all de los tres meses postrasplante.
encuentra elevada, por la falta de utilizacin de los depsitos de
hierro .

112

110

21
2. LAM M3.
El diagnstico ms probable de un paciente que 3. LAM M5.
presenta anemia crnica desde hace varios aos, con 4. LAM M6.
episodios hemolticos agudos en infecciones, ferropenia 5. LAM M7.
crnica y episodios de trombosis venosas repetidos, en el
que se demuestra una hemoglobina de 9,3
g/dL,reticulocitos=18x109/L, Leucocitos=2,8x109/L, La respuesta correcta es: 2
neutrofilos=0,75x109/L, neutrofilos=0,75x109/Ly
haptoglobina=indetectable, es: COMENTARIO:
1. Anemia aplsica por txicos qumicos. La t (15;17) de la LAM-M3 ocasiona el gen de fusin PML/RAR
2. Anemia de Fanconi. receptor del ac. Retinoco, que es causa de la incapacidad de
3. Betatalasemia mayor. maduracin de las clulas leucmicas. Esta anomala se corrige
4. Hemoglobinuria paroxstica nocturna. con el Tto. con ATRA, propio de esta forma de leucemia,
5. Anemia de Balckfan-Diamond. asociado a quimioterapia.

La respuesta correcta es: 4

COMENTARIO: 115
La HPN se caracteriza por hemlisis intravascular, con la
consiguiente hemoglobinuria y hemosiderinuria que pueden
ocasionar ferropenia, episodios trombticos venosos y
ocasionalmente diversas citopenias en el hemograma. El resto Cul de las siguientes afirmaciones en relacin a
de respuestas no son causa de trombosis y las enfermedades la Leucemia Mieloide Crnica es correcta?:
aplsicas no son causa de proceso hemoltico. 1. El cromosoma Ph se produce a partir de una
translocacin recproca entre los cromosomas 15 y 17.
2. El cromosoma Ph aparece en precursores
granulocticos, eritroides, megacariocticos, linfoides y
fibroblastos medulares.
113 3. El cromosoma Ph es una alteracin del cariotipo til
como marcador diagnstico de la LMC pero sin relacin
con la patogenia de la enfermedad.
4. El ocogn BCR-ABL codifica una protena (p210) con
Un paciente de 35 aos acude a Urgencias con actividad tirosinguinasa aumentada.
cuadro de cefaleas y disminucin del nivel de conciencia, 5. El mesilato de imatinib es un frmaco citotxico
asociado a un cuadro de petequias y equimosis. En las alquilante utilizado en la fase crnica de la enfermedad.
pruebas analticas se objetiva una anemia con criterios de
hemlisis microangioptica con una Hb de 8 gr/dl y
Plaquetas de 30.000 mm3 junto con datos de insuficiencia La respuesta correcta es: 4
renal. El diagnstico de sospecha es una prpura
trombtica trombocitopnica. En su opinin cul de las COMENTARIO:
siguientes sera la actitud teraputica ms idnea en este El cromosoma Filadelfia presenta el gen de fusin BCR/ABL,
paciente?: material gentico productor de la P-210 que es la culpable de la
1. Realizar dilisis renal pues la evolucin del cuadro renal malignizacin celular , favoreciendo la proliferacin celular e
marca la evolucin de este proceso. impidiendo la apoptosis . El tratamiento con Mesilato de
2. Dada la cifra de Hb y de Plaquetas, junto con el cuadro Imatinib impide esta accin oncognica, por lo que es el
hemorrgico florido, el tratamiento de eleccin es la frmaco de eleccin para el tratamiento de la enfermedad.
transfusin inmediata de hemates y plaquetas.
3. Los esteroides y los antiagregantes plaquetarios son el
tratamiento de eleccin.
4. La realizacin de recambio plasmtico con
plasmafresis diaria es el tratamiento de eleccin. 116
5. Se ha de iniciar cuanto antes el tratamiento con
inmunosupresores.

En la Leucemia Aguda Linfoblstica, la


La respuesta correcta es: 4 identificacin de factores pronsticos ha permitido reducir
la toxicidad relacionada con el tratamiento a los pacientes
COMENTARIO: considerados de bajo riesgo. Son factores que mantienen
El cuadro clnico de la PTT se caracteriza por una pntada valor pronstico favorable.: Edad de 1 a 9 aos, cifra de
consistente en trombopenia, hemlisis microangioptica , leucocitos inicial<25.000, ausencia de alterciones
fiebre, insuficiencia renal y trastornos neurolgicos. El mejor citogenticas de mal pronstico, as como:
tratamiento es el recambio plasmtico con plasmafresis. 1. Sexo masculino.
2. Cuando la cifra de plaquetas es inferior a 5x 109/L.
3. La presencia de fiebre de ms de 39C al diagnstico.
4. Si el procentaje de blastos en sangre perifrica supera
el 50%.
114 5. La rapidez de la respuesta al tratamiento inicial (<10%
de blastos en la mdula realizada el da 14 despus de
comenzado el tratamiento).

En cul de las siguientes subvariedades de la


clasificacin de las Leucemias aguda mieloides de la FAB La respuesta correcta es: 5
se da de forma caracterstica una translocacin que afecta
a los genes PML y RAR alfa (receptor alfa del cido COMENTARIO:
retinoico) determinado el gen hbrido PML-RAR alfa: La rapidez en la respuesta al tratamiento es mejor factor
1. LAM M1. pronstico que los datos clnicos o biolgicos en la LAL

22
quimioterpico antitumoral.
2. Interesa comenzar precozmente el tratamiento poco
agresivo con Melfaln y prednisona.
117
3. Se debe comenzar tratamiento con poli quimioterapia
tipo VBMCP.
4. Tratamiento con poli quimioterapia y consolidar con
Un paciente de 63 aos presenta adenopatas trasplante autlogo.
laterocervicales, axilares e inguinales. La biopsia de una 5. Trataiento con pulsos de dexametasona.
adenopata axilar muestra la presencia de clulas
pequeas hendidas y clulas grandes no hendidas con un
La respuesta correcta es: 1
patrn modular, CD19 y CD20 positivas, CD5 negativas,
presentando la translocacin t(14; 18). En relacin a la
enfermedad que presenta la paciente, cul de las COMENTARIO:
siguientes afirmaciones es FALSA?: Como otras neoplasias hematolgicas en fases asintomticas
1. En el momento del diagnstico la mayor parte de los ( LLC, LNH indolentes, macroglobulinemia) es correcto, dadas
pacientes se hallan en un estudio avanzado (III o IV). las escasas posibilidades de curacin actualmente, no realizar
2. Desde el punto de vista clnico es un linfoma tratamiento en esta etapa, demorando su inicio para fase
generalmente indolente. sintomtica.
3. La curacin de los enfermos es muy improbable, sobre
todo para los de estadio avanzado.
4. La mayora de los enfermos no presentan sntomas B al
diagnstico.
5. Su transformacin histolgica a un linfoma agresivo es 120
muy infrecuente.

La respuesta correcta es: 5 Una mujer de 68 aos con antecedentes de ACVA


isqumico derecho, cardiopata isqumica (angina crnica
estable), hipertensin arterial bien controlada y diabetes
COMENTARIO:
mellitus tipo 2, consulta por un episodio de AIT (ataque
La t ( 14;18) es propia del linfoma folicular, constituido por
isqumico transitorio). En la tomografa axial
clulas pequeas de ncleo hendido ( centrocitos ) y clulas
computarizada no se observan cambios en la imagen
grandes de ncleo no hendido ( centroblastos) , que
cerebral previa y en el electrocardiograma se confirma la
histolgicamente presenta ndulos o folculos tumorales en los
existencia de una fibrilacin auricular, con respuesta
glanglios linfticos. Se trata de una neoplasia de clulas B
ventricular normal, cuya reversin a ritmo sinusal, tanto
( marcadores CD 19 y CD 20), que no presentan el CD 5 propio
elctrica como farmacolgica, haba fracasado
de la LLC-B y el LMF. Es un linfoma indolente, habitualmente
previamente. En la ecografa se observa la existencia de
extendido al diagnstico, con pocas posibilidades de curacin y
una aurcula izquierda aumentada de tamao. Los das
frecuente evolucin a linfoma agresivo.
antes del ltimo episodio segua tratamiento con aspirina
(150 mg diarios). Cul sera su consejo teraputico?:
1. Iniciara tratamiento con acenocumarol como terapia
inicial o tras el empleo de heparina de bajo peso
molecular.
118
2. Sustituira la aspirina por clopidogrel.
3. Aadira clopidogrel a la aspirina.
4. Duplicara la dosis de aspirina.
De los siguientes enunciados, seale la respuesta 5. Antes de retirar la aspirina intentara una nueva
verdades respecto a lugar en el que se encuentra la reversin farmacolgica.
alteracin patognica principal en los Sndromes
Mielodisplsicos.
La respuesta correcta es: 1
1. En el microambiente de la mdula sea.
2. En la clula germinal pluripotencial clula "stem cell".
3. En los mecanismos de depsito de hierro en la mdula COMENTARIO:
sea. La paciente con FA de la pregunta presenta varios factores de
4. En alguna de las clulas "comprometidas" hacia una riesgo emblico: cardiopata, embolia previa, HTA y DM. No hay
lnea celular concreta. duda de la indicacin de anticoagulacin oral permanente. Las
5. La alteracin principal es en los receptores de los restantes alternativas son claramente inferiores en cuanto a
factores de crecimiento celular (citoquinas). prevencin de riesgo emblico.

La respuesta correcta es: 2 121

COMENTARIO:
Los SMD son mielopatias clonales como la HPN, los SMPC y las Ante un caso de Endocarditis infecciosa, todo lo
leucemias agudas, por lo tanto enfermedades de clulas siguiente es cierto, EXCEPTO:
pluripotenciales (panmielopatas). 1. Los criterios de Duke para el diagnstico de
endocarditis se basan en hallazagos clinicos, de
laboratorio y ecocardiogrficos.
2. En ausencia de tratamiento antibitico previo, es
improbable que la endocarditis por Enterococo o
119 Stafilococo aureus se presente con cultivo negativo.
3. La ecografa transtorcica detecta vegetaciones en ms
del 90% de los pacientes con endocarditis infecciosa
clnicamente confirmada.
Ante un paciente de 60 aos, diagnosticado de 4. No se debe administrar tratamiento antibitico emprico
Mieloma Mltiple yque se mantiene asintomtico: inicial a pacientes con endocarditis subaguda
1. No est justificado comenzar con tratamiento hemodinmicamente estable sobre todo a los que

23
hubieran recibido tratamiento antibitico en las dos 1. Escarlatina y erisipela.
semanas previas. 2. Erisipela y fiebre reumtica.
5. La endocarditis por estafilococo aureus sensible a 3. Glomerulonefritis aguda y fiebre reumtica.
Meticilina, no complicada y limitada a la vlvula tricspide 4. Carditis y erisipela.
podra tratarse con tal slo 2 semanas de Oxacilina y 5. Erisipela y glomerulonefritis aguda.
Gentamicina.
La respuesta correcta es: 3
La respuesta correcta es: 3
COMENTARIO:
COMENTARIO: La glomerulonefritis y la fiebre reumtica son complicaciones de
La sensibilidad de la ETE para el diagnstico de endocarditis es la infeccin por Streptococcus del grupo A mediadas por un
mayor que la ETT. De modo que si la sospecha clnica de mecanismo inmunolgico. La erisipela y la escarlatina son
endocarditis es alta (por ejemplo por hemocultivos positivos sin directamente producidas por la bacteria.
focalidad infecciosa) un ETT normal no descarta endocartitis y
se debe hacer ETE.

125

122
Hombre de 35 aos, cazador, no viajero, que
refiere cuadro febril de tres meses de evolucin, en forma
Ante los hallazagos que cabe esperar del anlisis de agujas vespertinas y presenta una gran
del lquido cefalorraqudeo en un cuadro de meningitis hepatoesplenomegalia, pancitopenia e
bacteriana, NO se encuentra: hipergammaglobulinemia policlonal >3g/dL. El
1. Glucosa <40 mgr/dL. diagnstico que sugieren estos datos es:
2. Abundantes hemates. 1. Fiebre tifoidea.
3. Protenas >45 mgr/dL. 2. Paludismo.
4. Cultivo positivo en el 40% de los casos. 3. Brucelosis.
5. Presin de apertura >180 cm de H2O. 4. Tularemia.
5. Leishmaniasis visceral.
La respuesta correcta es: 2
La respuesta correcta es: 5
COMENTARIO:
En la meningitis aguda bacteriana no tiene por qu haber COMENTARIO:
hemates en el LCR, salvo que la puncin lumbar sea La asociacin de pancitopenia visceromegalias e
traumtica. La presencia de hemates es ms sugestiva de hipergammaglobulinemia es caracterstica de la infeccin por
meningoencefalitis herptica. Leishmania.

123

Aspergillus fumigatus puede estar implicado en


una amplia variedad de cuadros clnicos. De las siguientes
cul NO es una manifestacin clnica habitualmente
causada por este microorganismo?:
1. Neumona en granulopnicos.
2. Salpingitis. 126
3. Otitis aspergilar de colonizacin.
4. Colonizacin de cavernas tuberculosas.
5. Asma alrgico.
Acerca de la neumona por Legionella pneumophila,
indique la respuesta correcta:
La respuesta correcta es: 2 1. Afecta casi siempre a pacientes inmunocomprometidos.
2. Se adquiere por inhalacin de las gotitas de Pflgge a
partir de pacientes que tosen o estornudan.
COMENTARIO:
3. Tiene una mortalidad global que supera el 70%.
Aspergillus es un hongo que se adquiere por inhalacin. No
4. Puede originar brotes epidrmicos, pero puede ser
produce habitualmente infeccin genital. Coloniza la va area.
tambin causa de neumona en casos espordicos.
Produce neumona en neutropnicos, infeccin ORL en
5. Afecta raramente a personas sanas.
inmunodeprimidos, aspergiloma y aspergilosis broncopulmonar
alrgica.
La respuesta correcta es: 4

COMENTARIO:
La infeccin por Legionella se adquiere por inhalacin de
124
aerosoles. Aunque aparece fundamentalmente en forma de
brotes epidmicos tambin puede haber casos espordicos,
principalmente en verano y otoo.
Qu dos enfermedades no supuradas pueden
aparecer despus de una infeccin local producida por
Streptococcus pyogenes (estreptococo b-hemoltico del
grupo A)?:

24
127 antihistamnicos para el picor. A las 48 horas volvi a
urgencias con muy mal aspecto: estaba afebril, tena
confusin mental, se observaban petequias en antebrazos
y piernas, edema en pies, TA 85/70. Pulso 110 l.p.min. de
Seale la afirmacin correcta:
amplitud pequeo. En los nuevos exmenes de la sangre
1. Las gastroenteritis por Shigella suelen darse en reas
destacaban: Valor hematocrito 46%, leucocitos 3600
hiperendmicas y tienen una incidencia estacional.
p.mm3 sin cambios en la frmula y plaquetas 65.000
2. La preparacin adecuada de los alimentos no permite
p.mm3. Glucosa 106 mg.p. dl. Creatinina 1,8 mg.p.dl.
prevenir las infecciones por Yersinia enterocoltica.
Sodio 126 mEq/l.Potasio 4,2 mEq/l. La placa de trax
3. Las salmonelas causantes de fiebres tifoidea y
mostraba un pequeo derrame pleural bilateral. Cul es,
paratfica son mantenidas por portadores humanos.
entre los siguientes, el diagnstico ms probable?:
4. Los signos y sntomas clnicos de la shigelosis aparecen
1. Malaria por Plasmodium Falcipararum.
pocas horas despus de la ingestin de los bacilos, dada la
2. Dengue.
resistencia de un microorganismo al pH cido del
3. Meningoencefalitis bacteriana.
estmago.
4. Fiebre tifoidea.
5. Las cepas de Escherichia coli enteroinvasivas producen
5. Neumona por Legionella Neumophila.
una diarrea acuosa, sin sangre ni leucocitos en las heces.

La respuesta correcta es: 2


La respuesta correcta es: 3

COMENTARIO:
COMENTARIO:
Paciente procedente del Sudeste Asitico con fiebre y mialgias
El ser humano acta como reservorio de la infeccin por
(fiebre quebrantahuesos), exantema cutneo y complicacin
Salmonella typhi y paratyphi, al quedar estas bacterias
en forma de lesiones hemorrgicas petequiales y trombopenia.
acantonadas en la vescula biliar y ser liberadas a travs de las
Este cuadro clnico es muy caracterstico del Dengue.
heces.

130
128

Un paciente HIV positivo y antecedentes de


Cul de los siguientes parsitos es transmitido al
diversas infecciones oportunistas sistmicas se presenta
hombre por mosquitos?:
con un cuadro de tres semanas de evolucin de trastornos
1. Dracunculus medinensis.
visuales. La RM craneal muestra lesiones occipitales
2. Loa Loa.
hipointensas en secuencias T1 e hiperintensas en
3. Onchocerca valvulus.
sucuencias T2 que no captan gadolinio ni tienen efecto de
4. Wuchereria bancrofti.
masa. El diagnstico ms probable es:
5. Toxocara canis.
1. Toxoplasmosis cerebral.
2. Linfoma cerebral primario.
La respuesta correcta es: 4 3. Tuberculoma cerebral.
4. Encefalitis herptica.
5. Leucoencefalopata multifocal progresiva.
COMENTARIO:
Wucherena bancrofti se transmite por picadura de mosquito.
Loa loa y Onchocerca volvulus se transmiten por picadura de La respuesta correcta es: 5
mosca. Los otros dos no lo hacen a travs de vectores.
COMENTARIO:
En un paciente con infeccin VIH y focalidad neurolgica de
evolucin subaguda junto con lesiones en RM que no captan
contraste y no tienen efecto masa, sugiere fuertemente el
diagnstico de Leucoencefalopata Multifocal Progresiva.

131

129 Paciente con infeccin por VIH y ltima


determinacin conocida de linfocitos CD4 de 234 cl/mL.
Durante los ltimos 6 meses no ha realizado tratamiento,
ni seguimiento mdico. Acude a consulta por presentar
Un hombre de 32 aos acudi a urgencias tres das fiebre, tos y disnea de una semana de evolucin. En la
despus de volver de su viaje de luna de miel en Vietnan, gasometra arterial se aprecia hipoxemia, la placa de trax
por presentar fiebre elevada de cuatro das de duracin muestra un infiltrado alveolo-intersticial bilateral y en una
con mialgias graves y cefalea intensa. El mismo da de su muestra de lavado bronco-alveolar la tincin con plata-
visita a urgencias comenz a presentar un exantema metenamina es positiva. Cul de las afirmaciones
maculo-papuloso pruriginoso. El examen de la sangre siguientes es correcta?:
mostr los siguientes datos: Valor hematocrito 38%, 1. Deberamos administrar cotrimoxazol por va
Leucocitos 3700 p.mm3 con 82% neutrfilos y 12% intravenosa.
linfocitos. Plaquetas 115.000 p.mm3. Placa de trax 2. Deberamos indicar una biopsia transbronquial para
normal. El paciente fue dado de alta con el diagnstico de confirmar el diagnstico.
cuadro viral, en tratamiento con paracetamol y 3. Los corticoides estn contraindicados por el riesgo de

25
inmunodepresin. ms probable?:
4. Tiene una infeccin pulmonar por CMV. 1. Meningitis bacteriana.
5. En caso de alergia a sulfamidas, podramos tratarle con 2. Meningitis vrica.
pentamidina inhalada. 3. Meningitis tuberculosa.
4. Encefalitis herptica.
5. Metstasis al SNC.
La respuesta correcta es: 1

La respuesta correcta es: 3


COMENTARIO:
Paciente infectado por VIH sin control en los ltimos 6 meses y
entonces ya con cifras cercanas a los 200 linfocitos CD-4 COMENTARIO:
totales; presenta neumona instersticial con hipoxemia y se Meningitis de evolucin maligna, con parlisis del VI par craneal
visualizan microorganismos con la tincin de Plata. El cuadro es y esas caractersticas del LCR sugiere fuertemente la posibilidad
muy caracterstico de infeccin por Pneumocystis jiroveci (antes de meningitis tuberculosa.
carinii). El tratamiento de eleccin es cotrimoxazol. Pentamidina
inhalada slo para profilaxis.

132

Hombre de 35 aos, con antecedentes de infeccin


por VIH, que acude al servicio de urgencias por presentar
fiebre, tos y esputos manchados de sangre de varias
semanas de evolucin. En una determinacin reciente la
cifra de linfocitos CD4 era de 166 cel./mL. La placa de 134
trax muestra un infiltrado en el LID y en la tincin de
esputo se observan bacilos cido-alcohol resistentes. Al
valorar este paciente, cul de las siguientes respuestas le Los microorganismos del gnero Chlamydia pueden
parece correcta?: implicarse en todos los cuadros que se mencionan,
1. Al tratarse de un paciente inmunodeprimido, precisa EXCEPTO uno:
tratamiento con 4 drogas antituberculosas 1. Conjuntivitis neonatal.
independentemente de la incidencia de resistencias a 2. Conjuntivitis de inclusin del adulto.
isoniacida en la comunidad. 3. Infecciones urogenitales.
2. La eleccin del tratamiento antiretroviral no afecta al 4. Linfogranuloma venreo.
tratamiento antituberculoso. 5. Chancro blando.
3. La afectacin de lbulos inferiores hace ms probable la
infeccin pulmonar por Mycobacterirum Avium (MAC).
4. Se debera esperar el resultado del cultivo de La respuesta correcta es: 5
micobacterias para iniciar tratamiento.
5. Es muy probable que la infeccin pulmonar coincida con COMENTARIO:
alguna afectacin extrapulmonar. El chancro blando es producido por Haemophilus ducreyi.

La respuesta correcta es: 5

COMENTARIO: 135
El cuadro clnico sugiere neumona tuberculosa en un paciente
infectado por VIH. Por su grado de inmunosupresin es
frecuente que presente simultneamente afectacin
extrapulmonar. Un paciente de 58 aos presenta cuadro clnico de
ictericia indolora. Las exploraciones realizadas muestran
que padece un colangiocarcinoma extraheptico confinado
al conducto biliar y dos adenopatas en el hilio heptico,
pero no tiene metstasis a distancia. Qu estadio
133 presenta?:
1. I A.
2. I B.
3. II A.
Una mujer de 60 aos consulta por presentar 4. II B.
durante las ltimas 3 semaas astenia, febrcula 5. IV.
vespertina, cefalea global y, durante los ltimos das, le
han notado confusin intermitentes y somnolencia
progresiva. A la exploracin se observa una temperatura La respuesta correcta es: 4
de 38C, somnolencia, dudosa rigidez de nuca y paresia de
VI par derecho. La placa de trax muestra un discreto COMENTARIO:
engrosamiento pleural apical derecho. El hemograma es La clasificacin TNM para el colangiocarcinoma extraheptico
normal y el perfil bioqumico bsico son normales. Tras define como:
realizar puncin lumbar se observan los resultados - T1: el tumor restringido a la va biliar.
siguientes en el LCR: presin de apertura 170mmHg, - N1: metstasis en los ganglios linfticos regionales.
protenas 140 mg/dL, glucosa 42 mg/dL, 270 clulas La presencia de un tumor T1 N1 M0 como el descrito en el caso
(70% mononucleares), tincin de gram, de Zielhl-Nielsen, clnico se clasifica como estadio IIB segn la clasificacin TNM
as como investigacin de antgenos bacterianos son del comit estadounidense conjunto sobre el cncer.
negativos. Cul, entre los siguientes, es el diagnstico

26
dos lneas de quimioterapia sin respuesta.
3. Paciente con cirrosis heptica y episodios de
encefalopata intermintentes en el tiempo.
136
4. Paciente con demencia senil y neumona aguda que
produce insuficiencia respiratoria.
5. Enfermo con EPOC infeccin bronquial que condiciona
En una paciente intervenida a los 40 aos de insuficiencia respiratoria global e ingreso en cuidados
cncer de colon, su historia familiar revela que una ta por intensivos.
va materna y su abuela materna padecieron la misma
enfermedad a los 52 y 64 aos respectivamente, y un to
La respuesta correcta es: 2
tambin por va materna fue intervenido de un
adenocarcinoma gstrico a los 45 aos de edad. Cul de
las siguientes determinaciones genticas sera til realizar COMENTARIO:
para un adecuado consejo gentico a la familia?: Pregunta moderadamente difcil sobre el paciente terminal. En
1. Determinacin del gen MSH2. esta ocasin, debes elegir el que peor pronstico tenga, de
2. Determinacin de los genes BCRA 1 y 2. forma que pueda hablarse de fase terminal. El peor de ellos
3. Determinacin del gen p53. es, claramente, el de la opcin 2 (refractario a radioterapia y a
4. Sobreexpresin del encogen Ha-Ras. dos lneas de quimioterapia, sin respuesta...). El resto de las
5. Determinacin del gen p16. opciones muestra problemas donde an no se han agotado las
armas teraputicas disponibles hoy da.
La respuesta correcta es: 1

COMENTARIO:
En esta pregunta nos plantean el caso de una paciente con un 139
cncer de colon hereditario no asociado a poliposis. Dentro de
este grupo de cncer colorrectal hereditario distinguimos dos
grandes sndromes, el sndrome de Lynch tipo II, en el que
Con cul de los siguientes tests puede valorarse el
adems del colon, existe el riesgo de tumores en otras
grado de dependencia para realizar las actividades bsicas
localizaciones como en endometrio, estmago y vas urinarias.
de la vida diaria en un paciente de 85 aos cuya situacin
El gen mutante que ms frecuentemente se encuentra afectado
clnica le impide desplazarse al centro de salud por su
es el Hmsh2 y el hMHL1, implicados en la reparacin del ADN,
propio pie?:
aunque se han descrito otras mutaciones.
1. Lawton-Brody.
2. Fargestrm.
3. Pfeiffer.
4. Goldberg.
5. Barthel.
137

La respuesta correcta es: 5


Seale cul es, entre los siguientes, el tratamiento
antiemtico ms adecuado en la obstruccin intestinal COMENTARIO:
maligna completa refractaria: Pregunta sin importancia para el MIR. La escala de Barthel es la
1. Octretido. nica que cumple esta condicin. Recuerda que la de Fagestrm
2. Metoclopramida. podras haberla descartado, pues aparece en el Manual de
3. Domperidona. Preventiva y valora la dependencia tabquica.
4. Cisaprida.
5. Enema de limpieza.

La respuesta correcta es: 1


140

COMENTARIO:
El octreotide es un anlogo de la somatostatina que puede
Respecto a los tratamientos habituales o hbitos
servir como antiemtico al disminuir las secreciones digestivas
de un paciente programado para una intervencin
a varios niveles. No puede emplearse domperidona, cisaprida ni
quirrgica bajo anestesia general, una de las siguientes
metoclopramida por ser procinticos (observa que se trata de
respuestas siguientes es correcta:
una obstruccin intestinal refractaria), ni tampoco un enema de
1. Los bloqueantes beta adrenrgicos deben mantenerse
limpieza. Si se emplease un procintico, aumentara el
durante todo el perodo preoperatorio.
peristaltismo de forma muy marcada al haber distalmente una
2. Los antiparkinsonianos deben suspenderse 48 horas
obstruccin...
antes.
3. La heparina de bajo peso molecular a dosis profilcticas
se asocia a hemorragia intraoperatoria significativa.
4. La ticlopidina no se asocia a riesgo de hemorragia
intraoperativa significativa.
138
5. La evitacin de fumar cigarrillos durante las 24 horas
previas carece de beneficio intraoperatorio.

Cul de las siguientes situaciones corresponde a


La respuesta correcta es: 1
la fase terminal de la enfermedad?:
1. Cncer de mama con matstasis seas, ganglionares,
hepticas y pulmonares que slo ha recibido COMENTARIO:
hormonoterapia. Pregunta sobre estudios preoperatorios a la que no debes
2. Cncer de pulmn intervenido hace un ao con recada conceder excesiva importancia. La opcin correcta es la 1
local y metstasis hepticas tratadas con radioterapia y porque no existe indicacin, en general, de retirar esta

27
medicacin antes de una intervencin quirrgica. El resto de las informan sobre la presencia de depsitos de IgG a nivel de la
opciones dicen lo contrario a la verdad, y podran ser intuidas o membrana basal. En definitiva, pregunta fcil que se explica en
deducidas a partir de conocimientos de otras asignaturas. clase y en el Manual con claridad.

141 143

En relacin con la valoracin preoperatoria de un Paciente de 45 aos que desde hace 1 mes
paciente para ciruga programada, la consulta presenta debilidad a nivel de cintura escapular y pelviana.
preanestsica tiene por finalidad: En la piel se objetiva edema palpebral y coloracin
1. La modificain de la tcnica anestsica prevista. eritematoviolcea periorbitaria y lesiones
2. La variacin del procedimiento quirrgico previsto. eritematodescamativas sobre prominencias sea de dorso
3. El rellenar un cuestionario sobre los antecedentes de manos. El diagnstico sera:
anestsico-quirrgicos. 1. Lupus eritematoso sistmico.
4. La prescripcin de un frmaco ansioltico para reducir la 2. Artritis reumatoide.
ansiedad generada por la prxima ciruga. 3. Dermatomiositis.
5. El establecer la presencia de enfermedades 4. Eritema polimorfo.
diagnosticadas o no, que pueden determinar el riesgo de 5. Esclerodermia.
complicaciones del procedimiento quirrgico previsto.
La respuesta correcta es: 3
La respuesta correcta es: 5
COMENTARIO:
COMENTARIO: La presencia de debilidad muscular proximal aunque no nos
En la actualidad es habitual que los pacientes que van a ser mencionen elevacin de enzimas musculares es propia de las
sometidos a una intervencin quirrgica programada pasen miopatias inflamatorias. El eritema en heliotropo y las papulas
primero por una consulta de Preanestesia, donde se establece y de Gottron que describen permiten confirmar el diagnstico de
se estudia la patologa previa del paciente que puede dermatopolimiositis. Las otras opciones no son en absoluto
condicionar complicaciones del procedimiento quirrgico. Otras dudosas.
finalidades son la informacin al paciente sobre los riesgos del
procedimiento anestsico y programar la medicacin previa a la
intervencin en caso de que la requiera.

144

142
Acude a consulta una mujer de 19 aos que
presenta, lesiones eritemetoescamosas, edema y alguna
vesicula en la cara, escote, dorso de las manos y
Hombre de 76 aos portador de virus C y diabtico antebrazos. Las lesiones tienen 12 horas de evolucin y
desde hace 10 aos, que presenta lesiones ampollosas de han aparecido tras una escursin al campo. Entre los
contenido serohemorrgico en extremidades superiores e antecedentes personales destaca acn vulgar en
inferiores, y erosiones a nivel de la mucosa bucal. La tratamiento con retinoides tpicos y doxiclicina oral. El
biopsia cutnea muestra una ampolla subepidrmica, se diagnstico ms probalbes es:
realiza inmunofluorescencia directa que muestra la 1. Erupcin lumnica poliforma.
presencia de depsitos de IgG a nivel de la membrana 2. reaccin fototxica.
basal. El diagnstico ms probable es: 3. Urticaria solar.
1. Eritema multiforme. 4. Eritrodermia por frmacos.
2. Dermatitis herpetiforme. 5. Fotodermatosis por hipersensibilidad.
3. Penfigoide ampolloso.
4. Toxicodermia ampollosa.
La respuesta correcta es: 2
5. Porfiria cutnea tarda.

COMENTARIO:
La respuesta correcta es: 3
Paciente que presenta lesiones en reas fotoexpuestas, y por lo
tanto es obvio pensar en que la exposicin solar tiene mucho
COMENTARIO: que ver con el cuadro. Para incidir ms en esta pista, nos dicen
Caso clnico en el que desde el comienzo te hablan de ampollas, que las lesiones aparecen tras una excursin al campo. El
tanto en extremidades como en mucosa bucal. Debes orientar paciente est en tratamiento para su acn con retinoides
por tanto el diagnstico dentro del grupo de las enfermedades tpicos y doxiclina oral. Unos de los efectos secundarios
ampollosas. El hecho de que te digan que se da en un paciente caractersticos de las tetraciclinas es la fotosensibilidad. En el
con hepatitis C no es ms que un dato para despistar, y que captulo de farmacologa de habla de l en el apartado de
asocies las ampollas a la porfiria cutnea tarda. A continuacin reacciones adversas de las tetraciclinas. En cuanto a los
nos suministran datos definitivos para el diagnstico, y que se retinoides tpicos, acentan la reaccin fototxica que producen
exponen con claridad en la tabla 14 que se encuentra en el las tetraciclinas. De hecho, se suele evitar pautar retinoides en
tema 15 del Manual de Dermatologa. El primer dato es la pacientes que se exponen al sol de manera habitual para evitar
afectacin de la mucosa bucal, que limita el posible diagnstico este efecto.
diferencial a cuatro entidades (eritema multiforme, pnfigo
vulgar, penfigoide ampolloso y gingivoestomatitis herptica). El
nivel de la ampolla (subepidrmica) y la edad del paciente (76
aos) prcticamente nos confirma el diagnstico (penfigoide
ampolloso). Para darnos todava ms facilidades tambin nos 145

28
147
Un paciente de 23 aos acude a consulta con un
brote intenso de dermatitis atpica que afecta a la mayor
parte dela superficie corporal y es muy pruriginoso. El
tratamiento que dee evitar es: Uno de los siguientes signos o sntomas NO
1. Antihistamnicos tpicos. esperara encontrar en una uvetis anterior aguda o
2. Tacrlimus Tpico. iridociclitis:
3. Ciclosporina oral. 1. Hiperemia ciliar o pericorneal.
4. Corticoides orales. 2. Depositos celulares sobre el endotelio corneal.
5. Corticoides Tpicos. 3. Depositos sobre la membrana Bowman.
4. Clulas en humor acuoso.
5. Sinequias posteriores.
La respuesta correcta es: 1

La respuesta correcta es: 3


COMENTARIO:
La dermatitis atpica es la enfermedad infantil ms frecuente
en la infancia, y como tal el tratamiento queda expuesto con COMENTARIO:
claridad en el Manual de Dermatologa. Todas las opciones que La uveitis anterior aguda, entra dentro del diagnstico
te indican en el examen se enuncian en el Manual. Tacrolimus diferencial del ojo rojo doloroso, junto con el glaucoma agudo y
tpico; indicado en los eccemas resistentes a los corticoides la queratitis. Recuerda que tanto en la uveitis como en la
tpicos. Ciclosporina oral; para el eccema atpico severo en queratitis, la pupila est en miosis, contrariamente a lo que
adultos (como el caso clnico de la pregunta). Tanto los sucede en el glaucoma agudo, en el cual la pupila est en
corticoides orales como tpicos son tratamientos clsicos del midriasis. En el ojo rojo doloroso, la inyeccin suele tener un
eccema atpico, oralmente estn indicados en brotes intensos, componente ciliar o periquertico (es ms marcada en la
como el caso clnico de la pregunta. Los antihistamnicos orales proximidad de la crnea), al contrario de lo que sucede en la
estn indicados para aliviar el prurito, pero nunca tpicos. En el conjuntivitis, en la cual es ms marcada en los fondos de saco.
tema 1 del captulo de Dermatologa (pgina 7) te explicamos Lo que define la uveitis es la presencia de clulas flotando en
que los antihistamnicos se utilizan siempre por va sistmica y cmara anterior (fenmeno de Tyndall). Despus de cierto
debe evitarse su empleo por va tpica, ya que producen tiempo de evolucin pueden aparecer sinequias posteriores (el
fotosensibilidad. iris inflamado se adhiere a la superficie anterior del cristalino) y
depsitos de clulas inflamatorias sobre la superficie posterior
de la crnea (depsitos endoteliales), pero en ningn caso
sobre la superficie anterior como se afirma en la opcin nmero
3.
146

Paciente de 75 aos que consulta por una mcula 148


pigmentada heterocroma, de contorno irregular, de unos 2
x 3 cm de dimetro, localizada en mejilla derecha, que ha
experimentando un crecimiento muy lento en los ltimos
aos. Cul sera el diagnstico ms probable?: En cul de las siguientes afecciones no se produce
1. Melanoma lentiginoso acral. edema de papila?:
2. Lentigo malilgno. 1. Meningioma de la vaina del nervio ptico.
3. Carcinoma Basocelular pigmentado. 2. Neuritia ptica poe esclerosis mltiple.
4. Nevus de Ota. 3. Intoxicacin por alcohol metlico.
5. Eritema fijo pigmentario. 4. Hipertensin intracraneal.
5. Atrofia ptica hereditaria dominante.

La respuesta correcta es: 2


La respuesta correcta es: 5

COMENTARIO:
Ante una mcula pigmentada de contorno irregular que va COMENTARIO:
creciendo progresivamente evidentemente debemos pensar en La presencia de edema de papila es un signo
una lesin tumoral maligna. De las opciones que te plantean neurooftalmolgico que puede aparecer en variadas situaciones
rpidamente debes descartar la opcin 1, ya que nos dicen que y que obliga a llevar a cabo un amplio diagnstico diferencial:
el paciente la presenta en la mejilla. La opcin 4 tambin tiene
poco sentido, ya que se trata de una lesin benigna que se 1. Hipertensin intracraneal: en este caso el aumento de
presenta en sujetos jvenes en zonas inervadas por la 1 y 2 presin dificulta el trasporte axoplasmtico y conduce a la
rama del trigmino, y afecta casi siempre al ojo. Menos sentido aparicin de un edema de papila, que por definicin es bilateral
tiene la opcin 5, ya que se trata de una lesin que no presenta y recibe el nombre de papiledema. 2. Tumores de la vaina del
las caractersticas clnicas que te explican en el enunciado y nervio ptico, por un mecanismo similar al de la hipertensin
adems es una toxicodermia, y en el enunciado no te hablan en intracraneal, pero en este caso el edema de papila ser
ningn momento del antecedente de la toma de un frmaco. La unilateral. 3. La HTA en su fase maligna. 4. Las neuritis
descripcin clnica del tumor coincide de forma prcticamente independientemente de su etiologa (inflamatoria, txica o
literal con la que te hacemos en el Manual (Tema 24. Melanoma isqumica), siempre y cuando sean anteriores.
Maligno) para el lntigo maligno-melanoma. Es un tumor que
aparece en personas de edad avanzada (75 aos), que se En esta pregunta la opcin nmero 2 puede resultar un poco
caracteriza por una mancha pardonegruzca discrmica, de confusa, pues la neuritis desmielinizante que aparece en el
bordes irregulares, que crece de forma lenta, en cuestin de contexto de la esclerosis mltiple suele ser retroblulbar y por
aos. Por lo explicado en el Manual y por lo visto en clase no eso lo habitual es que no exista edema de papila. Sin embargo
debe haber ninguna duda en que el diagnstico correcto es el un porcentaje pequeo son anteriores y por ello la opcin falsa
lntigo maligno. es la nmero 5.

29
149
En cuanto a los tumores de glndulas salivares,
cite lo correcto:
1. El 80% de los mismos se dan en la partida, el 80% de
Entre las manifestaciones del glaucoma crnico los cuales son malignos.
simple ( glaucoma primario de ngulo abierto), usted NO 2. El adenoma pleomrfico es la neoplsia ms frecuente
espera encontrar: de las glndulas salivales mayores, siendo de crecimiento
1. Alteraciones de la papila o cabeza del nervio ptico. lento y generalmente asintomtico.
2. Alteraciones del campo visual. 3. El tumor de Warthin es casi exclusivo de la submaxilar,
3. Cifras elevadas de presin intraocular. nunca bilateral y ms frecuente en mujeres.
4. Ausencia de fluctuacin o variacin diuran de la presim 4. El carcinoma epidermoide es la neoplasia maligna ms
intraocular. frecuente.
5. Ausencia de sintomatologa en los estadios iniciales. 5. La parlisis del nervio facial es signo de benignidad.

La respuesta correcta es: 4 La respuesta correcta es: 2

COMENTARIO: COMENTARIO:
El glaucoma crnico es la neuropata ptica ms frecuente, y tal Globalmente los tumores de glndulas salivares:
como expresa la opcin nmero 1, el factor de riesgo ms
importante es el aumento de presin intraocular. Este aumento - El 70% son benignos y 30% malignos.
de PIO daa de forma progresiva el nervio ptico y produce un - El 75-80% en partida, 10% submaxilares, 10% en glndulas
aumento de la excavacin papilar, y la correspondiente prdida salivares menores.
de campo visual (opciones 1 y2 correctas). Como la parte - A mayor tamao de la glndula, mayor porcentaje de tumores
central del campo visual se afecta en etapas tardas, el paciente benignos (partidas 80% benignos)
no es consciente de su enfermedad y por ello est enfermedad
debe ser sometida a programas de screening, y lo expresado en De los tumores benignos de glndulas salivares en partida el
la opcin nmero 5 es correcto. La opcin incorrecta es la ms frecuente es el Adenoma pleomorfo (tumoracin indolora,
nmero 4, en el paciente glaucomatoso, como en el no de crecimiento lento) seguido del tumor de Whartin o
glaucomatoso, existe un ritmo circadiano en la presin Cistoadenolinfoma (puede ser bilateral en el 15% , indoloro, de
intraocular, y estas variaciones pueden dificular la deteccin de crecimiento lento, consistencia qustica en cola de partida ) De
la hipertensin intraocular. los tumores malignos el ms frecuente es el carcinoma
mucoepidermoide y el carcinoma epidermoide a diferencia de
otros neoplsicos de cabeza y cuello es infrecuente. Los
tumores malignos de partidas tienen crecimiento rpido,
invasin de la piel o planos profundos, dolor, trismus y parlisis
150 facial, a diferencia de los tumores benignos.

A una mujer de 30 aos se le diagnostica una


neuritis ptica retrobulbar. Qu enfermedad sistmica
debe sospecharse por su mayor frecuencia de asociacin a 152
esta patologa oftalmolgica?:
1. Esclerosis mltiple.
2. Artritis reumatoide. Mujer de raza blanca, de 38 aos, que manifiesta
3. Diabetes mellitus. episodios de otorrea desde la infancia: ha presentado seis
4. Espondilitis anquilopoytica. episodios de otorrea en el oido derecho, en los ltimos 20
5. Aneurisma cerebral. meses, sin otalgia, con hipoacusia de odo derecho desde
la infancia. A la exploracin por micro-otoscopia, se
La respuesta correcta es: 1 aprecia una perforacin margial pstero-superior y atical
amplia con restos de supuracin y osteitis del reborde
seo. Se confirma una Hipoacusia transmisiva del oido
COMENTARIO: derecho. Rinne negativo en odo derecho , Weber
La neuritis retrobulbar aparece habitualmente en mujeres, lateraliza a la derecha. Que diagnstico considera ms
durante la edad media de la vida. Su mecanismo es acertado?
desmielinizante, y por ello existe una asociacin muy fuerte con 1. Otitis externa crnica.
la esclerosis mltiple. El resto de las enfermedades que nos 2. Otitis media crnica colesteatomatosa.
presentan, pueden tener manifestaciones oftalmolgicas, pero 3. Otitis media tubrica secretora crnica.
no se asocian a la aparicin de neuritis. Recuerda que la artritis 4. Otosclerosis.
reumatoide se asocia a la aparicin de ojo seco y escleritis. La 5. Timpanosclerosis.
espondilitis es la causa ms frecuente de uveitis anterior aguda
no idioptica, y la diabetes en un porcentaje de los pacientes da
lugar a la aparicin de retinopata diabtica. Los aneurismas La respuesta correcta es: 2
cerebrales pueden producir patologa oftalmolgica por
comprometer los nervios oculomotores (recuerda que el COMENTARIO:
aneurisma de la comunicante posterior es la causa ms Las otitis medias crnicas a diferencia de las otitis media y
frecuente de parlisis compresiva del III par craneal). externa aguda, se manifiestan por otorrea crnica persistente
con hipoacusia de transmisin progresiva por daos tmpano-
osiculares que producen, sin otalgia. El colestecitoma es una
forma de otitis media crnica con capacidad de producir osteitis
y produce perforaciones timpnicas marginales sobre todo a
151 nivel atical en la parte flcida. La otitis media secretora o serosa
y la otosclerosis, no producen otorrea ni perforacin timpnica.
La timpanosclerosis es una complicacin de las otitis medias
crnicas simples con formacin de placas de calcio alrededor de
la cadena osicular y membrana timpnica.

30
153 155

Hacemos referencia a los tumores malignos de Uno de los siguientes trastornos NO implica
laringe. Cul de las siguientes afirmaciones es ERRONEA? obligatoriamente la derivacin desde atencin primaria al
servicio de Salud Mental para tratamiento especializado:
1. El cncer larngeo ms frecuente es el carcinoma 1. Varn de 25 aos con trastorno de pnico y agorafobia
epidermoide. de 6 aos de evolucin.
2. Los tumores subglticos son los de localizacin ms 2. Mujer de 30 aos con cuadro psictico secundario a
frecuente y sintomatologa ms precoz. consumo de cocana.
3. En los tumores supraglticos el primer sntoma puede 3. Varn de 59 aos con ideas delirantes de celos de 10
ser una adenopata en la regin laterocervical del cuello. aos de evolucin.
4. En los tmores glticos el primer sntoma suele ser la 4. Varn de 27 aos que presenta intento de suicidio tras
disfona persistente y progresiva. ruptura sentimental.
5. En los tumores supraglticos la disnea puede 5. Mujer de 48 aos con trastorno adaptativo tipo
presentarse antes de la disfona. depresivo, secundario a conflictividad laboral.

La respuesta correcta es: 2 La respuesta correcta es: 5

COMENTARIO: COMENTARIO:
El tipo histolgico ms frecuente de los tumores malignos de No figura literal en el manual. Se trata de aplicar el sentido
laringe es el carcinoma epidermoide, siendo excepcionales los comn y ordenar los diagnsticos del ms grave (psicosis,
de la estirpe histolgica. La localizacin ms frecuente en los suicidios) al ms leve (adaptativo).
pases anglosajones es la glotis y en los pases mediterrneos,
la supergltica. Los tumores subglticos son muy raros (<5%
de los de laringe). La clnica de presentacin de los
supraglticos es poco especfica, inicialmente pueden dar
sntomas muy parecidos a una faringitis crnica y al ser muy 156
linfgeno debutar como adenopata laterocervical. Inicialmente
el crecimiento es en el compartimento supragltico y puede
manifestarse con obstruccin larngea sin afectar a la glotis, por
lo tanto, con disfona por lesin de cuerdas. Los tumores Cul de las siguientas afirmaciones sera
subglticos en estadios iniciales son asintomticos, INCORRECTA en el seguimiento de un paciente tratado con
manifestndose en estadios avanzados por clnica de disnea y sales de litio?:
disfona. 1. Precisa para su control de determinaciones de litemia.
2. Si el paciente desarrollase hipertensin arterial la
primera medida a tomar sera la supresin gradual del litio
y la introduccin de otro frmaco (p. ej. Lamotrigina)
3. Es preciso controlar la funcin tiroidea y renal.
4. El litio puede producir efectos antiagresivos
154
independientes de su accin desestabilizante.
5. Ninguna evidencia respalda una teora de deficiencia de
litio en el trastorno bipolar.
Un paciente de 25 aos ha sufrido una cada de la
bicicleta golpendose en el mentn, limitacin dolorosa de
La respuesta correcta es: 2
la apertura bucal con desviacin hacia el lado derecho y
maloclusin dentaria consistente en falta de contacto a
nivel de los molares del lado izquierdo. Qu fractura COMENTARIO:
facial sospechara y qu prueba solicitara para Otra vez el litio! Las opciones 1,3 y 5 son bsicas ( y ciertas)
diagnosticarla?: con las dos restantes es fcil razonar.
1. Cndilo mandibular izquierdo/Tomografa axial
computerizada (TAC) .
2. Fractura malar/ Tomografa emisin positrones (TEP).
3. Cndilo mandibular derecho/ Ortopantomografa.
4. Cndilo mandibular derecho/ resonancia magntica 157
nuclear (rmn)
5. Fractura etmoides / Gammagrafa sea.

Los trastornos psiquitricos que con mas


La respuesta correcta es: 3 frecuencia se diagnostican en Atencin Primaria son:
1. Trastornos depresivos.
COMENTARIO: 2. Trastornos de ansiedad.
El traumatismo sobre la regin anterior de la mandbula, el 3. Alcoholismo.
mentn, puede provocar una fractura en el cndilo mandibular. 4. Psicosis.
La fractura del cndilo nandibular derecho provoca un 5. Demencias.
desplazamiento del arco mandibular hacia ese lado por el efecto
de los pterigoideos insertados en la rama mandibular, lo que se La respuesta correcta es: 1
pone de manifiesto por una maloclusin dentaria sobre todo en
el lado izquierdo donde se pierde el contacto entre los molares
superiores e inferiores.. El diagnstico radiolgico se realiza con COMENTARIO:
una ortopantomografa, aunque la TC tambin es til. Otra pregunta de ATENCION PRIMARIA que no viene literal en
el manual, aunque si se comenta algo.En clase s se explica.

31
grave las complicaciones que se nos dan. No viene literal en
ningn libro.
158

Mujer de 58 aos, vive sola, con antecedentes de 161


HTA y artritis reumatoide, con buen control farmacolgico.
Presenta alucinaciones auditivas y cenestsicas, con ideas
delirantes de perjuicio con los vecinos y de contenido
En las pacientes con anorexia nerviosa que llevan
mstico-religioso de 4 meses de evolucin. Cul sera el
ms de un ao con amenorrea es necesario practicar:
diagnstico ms probable?:
1. Una densitometra sea.
1. Demencia frontal.
2. Una determinacin de niveles sricos de cortisol.
2. Sndrome confusional agudo.
3. Una determinacin de los niveles de prolactina.
3. Depresin delirante.
4. Un electroencefalograma.
4. Esquizofrenia de inicio tardo.
5. Un test de supresin con dexametasona.
5. Psicosis psicgena.

La respuesta correcta es: 1


La respuesta correcta es: 4

COMENTARIO:
COMENTARIO:
Un tema de moda (ALIMENTACION) y una complicacin bien
El primer Caso Clnico. Fcil. Alucinaciones auditivas y delirios
conocida (osteopenia secundaria al hipoestronismo debido a la
de perjuicio sugieren esquizofrenia sea cual sea la edad sobre
malnutricin). En clase se insiste en ello.
todo cuando no nos comentan otros sntomas.

162
159

Una adolescente de 16 aos acude a la consulta


Cul de las siguientes afirmaciones con respecto
contando que lleva cerca de un ao sintindose ms
al suicidio es INCORRECTA?:
cansada, con poco apetito y dificultades para concentrarse
1. El riesgo de suicidio es mayor en los familiares de
en los estudios. Cuando se le pregunta comenta tambin
pacientes depresivos que en familiares de pacientes con
que sale menos con las amigas y se muestra pesimista
mana o esquizofrenia.
respecto de su futuro. El diagnstico ms probable sera:
2. Las tasas de suicidio son ms altas en las zonas
1. Anorexia.
urbanas que en las rurales.
2. Distimia.
3. Un perodo de menor riesgo al suicidio en pacientes
3. Depresin mayor.
esquizofrnicos son los meses siguientes al alta
4. Trastorno de ansiedad.
hospitalaria.
5. Agorafobia.
4. En los ancianos, los intentos de suicidio son menos
frecuentes, pero el xito es mayor.
5. Los varones deprimidos son ms proclives al suicidio La respuesta correcta es: 2
que las mujeres.
COMENTARIO:
La respuesta correcta es: 3 Segundo Caso Clnico. Clnica depresiva de cerca de 1 ao de
evolucin en adolescente. Segn la DSM para el diagnstico de
DISTIMIA en adolescentes es necesario al menos 1 ao de
COMENTARIO:
evolucin; segn la CIE sera un EPISODIO DEPRESIVO. Desde
Un clsico: el suicidio. Las opciones 2,4 y 5 son evidentes (y
este punto de vista temporal, la pregunta sera impugnable, no
ciertas). La correcta viene literal en el MANUAL.
obstante, el cuadro clnico es ms compatible con distimia que
con episodio depresivo.

160
163

Todas las siguientes, EXCEPTO una, son


indicaciones de ingreso psiquitrico en pacientes con
Sealar cul de los siguientes trastornos NO parece
trastorno de Personalidad lmite. Seale la INCORRECTA:
relacionado con el espectro de la patologa obsesivo-
1. Peligor inmediato para otros.
compulsiva:
2. Impulsividad suicida o intentos suicidas serios.
1. El trastorno dismrfico corporal.
3. Episodio psictico transitorio con impulsividad.
2. La amnesia psicgena.
4. Crisis en el tratamiento con repercusiones en la vida y
3. La hipocondra.
el entorno del paciente.
4. Los tics.
5. Episodio depresivo.
5. La cleptomana.

La respuesta correcta es: 5


La respuesta correcta es: 2

COMENTARIO:
De nuevo hay que razonar y ordenar de ms grave a menos

32
COMENTARIO: desencadena el parto prematuro.
No viene literal en el MANUAL pero por referencias y parecidos 4. todas las embarazadas deben someterse a una prueba
clnicos se puede identificar el diagnstico que menos pega. El no treponmica en el primer control del embarazo, incluso
espectro obsesivo lo forman distintas enfermedades en las zonas con escasa prevalencia de sfilis prenatal.
psiquitricas afines al TOC: hipocondra ,dismorfofobia, tics, 5. El riesgo de infeccin fetal a partir de una madre con
control de impulsos. sfilis precoz no tratada es inferior al 25%:

La respuesta correcta es: 4

164 COMENTARIO:
Pregunta difcil y poco esperada, ya que es la primera vez que
se pregunta sobre la sfilis congnita. Aunque la respuesta
correcta est recogida en el manual puedes dudar entre las
Un paciente psictico, en tratamiento con
opciones 2 y 3. Los fetos infectados antes de la 18 semanas, no
medicacin neurolptica, refiere sensacin subjetiva de
desarrollan signos debido a su inmunoincompetencia. La
inquietud. A la exploracin, se ve que es incapaz de
reaccin de Jarisch-Herxheimer aparece en el 40% de los
relajarse, va y viene por la consulta alterna entre sentarse
pacientes tratados, y el 30% desarrollan contracciones
y levantarse, y cuando est de pie, se balancea de pierna a
regulares que generalmente ceden a las 24 horas de
pierna. El cuadro descrito sera compatible con el
tratamiento. En pacientes alrgicos a penicilina se deben
siguiente trastorno inducido por frmacos:
realizar pruebas cutneas para confirmarla y proceden a una
1. Parkinsonismo.
desensibilizacin debido a que no existe alternativa a la
2. Sndrome neurolptico maligno.
Penicilina durante el embarazo. El riesgo de infeccin fetal en
3. Sndrome de piernas inquietas.
una sfilis precoz no tratada es del 50-80%.
4. Distona aguda.
5. Acatisia.

La respuesta correcta es: 5


167
COMENTARIO:
Tercer Caso Clnico. Este muy fcil. Inquietud secundaria al
tratamiento con antipsicticos es la definicin de ACATISIA. Una mujer de 25 aos consulta por un retraso
Bsico, bsico. menstrual de 3 semanas y metrorragia escasa desde hace
24 horas. Su estado general es bueno, no tiene dolor, y la
exploracin ginecolgica no muestra alteraciones excepto
el sangrado escaso procedente de la cavidad uterina. el
test de embarazo es positivo y la determinacin de beta
165 hCG en plasma es del 600 mUI/ml. Por ecografa
transvaginal se observa un tero normal con un
endometrio homogneo de aspecto secretor de 12 mm de
espesor. En el ovario derecho hay una formacin que
Los siguientes hallazgos ecogrficos se consideran parece un cuerpo lteo normal No hay lquido libre en la
actualmente marcadores de cromosomopata EXCEPTO cavidad abdominal. Cul de las siguientes es la indicacin
uno: ms correcta?:
1. Ausencia del tabique nasal. 1. Repetir seriadamente cada 2-3 das la ecografa y la
2. Ectasia pilica. beta-hCG.
3. Flujo invertido en el ductus venoso. 2. Reposo absoluto y repetir la ecografa a las 2-3
4. Acortamiento de la longitud del fmur. semanas.
5. El retardo en la fusin del amnios y el corion. 3. Legrado uterino.
4. Tratamiento con Metotrexato por va sistmica.
La respuesta correcta es: 5 5. Laparoscopia.

COMENTARIO: La respuesta correcta es: 1


Pregunta comentada en clase y cuyas respuestas estn
incluidas en el Manual. De las respuestas que nos dan, la 2 y la COMENTARIO:
4 son marcadores del segundo trimestre y la 1 y la 3 del primer Caso clnico incluido en el manejo diagnstico de las
trimestre. El retardo en la fusin del amnios y el corion no se metrorragias del primer trimestre de gestacin. En este caso se
relaciona con ninguna cromosomopata ni retraso del debe hacer el diagnstico diferencial entre una gestacin
crecimiento y slo representa, en algunos casos, una pequea incipiente, una gestacin ectpica o un aborto completo. En
dificultad en la realizacin de la amniocentesis. esta pregunta la determinacin seriada cada 2-3 das de los
niveles sanguneos de BHCG nos permitir diagnosticar una
gestacin ectpica si no se duplica. Niveles en sangre de BHCG
superiores a 1.000 mUI/ml. Permiten ver el saco gestacional
intrauterino, por tanto, la ecografa tambin sera
166 imprescindible.

En relacin con la sfilis en el embarazo slo una de


las siguientes afirmaciones es correcta:
1. En embarazadas alrgicas a penicilina el tratamiento de 168
eleccin son las tetraciclinas.
2. Las lesiones de sfilis congnita, se desarrollan en
general antes del cuarto mes de gestacin. Paciente de 34 semanas de gestacin, que acude a
3. La reaccin de Jarisch-Herxheimer en el embarazo urgencias por presentar contracciones uterinas con la

33
sospecha de amenaza de parto pretrmino. Se realiza 2. Placenta previa.
registro cardiotocogrfico fetal, comprobndose la 3. Desprendimiento de placenta.
existencia de 2 contracciones en 20 minutos y una 4. Crioamnionitis hemorrgica.
longitud cervical medida por ecografa de 3 cm. Cul 5. Rotura de vasos previos.
sera la actitud correcta?:
1. Ingresar a la paciente y dejar evolucionar, pues el feto
La respuesta correcta es: 3
ya est maduro bajo el pjnto de vista pulmonar.
2. Dejar que se desplace nuevamente a su domicilio
advirtindole que realice reposo y que acuda de nuevo a COMENTARIO:
urgencias si la dinmica se hace ms frecuente. Pregunta sencilla y la respuesta correcta aparece en el manual.
3. Ingresar a la paciente y administrarle tocolticos para Se debe hacer el diagnstico diferencial entre las metrorragias
frenar la dinmica. del tercer trimestre y el punto clave es el aumento de tono
4. Ingreso en la sala de expectantes repitiendo, si sigue la uterino y claramente doloroso a la palpacin caracterstico del
dinmica, la ecografa vaginar para ver si existe desprendimiento prematuro de la placenta normalmente inserta
modificacin del cervix. o abruptio placentae. En la rotura uterina adems de la
5. Ingreso y administracin de tocolticos y corticoides metrorragia, cursa con dolor abdominal intenso, pero no se
para la maduracin pulmonar. detecta aumento del tono uterino y se palpan las partes fetales.

La respuesta correcta es: 2

COMENTARIO: 171
Pregunta de dificultad intermedia debido a que todos los datos
clnicos expuestos se encuentran en el lmite por decidir la
decisin de tratar o no a la paciente. El tratamiento tocoltico y
maduracin pulmonar con corticoides est indicado entre las 24 La infeccin por estreptococo del Grupo B en el
y 34 semanas. Entre las 34 y 37 semanas no se recomienda recin nacido puede prevenirse administrando
debido al escaso beneficio fetal obtenido. Por tanto, aunque antibiticoterapia profilctica a las embarazadas
realmente este paciente presenta una amenaza de parto portadoras o a las que presentan factores de riesgo.
prematuro, debido a su edad gestacional y a las escasas Respecto a este problema, seale la afirmacin
modificaciones cervicales, la conducta ms adecuada sera dar INCORRECTA:
de alta a la paciente para realizar reposo domiciliario y 1. La profilaxis se realiza durante el parto.
advertirle que vuelva si presenta mayor dinmica ya que si se 2. En las gestante salrgicas a penicilina puede utilizarse
inicia la fase activa del parto se debera ingresar a la paciente y eritromicina.
dejar evolucionar espontneamente el parto. 3. La va ms frecuente de infeccin fetal es
transplacentaria.
4. El cribado en las embarazadas se realiza entre las
semanas 35-37 de gestacin.
5. La incidencia de infeccin neonatal por estreptococo B
proporcionalmente es mayor entre recin nacidos
169
prematuros que entre recin nacidos a trmino.

La respuesta correcta es: 3


Cul de las siguientes es la causa ms frecuente
de coagulacin intravascular diseminada durante el
embarazo?: COMENTARIO:
1. Diabetes gestacional. Pregunta fcil, preguntada en mltiples ocasiones y reflejada en
2. Placenta previa. el Manual. La va de infeccin del recin nacido es por
3. Hipertensin arterial crnica asociada al embarazo. transmisin vertical en el momento del parto y no va
4. Desprendimiento precoz de placenta. transplacentaria.
5. Aborto incompleto.

La respuesta correcta es: 4


172
COMENTARIO:
Pregunta fcil que ha aparecido varias veces en exmenes
previos. La respuesta aparece en el manual. El desprendimiento
prematuro de placenta normalmente inserta es la causa ms Una mujer de 38 aos consulta por una secrecin
frecuente de desarrollo de una coagulacin intravascular vaginal anormal con mal olor, como a "pescado podrido",
diseminada y constituye la causa ms frecuente de trastornos que se hace ms intenso despus del coito. No tiene
de la coagulacin en el embarazo. Otras causas menos prurito genital. En la exploracin se observa una
frecuentes son el feto muerto intratero y el aborto complicado. abundante secrecin blanco-griscea, que no se adhiere a
las pareces vaginales. Al mezclar una muestra de la
secrecin con una gota de hidrxido potsico al 10% se
aprecia claramente el mal olor referido. Cul de los
siguientes es el tratamiento de eleccin?:
1. Amoxicilina con cido Clavulnico por va oral.
170
2. Clotrimazol por va intravaginal.
3. Doxiciclina por va oral.
4. Clindamicina por va intravaginal.
Gestante de 38 semanas que ingresa con trabajo 5. Fluconazol por va oral.
de parto. Durante el perodo de dilatacin presenta cuadro
de dolor brusco. A la exploracin usted objetiva
La respuesta correcta es: 4
metrorragia escasa y aumento del tono uterino a la
palpacin abdominal que resulta muy doloroso. Cul sera
su diagnstico?: 1- Rotura uterina.

34
COMENTARIO: COMENTARIO:
Pregunta fcil en la que debes hacer el diagnstico diferencial Pregunta de dificultad intermedia aunque puede responderse
entre las tres causas ms frecuentes de vulvovagintits, la correctamente con la informacin recogida en el manual. De
Gardnerella vaginalis, la Cndida albicans y la Trichomona manera lgica una oclusin tubrica disminuir las posibilidades
vaginalis. Los datos clnicos son la leucorrea blanco-grisceos, de extensin de la infeccin haca tronco y pelvis. El mtodo
el olor caracterstico a pescado en descomposicin y la prueba ms especfico para el diagnstico continua siendo la
de las aminas. El tratamiento de eleccin es la va local con laparoscopia aunque en la mayora de las ocasiones no sea
clindamicina o metronidazol. necesario realizarla ya que se llega fcilmente al diagnstico
con los criterios clnicos. El tratamiento antibitico de amplio
espectro puede hacerse de manera ambulatoria si la EIP es leve
o moderada. Ante dudas en el diagnstico es preferible
instaurar tratamiento debido a la baja incidencia de efectos
173 adversos de la antibioterapia y la alta tasa de secuelas de EIP
no tratadas. Aunque est indicada la retirada del DIU en
pacientes portadores de ese mtodo anticonceptivo, lo deseable
es realizarla despus de la instauracin del tratamiento
Cul sera el primer cambio hormonal de la pre- antibitico, para evitar que la manipulacin del DIU favorezca la
menopausia?. Seale la respuesta correcta: infeccin ascendente.
1. FSH y LH disminudas.
2. FSH y LH aumentadas.
3. FSH normal y LH aumentadas.
4. FSH aumentada y LH normal.
5. FSH aumentada y LH disminuda.
175

La respuesta correcta es: 4


Una mujer de 65 aos consulta por metrorragias
COMENTARIO: escasas desde hace tres meses. La exploracin
Pregunta sencilla contestada claramente en el Manual. Los ginecolgica es normal. En una ecografa transvaginal se
cambios endocrinolgicos observados en la poca de la observa un tero de 7 x 3 x 4 cm. con un endometrio de 14
premenopausia se caracterizan por la bajada de la inhibina-g al mm. de espesor, y unos ovarios atrficos. La citologa
no existir retroaccin negativa se produce aumento de la crvicovaginas informa de un frotis atrfico sin otras
gonadotropina FSH, que es la modificacin endocrina ms alteraciones celulares. En una biopsia endometrial
precoz del climaterio. La LH est normal o aumentada. realizada por aspiracin con una cnula flexible de tipo
Cornier o Pipelle (microlegrado) se informa de escaso
material endometrial de tipo atrfico, insufiente para un
diagnstico endometrial adecuado. Cul de las siguientes
indicaciones es la ms adecuada?:
174 1. Tratamiento hemosttico con estrgenos +
progestgenos, seguido de progestgenos cclicos cada
mes durante 6 meses.
2. Histerectoma.
En relacin con la Enfermedad Inflamatoria 3. Repeticin de la biopsia se vuelve a sangrar, y en caso
Plvica, seale la opcin INCORRECTA: contrario repeticin de la ecografa a los 4-6 meses.
1. La esterilizacin tubrica (ligadura tubrica) disminuye 4. Histeroscopia.
el riesgo de Enfermedad Inflamatoria Plvica. 5. Completar el estudio con determinacin de marcador
2. La laparoscopia es el mtodo ms especfico para tumoral CA 12,5 y otras pruebas de imagen com RNM o
establecer el diagnstico de salpingitis aguda. TAC.
3. El tratamiento puede hacerse de forma ambulatoria
mediante una combinacin antibitica que posea amplio
espectro de actividad. La respuesta correcta es: 4
4. Si la paciente es portadora de DIU es necesario retirarlo
previamente al inicio de cualquier tratamiento antibitico. COMENTARIO:
5. Las pacientes con un diagnstico dudoso de Pregunta fcil. Se debe sospechar un cncer de endometrio
Envermedad Inflamatoria Plvica deben ser tratadas, ya ante toda metrorragia en una paciente peri o postmenopusica,
que es preferible el tratamiento precoz al objeto de evitar como es el caso clnico de esta pregunta. Es imprescindible por
las secuelas. el diagnstico de cncer de endometrio la biopsia endometrial
dirigida mediante histeroscopia, por tanto la respuesta correcta
La respuesta correcta es: 4 es la n 4. No se debe esperar a que vuelva a sangrar para
repetir la biopsia, sobre todo, teniendo en cuenta que por
ecografa el grosor endometrial es mayor de 5mm.

176

Las pacientes con jsndorme de ovarios


poliqusticos presentan un aumento de riesgo a largo
plazo para los siguientes procesos EXCEPTO:
1. Diabetes mellitus tipo 2.
2. Hipertensin arterial.
3. Osteopenia.
4. Dislipemia.
5. Coronariopata.

35
La respuesta correcta es: 3 179

COMENTARIO:
Pregunta de baja dificultad. Se ha demostrado las pacientes con En relacin con el cncer de endometrio cul de
SOP hay una insulino resistencia que produce un aumento en las siguientes respuestas es la verdadera?:
los niveles de insulina y por tanto se estimula la actividad 1. El cncer de endometrio es ms frecuente en mujeres
cromatasa en las clulas de la granulosa que convierten los premenopusicas.
andrgenos en estrgenos. Por tanto estas pacientes no 2. La contracepcin oral durante ms de tres aos es un
presentan osteopenia y s un riesgo elevado de paceder factor de riesgo.
diabetes mellitus tipo 2 acompaado de HTA, dislipemia y por 3. El tratamiento con tamoxifeno es un factor de riesgo.
tanto un riesgo mayor de coronariopata en la edad adulta. 4. La multiparidad es un factor de riesgo.
5. Es menos frecuente que el cncer de ovario.

La respuesta correcta es: 3


177
COMENTARIO:
Pregunta muy fcil y con mencin directa en el Manual. El
tamoxifeno pertenece a los moduladores selectivos, de los
Que afirmacin es INCORRECTA respecto a las
receptores de estrgenos y tiene accin estrognica sobre el
variables pronsticas del cncer de mama:
endometrio.
1. El tamao del tumor y el estado de los ganglios
linfticos axilares son factores de valor pronstico
probado.
2. La recidiva del cncer de mama es ms probable en los
tumores que poseen receptores hormonales de estrgenos
180
y de progesterona.
3. La sobreexpresin del gen c-erbB-2 (HER-2/neu) puede
ser indicativa de resistencia/sensibilidad a determinados
tratamientos. Una nia de 6 aos, diagnosticada de coartacin de
4. Eklevadas concentraciones de PCNA (Ki67) indican peor aorta consulta por talla baja. En la exploracin fsica se
pronstico. observa talla en Percentil 3 para su edad y pterigium coli.
5. La presencia de un nmero elevado de microvasos en el Cul de los siguientes es el diagnstico ms probable y
tumor se acompaa de un peror pronstico. que exploracin realizara para confirmarlo?:
1. Dficit de GH y determinacin de IGF-1 srica.
2. Sndrome de turner y cariotipo.
La respuesta correcta es: 2
3. Hipocondroplasia y radiografas de esqueleto seo
completo.
COMENTARIO: 4. Hipotiroidismo y determinacin de TSH y T4 sricas.
Dentro de las pregunta que se han hecho sobre los factores 5. Disgenesia gonadal pura y cariotipo.
pronsticos del cncer de mama es una de las ms sencillas. El
ms importante es el nmero de ganglios afectados por tanto la
La respuesta correcta es: 2
n 1 es falsa. La presencia de receptores estrognicos predice
buena respuestas a la terapia hormonal, por lo que su ausencia
es signo de mal pronstico. Actualmente se dispone de un COMENTARIO:
tratamiento especfico. Caso clnico muy sencillo. Ante una nia que asocie la existencia
de talla baja, coartacin de aorta y pterigium coli, se debe
sospechar siempre la existencia de un sndrome de Turner. El
diagnstico de este sndrome frecuente, se realiza con el
cariotipo que demostrar la existencia en la mitad de los casos
178 de la anomala 44X. El resto de las opciones pueden cursar con
talla baja pero no asocian las anomalas cardacas y el
pterigium coli.
La menopausia tarda aumenta el riesgo de
desarrollar un cncer de mama:
1. Si hay antecedentes familiares de cncer de mama.
2. En mujeres sometidas a Terapia Hormonal Sustitutiva
181
(THS).
3. En casos de menarquia precoz.
4. En todas las mujeres.
5. En multparas. Todas las siguientes son causa de retraso en la
eliminacin del meconio EXCEPTO:
1. Fibrosis qustica.
La respuesta correcta es: 4
2. Drogadiccin materna.
3. S. del colon izquierdo pequeo.
COMENTARIO: 4. Aganglionosis rectal.
Pregunta muy fcil y reflejada en el Manual. Existen mltiples 5. Hipotiroidismo congnito.
factores de riesgo como el factor gentico (BRCA-1, BRCA-2),
sobre todo en etapas anteriores a la menopausia. La
La respuesta correcta es: 5
menopausia tarda aumenta el riesgo en todas las mujeres por
el efecto que ejercen los estrgenos sobre el tejido mamario a
lo largo del tiempo, independientemente de otros factores que COMENTARIO:
se puedan aadir. Esta es una pregunta directa, bastante asequible. Recuerda que
se define el retraso en la evacuacin de meconio como la no
eliminacin del mismo en las primeras 48 horas de vida. Sus

36
causas principales son seis: colon izquierdo hipoplsico
(respuesta 3), fibrosis qustica (respuesta 1), aganglionosis
rectal (respuesta 4), drogadiccin materna (respuesta 2),
184
prematuridad y tratamiento materno con sulfato de magnesio.
La respuesta falsa es la nmero 5. El hipotiroidismo congnito a
estreimiento, pero no a trastornos meconiales.
Nia de 9 aos con padres obesos que consulta
desde los 6 aos ha aumentado excesivamente de peso
aunque el aumento de talla ha sido importante. Refieren
un buen apetito y una ingesta consecuente de alimento y
182 una tendencia a la inactividad. A la exploracin fsica los
datos ms relevantes con un ndice de Masa Corporal de
158% con telarquia 2/4 bilateral y un vello pubiano 3/6
con inicio de vello axilar. Enumero los posible diagnsticos
Nia de 7 aos que presenta pubarquia grado II-
y que pruebas complementarias solicitara:
III sin telarquia asociada. Cul de las siguientes
1. Diagnstico: Pubertad precoz idioptica. Solicitara una
afirmaciones es FALSA?:
determinacin de FSH y LH para distinguir si es central o
1. La aparicin precoz de pubarquia es mucho ms
perifrica.
frecuente en nias que en nios.
2. Diagnstico: Pubertad precoz secundaria a la obesidad.
2. En esta paciente est incrementando el riesgo de
Solicitara FSH LH y 17 beta estradiol y si los valores son
Hiperandrogenismo ovrico e Hipoerinsulinismo en la
elevados iniciara tratamiento con andrgenos.
adolescencia, por lo que es recomendable realizar
3. Diagnstico: Obesidad y adelanto puberal secundaria.
seguimiento.
Solicitara FSG, LH y 17 beta estradiol y maduracin
3. El que se asocien axilarquia y aumento del olor corporal
esqueltica para comprobarlo. Iniciara en el momento
no implica un cambio de actitud.
restriccin calrica y valoracin a los 6 meses.
4. Est indicado realizar una radiografa de mueca y una
4. Diagnstico: Adelanto puberal y obesidad como
determinacin basal de DHEA y 17-OH-progesterona.
problemas independientes. Tras un estudio de la funcin
5. Esta situacinn conduce en la mayora de las ocasiones
tiroidea y de los niveles de FSH y LH, iniciara tratamiento
a un crecimiento acelerado y desarrollo puberal completo,
con progestgenos para retrasar la pubertad.
disminuyendo la talla final.
5. Diagnstico: Obesidad, adelanto puberal, ambas por un
posible hipotiroidismo. Solicitara FT4 y TSH y aunque los
La respuesta correcta es: 5 valores fueran normales si la maduracin esqueltica est
atrasada iniciara tratamiento sustitutivo.
COMENTARIO:
Pregunta bastante complicada. La paciente presenta un La respuesta correcta es: 3
desarrollo prematuro de una caracterstica puberal de forma
aislada. A este cuadro se le denomina pubarquia o adrenarquia
COMENTARIO:
precoz, y se incluye dentro de la pubertad precoz
Pregunta relacionada con las consecuencias clnicas de la
verdaderamente incompleta. Es tpico que estos pacientes no
obesidad en la infancia. En este caso nos presentan a una nia
requieran ningn tratamiento y que tengan el desarrollo puberal
con obesidad severa, confirmndose con la exploracin fsica un
en el momento esperado (respuesta 5 falsa). Es mucho ms
adelanto puberal (presenta telarquia y pubarquia asociados).
frecuente en nios y parece asociarse a un riesgo de
Este hecho est relacionado con el aumento de las hormonas
hiperandrogenismo en la adolescencia. La realizacin de
sexuales por el tejido adiposo y el tratamiento de eleccin sera
radiografa de mueca y determinacin hormonal es til en el
la prdida de peso. Sera apropiado realizar un estudio
diagnstico diferencial.
hormonal completo y valorar la edad sea, que en estos nios
suele estar aumentada.

183
185

En el diagnstico de laboratorio de intolerancia-


malabsorcin de Lactosa todas las pruebas son tiles
Nio de 2 aos que presenta fiebre elevada de 4
EXCEPTO:
das de evolucin, con afectacin del estado general, y
1. Determinacin de cuerpos reductores en heces.
ligera irritabilidad. A la exploracin destaca la presencia
2. Anlisis de H2 en aire espirado.
de un exantema maculopapuloso discreto en tronco e
3. Test de ureasa en la biopsia. intestinal.
hiperemia conjuntival bilateral sin secrecin. Presenta
4. Determinacin de pH fecal.
adems enrojecimiento bucal con lengua aframbuesada e
5. Determinacin de cido lctico en heces.
hiperemia farngea sin exudados amigdalares, adems de
adenopatas laterocervicales rodaderas de unos 1,5 cm de
La respuesta correcta es: 3 tamao. Ha recibido 3 dosis de Azitromicina. Cul, entre
los siguientes, es el diagnstico ms probable?:
1. Exantema sbito.
COMENTARIO: 2. Sndrome de Kawasaki.
Esta es una pregunta directa, en la que se pregunta sobre las 3. Escarlatina.
medidas diagnsticas de la malabsorcin de lactosa. En este 4. Rubeola.
entidad, los cuerpos reductores en heces dan positivo, al ser la 5. Mononucleosis infecciosa.
lactosa un azcar reductor (R: 1). Cuando la lactosa se
malabsorbe, ste es fermentada por la flora saprofita del colon.
La fermentacin da lugar a sustancias cidas (R 4 y 5) y gas (R La respuesta correcta es: 2
2). El diagnstico de certeza se establece a travs de la biopsia
intestinal con cuantificacin de la actividad de lactasa, no de
COMENTARIO:
ureasa. Por eso, la opcin 3 es falsa.
Esta pregunta expone, en forma de caso clnico, la historia de
un paciente con enfermedad de Kawasaki. Esta entidad se

37
diagnostica a travs del cumplimiento de sus criterios (R4). La gammagrafa con MIBG sigue emplendose como
diagnsticos. Adems de la fiebre, aqu aparecen 4: exantema, tcnica diagnstica (R5). La opcin correcta es la nmero 3. En
conjuntivitis bilateral no purulenta, cambios en la mucosa oral todo neuroblastoma hay que verificar si hay afectacin de M.O.
(eritema labial, lengua aframbuesada) y adenopatas.

188
186

Un nio de 7 meses deshidratado, con vmitos,


Lactante de 2 meses de edad que hace tres das diarrea y fiebre nos llega la siguiente analtica: Hb 11.6 gr
comenz con mocos nasales acuosos, tos y estornudos. %; Hto: 39%; Leucocitos: 14.900 mm3 (Neutrfilos: 63%;
Desde ayer presenta taquipnea con 60 rpm, tiraje Linfocitos: 30%; Monocitos: 7%); Plaquetas: 322.000
intercostal con aleteo nasal. En urgencias observan distrs mm3; Osmolaridad: 295 mOs/L; Na: 137 mEq/L; K: 5.6
respiratorio con sibilancias inspiratorias y espiratorias y mEq/L; Ca inico: 1,2 mmol/L; pH: 7,20; pCO2: 25 mmHg;
algunos crepitantes bilaterales. Una Rx de trax muestra CO3H: 11 mEq/L; E.B.:- 19 mEq/L; Lactato: 5.3 mmol/L,
hiperinsuflaccin bilateral con una atelectasia laminar y Creatinina: 4.2 mgr%. Cul es la valoracin metablica
corazn pequeo. Presenta: pH 7,24; pCO2: 58 mmg; de esta deshidratacin?:
CO3H: 21 mEq/L. Cul es el diagnstico ms probable?: 1. Deshidratacin isotnica con acidosis mixta.
1. Bronconeumona bilateral. 2. Deshidratacin hipotnica con acidosis meta blica.
2. Crisis asmtica de origen infeccioso. 3. Deshidratacin isotnica con acidosis metablica.
3. Miocarditis con insuficiencia cardaca congestiva. 4. Deshidratacin hipertnica con acidosis metablica.
4. Neumonitis intersticial. 5. Deshidratacin isotnica con acidosis respiratoria.
5. Bronquiolitis.
La respuesta correcta es: 3
La respuesta correcta es: 5
COMENTARIO:
COMENTARIO: Esta pregunta es muy asequible. Simplemente has de valorar
Una nueva pregunta en forma de caso clnico, perteneciente una analtica general en un paciente peditrico. Este nio tiene
esta vez al bloque de respiratorio. Recuerda que la bronquiolitis acidosis (pH<7.35) metablica (bicarbonato menor de 20); por
es el primer episodio de dificultad respiratoria sibilante y de otro lado, como tiene un Na comprendido entre 135 y 145, su
causa infecciosa en un nio de menos de dos aos. Aqu tienes deshidratacin es isotnica o normonatrmica.
todos los elementos para acertar su diagnstico: 2 meses de
edad, antecedente de catarro de vas altas, evolucin trpida,
aparicin de problemas respiratorios (taquipnea, distrs) y
exploracin compatible (sibilancias difusas). Las pruebas
complementarias engalanan el caso, pero no aportan nada 189
(hiperinsuflacin en la Rx trax, acidosis respiratorias).

Nio de 3 aos de edad que tras un golpe en el


colegio presenta una lesin hemorrgica que cicatriza mal.
El paciente haba acudido antes en diversas ocasiones a
187
Dermatologa por eccema en cara y brazos, y a Pediatra
por infecciones respiratorias. El hemograma indica
leucocitos y eritrocitos normales, pero plaquetas escasas y
Lactante varn de 10 meses de edad al que en un pequeas. Cul es el diagnstico ms probable?:
examen rutinario de salud se le descubre una masa en 1. Edema angioneurtico hereditario.
flanco izquierdo, dura, que sobrepasa lnea media. En la 2. Sndrome de Wiskott-Aldrich.
ecografa abdominal dicha masa se corresponde con un 3. Hemofilia A.
tumor slido localizado en la glndula suprarrenal 4. Prpura trombopnica inmune.
izquierda. Cul de las siguientes afirmaciones es cierta?: 5. Dermatitis atpica.
1. La edad inferiro al ao empeora el pronstico.
2. Si presentara metstasis hepticas estara
La respuesta correcta es: 2
contraindicado el tratamiento quirrgico.
3. Debe realizrsele un aspirado de mdula sea como
parte del estudio de extensin. COMENTARIO:
4. El tumor que presenta es ms frecuente en pacientes En un nio pequeo la presencia de eczemas, trombopenia e
con hemihipertrofia. infecciones de repeticin es muy sugestivo de Sndrome de
5. El empleo de la gammagrafa con Wiskott-Aldrich.
metayodobencilguanidina ha sido abandonado por
tcnicas ms especficas.

La respuesta correcta es: 3


190

COMENTARIO:
Esta pregunta ilustra un caso clnico correspondiente a un
neuroblastoma de la suprarrenal izquierda. La opcin 1 es falsa, Todo lo que sigue acerca del sndrome nefrtico en
pues la edad inferior a un ao es uno de los factores el nio es cierto, EXCEPTO:
pronsticos favorables. La presencia de metstasis en hgado, 1. Colesterol srico elevado.
piel o M.O. no contraindica la reseccin del tumor primario, 2. El 85% se debe a la forma histolgica de enfermedad de
pues responden bien a quimioterapia coadyuvante (R2). El cambios mnimos.
tumor de Wilms se asocia hemihipertrofia, no el neuroblastoma 3. Reabsorcin reducida de sodio por el rin.

38
4. Triglicridos sricos elevados. 193
5. La hipoalbuminemia es la causa de la hipoproteinemia.

La respuesta correcta es: 3 En un ensayo clnico se comparan 3 tratamientos


(p.e. placebo, tratamiento establecido y un tratamiento
nuevo). La variable respuesta es contnua (p.e. nivel de
COMENTARIO:
glucosa en sangre). Si la variable no tiene una
En el sndrome nefrtico la aparicin de edemas deplecciona el
distribucin normal, el test correcto para comparar la
volumen de lquido intravascular lo que producira la elevacin
respuesta es?:
de renina y aldosterona. Ello conlleva el aumento de
1. La t de Student.
reabsorcin de sodio y agua en el tbulo proximal.
2. El test de Wilcoxon.
3. Anlisis de la varianza.
4. El test de Krusal-Wallis.
5. El test ji-cuadrado.
191
La respuesta correcta es: 4

En relacin con la fstula traqueoesofgica COMENTARIO:


congnita, refiera cul de las complicaciones siguientes al Las pruebas no paramtricas son las adecuadas cuando las
tratamiento quirrgico es la ms frecuente: variables no siguen una distribucin normal. En este ensayo
1. Fstula anastomtica. clnico la variable 1 es cualitativa (3 tratamientos) y la variable
2. Estenosis esofgica. 2 cuantitativa (glucemia), as que el test que se debe utilizar
3. Fstula traqueoesofgica recidivante. es: Kruskal-Wallis.
4. Reflujo gastroesofgico.
5. Traqueomalacia.

La respuesta correcta es: 4


194

COMENTARIO:
La atresia de esfago es una entidad quirrgica que cuyo
Cul de las siguientes afirmaciones es correcta en
tratamiento acarrea muy diversas complicaciones, entre las
relacin a la estadstica?:
cuales la ms frecuente es el reflujo gastroesofgico (R 4) por
1. La estadstica tiene entre sus funciones la correccin de
mala motilidad del esfago. Con menos frecuencia aparecen
los errores y sesgos de una mal diseo.
traqueomalacia (R5), estenosis (R2) o refistulizacin (R 1 y 3).
2. Ante un estudio con sesgos nos pdemos fiar de los
resultados si el valor de significacin encontrado es
razonablemente pequeo, por ejemplo p<0,0001.
3. La significacin estadstica es un criterio objetivo por lo
que puede suplir la objetividad del juicio clnico.
192 4. La mejor manera de investigar consiste en la obtencin
del mayor nmero p posibles, guindonos por los
resultado obtenidos.
Una nia de 2 aos se ha encontrado bien hasta 5. El valor de la significacin nos orienta sobre cul es la
hace aproximadamente 12 horas, momento en que probabilidad de que la diferencia observada sea debida
empez con un cuadro de letargia, vmitos y episodios de exclusivamene al azar.
llanto intermitente con un estado de dolor visible.
Coincidiendo con los episodios, retrae las piernas hacia el La respuesta correcta es: 5
abdomen. En la consulta emite una deposin de color rojo
oscuro; el abdomen muestra plenitud y un discreto dolor a
la palpacin. El diagnstico ms probable es: COMENTARIO:
1. Estenosis hipertrfica de ploro. El objeto de la Estadstica es determinar cul es la probabilidad
2. Apendicitis. de que los resultados obtenidos en un estudio se puedan
3. Infeccin urinaria. explicar por el azar. Esta probabilidad se corresponde con el
4. Invaginacin grado de significacin.
5. Enfermedad ulcerosa pptica.

La respuesta correcta es: 4


195
COMENTARIO:
Esta es una pregunta clsica en Pediatra, que ilustra una
historia compatible con invaginacin intestinal. Por qu? Muy
Cul de las siguientes es una caracterstica tanto
fcil. La edad del paciente est comprendida entre 3 meses y 6
de los estudios epidemiolgicos de cohorte como de los
aos de edad, que es la edad en la que suele aparecer la
ensayos clnicos controlados?:
invaginacin. Tiene episodios de llanto paroxstico con
1. Pueden ser retrospectivos.
encogimiento de piernas, que alternan con otros de letargia,
2. En el anlisis de sus resultados se compara la variable
que son los sntomas cardinales de esta entidad. Adems ha
resultado entre los sujetos con y sin el factor de
emitido una deposicin en jalea de grosella, que suele
exposicin.
acompaar a las invaginaciones intestinales. Simplemente haba
3. El investigador decide qu pacientes sern expuestos al
que formular el diagnstico de presuncin.
factor en estudio.
4. Forman parte de los llamados estudios de tipo
transversal.
5. La existencia de un grupo control permite mejorar la
validez externa del estudio.

39
La respuesta correcta es: 2
Respecto al anlisis de los resultados de un estudio
epidemiolgico de cohorte mediante el riesgo relativo es
COMENTARIO: cierto que:
Tanto en los estudios de cohortes como en los ensayos clnicos 1. Se calcula dividiendo la incidencia de enfermedad en lo
hay un grupo de expuestos (a un factor de riesgo y a un no expuestos entre la incidencia de la enfermedad en los
tratamiento respectivamente) y un grupo de no expuestos. expuestos.
2. Carece de unidades.
3. Es una medida del efecto absoluto del factor de riesgo
que produce la enfermedad.
4. En los estudios de cohorte tiene el mismo valor que el
196 "odds ratio".
5. Su lmite inferior es 1.

Tenemos inters en estudiar la relacin que pueda La respuesta correcta es: 2


existir entre la utilizacin de determinado medicamento
antiasmtico y las muertes por asma. Para ello,
disponemos de los datos de los ltimos 10 aos de la COMENTARIO:
mortalidad por asma en determinada rea de Salud, y del El riesgo relativo es el cociente entre la incidencia en expuestos
consumo de este medicamento en el mismo mbito. Con y la incidencia en no expuestos y carece de unidad. Oscila entre
estos datos, qu tipo de diseo de estudio podra cero y + infinito.
realizarse?:
1. Estudio de cohorte retrospectivo.
2. Estudio de caso-control anidado.
3. Estudio cuasi-experimental.
4. Estudio de tendencias temporales. 199
5. Ensayo de campo.

La respuesta correcta es: 4 Respecto al ensayo clnico controlado, indique cul


de las siguientes afirmaciones es cierta:
1. Es un mtodo experimental.
COMENTARIO: 2. Su caracterstica fundamental es el enmascaramiento
En los estudios ecolgicos de tendencias temporales la unidad del estudio.
de anlisis son grupos de individuos, no individuos. En este 3. Es un mtodo observacional.
estudio que nos describen precisamente slo disponemos de 4. Debe incluir como mnimo 100 pacientes.
datos poblacionales, no individuales acerca de la mortalidad por 5. Debe durar como mnimo una semana.
asma y del consumo de cierto medicamento.

La respuesta correcta es: 1

COMENTARIO:
197 Se trata de una pregunta directa acerca de las caractersticas
de diseo de un ensayo clnico. Los estudios epidemiolgicos se
pueden dividir como bien sabis en dos grupos: descriptivos y
Se cree que un neurolptico produce analticos. Estos ltimos, se subdividen en observacionales
hiperprolactinemia. Para averiguarlo, se disea un estudio (cuyos dos representantes ms importantes son el estudio de
en el que se recogen pacientes diagnosticados de casos y controles y el estudio de cohortes) y experimentales. El
hiperprolactinemia y se aparean con pacientes de su diseo experimental por excelencia es el ENSAYO CLNICO
misma edad y sexo, diagnosticados de EPOC, enfermedad (opcin 1 correcta y opcin 3 incorrecta). Aprovechemos para
cardiovascular y fracturas traumticas, recogiendo de su recordar las tres caractersticas propias de un diseo
historia clnica si han consumido o no el frmaco. Estamos experimental: - introduccin del factor de estudio por parte del
ante un estudio con un diseo, de qu tipo?: investigador (el investigador es activo). - asignacin aleatoria
1. Cohortes con grupo control. de los participantes a los distintos grupos de estudio. El
2. Casos y controles. enmascaramiento es una tcnica para controlar los sesgos en la
3. Cohorte retrospectivo. medicin de la variable resultado. Si utilizamos este
4. Ensayo clnico. procedimiento decimos que el ensayo clnico es ciego (simple,
5. Casos y controles anidados. doble o triple). Sin embargo, no todos los ensayos clnicos son
ciegos. Segn nuestro enunciado no podemos asumir que el
nuestro lo sea (opcin 2 incorrecta). Tan solo sabemos que
La respuesta correcta es: 2 contaba con un grupo control (ensayo clnico controlado,
tampoco todos los ensayos clnicos lo son). Con respecto a las
COMENTARIO: opciones 4 y 5 que tambin son incorrectas hemos de decir que
Comenzamos este estudio con un grupo de personas que no existe un mnimo de duracin o de tamao muestral. Estos
padecen hiperprolactinemia (enfermedad de estudio) y otras aspectos estn condicionados por el objetivo del estudio y la
que no. Buscamos el antecedente de consumo de un variable utilizada. Por eso, se hace la predeterminacin del
neurolptico (factor de estudio). Se trata de un estudio de tamao muestra.
casos y controles.

200
198

Un estudio en el que se seleccionan sujetos libre de


enfermedad que son clasificados segn el nivel de

40
exposicin a posibles factores de riesgo y son seguidos COMENTARIO:
para observar la incidencia de enfermedad a lo largo del Es la nica pregunta de este apartado en la que han pedido un
tiempo es un: clculo numrico. Nos preguntan directamente el clculo del
1. Estudio de casos y controles. NNT, uno de los ndices utilizados en el anlisis de los
2. Ensayo clnico. resultados de un ensayo clnico. Simplemente no tenemos que
3. Estudio de una serie de casos. limitar a aplicar la frmula. Esta pregunta ha cado todos los
4. Estudio transversal. aos en las ltimas 3 o 4 convocatorias. Este ao no se han
5. Estudio de cohortes. molestado ni en cambiar los nmeros. El NNT (nmero de
pacientes a tratar para evitar un evento) se deriva de la
REDUCCIN ABSOLUTA DEL RIESGO. La reduccin absoluta del
La respuesta correcta es: 5
riesgo es la diferencia de incidencias del evento en el grupo
sometido al tratamiento experimental y el grupo control. En
COMENTARIO: nuestra pregunta la mortalidad en el grupo placebo es del 10%
Se trata de una pregunta sencilla y clsica. Casi todos los aos y en el grupo experimental 5%. Por tanto, la RAR es del 5%.
caen preguntas de este tipo. Nos dan informacin sobre el Esto quiere decir gracias al tratamiento evitamos 5 muertes por
diseo de un estudio epidemiolgico y nos piden que cada 100 pacientes tratados. Por tanto, deberemos tratar a 20
identifiquemos con cul se corresponde. Vemos que se trata de pacientes con el nuevo frmaco para evitar una muerte (opcin
un estudio analtico, el investigador pretende estudiar la 2 correcta).
relacin entre distintos factores de riesgo y la aparicin de
enfermedad. Esto nos permite descartar estudios descriptivos
(opciones 3 y 4). Vemos que si bien el investigador desempea
un papel pasivo, ya que no ha decidido a qu factores de riesgo
se van a exponer los participantes ni el nivel de exposicin. Tan 202
solo se limita a observar lo que ocurre a lo largo del tiempo. Es,
por tanto, un estudio de tipo obsevacional (opcin 2 incorrecta).
Una vez llegados a este punto, solo nos queda fijarnos en el
Respecto a la utilizacin del placebo como grupo
criterio utilizado para la seleccin de los sujetos. En relacin a
control en los ensayos clnicos, seale la respuesta
esto vemos que se han seleccionado en funcin del factor de
INCORRECTA:
riesgo, estando todos sanos al inicio del estudio. En el
1. La comparacin con placebo es la nica forma de
seguimiento se pretende estudiar la aparicin de enfermedad.
evaluar la eficacia absoluta de un frmaco.
La pregunta no est describiendo claramente un diseo de
2. el uso del placebo puede presentar problemas ticos
cohortes (opcin 5 correcta).
cuando existe un tratamiento de eficacia probada para la
enfermedad que se evala.
3. El uso de placebo podra estar justificado cuando la
enfermedad tiene carcter banal.
4. El uso de placebo est justificado si no existe un
201
tratamiento de referencia de eficacia demostrada o ste
presenta efectos adversos muy graves.
5. En los ensayos clnicos controlados con placebo no se
En un ensayo clnico aleatorizado, doble-ciego y debe informar al paciente de que puede recibir placebo
controlado con placebo, se evalu el efecto sobre la porque se rompera en enmcascaramiento.
mortalidad de un nuevo frmaco en pacientes con
hiperlipidemia y sin antecedente de cardiopata isqumica.
La respuesta correcta es: 5
Despus de un seguimiento medio de cindo aos se
encontr una mortalidad del 10% en el grupo placebo y
del 5% en el grupo de tratamiento (diferencia COMENTARIO:
estadsticamente signiticativa con p<0.05). Calcule en Se trata de una pregunta acerca del uso de placebo. Es la
NNT, que se define como el nmero de pacientes que segunda vez que preguntan sobre este aspecto. Hemos de
tenemos que tratar con el nuevo frmaco durante 5 aos tener en cuenta los grupos control de un ensayo clnico pueden
para evitar una muerte: ser de muy distinto tipo. Entre ellos, se encuentra el placebo,
1. 5%. que es el nico que permite valorar la eficacia absoluta del
2. 20. tratamiento. El resto de controles (histricos, otro frmaco o
3. 0.5. una intervencin no farmacolgica como el tratamiento
4. 50%. quirrgico o la educacin sanitaria) solo nos pueden informar de
5. 1. la eficacia relativa, esto es, en comparacin con otra medida. El
uso de placebo plantea problemas de ndole tica ya que no es
lcito negar un tratamiento de eficacia demostrada. Lo ms
La respuesta correcta es: 2
importante en relacin con este aspecto queda recogido en las
opciones 2, 3 y 4 que son ciertas. Es importante tener en
cuenta que es ms fcil encontrar diferencias si comparamos
con placebo que si comparamos con otro frmaco. La respuesta
incorrecta es la 5, y esto es precisamente lo que preguntaron
en el MIR. En el documento del consentimiento informado
queda recogida la informacin relevante acerca del diseo y
objetivo del estudio en el que se le propone participar. En caso
de que sea un estudio controlado con placebo el paciente debe
conocer que entre las alternativas de tratamiento que podra
llegar a recibir tras la aleatorizacin est el placebo (sustancia
inactiva). Esto no afecta al enmascaramiento porque es una
informacin proporcionada antes de la asignacin y de que el
individuo de su consentimiento.

203

41
aleatoriamente a ser tratados con eprosartan o placebo.
Estamos planeando hacer un ensayo clnico para 4. Un ensayo clnico paralelo con diseo factorial de 2x2
comparar la eficacia de dos antibiticos para el en el que cada paciente se asigna aleatoriamente a recibir,
tratamiento de la neumona. Cul de los siguientes datos por un lado eprosartan o placebo, y por otro lado vitamina
NO es til para calcular el tamao de la muestra?: E o placebo.
1. El nmero de pacientes que ingresan con diagnstico de 5. Es mejor hacer dos ensayos clnicos separados, uno
neumona en nuestro centro, que es de 5 a la semana. para eprosartan y otro para vitamina E, cada uno de ellos
2. El porcentaje de pacientes que se curan con el frmaco con un diseo paralelo, aleatorizado, doble-ciego y
de referencia, que en los estudios publicados es un 85%. controlado con placebo.
3. La probabilidad de obtener un resultado falso positivo
(error tipo 1), que la fijamos en un 5%.
4. La proporcin de prdidas de pacientes previstas, que La respuesta correcta es: 4
en otros estudios similares de la literatura era de un 10%.
COMENTARIO:
5. La diferencia mnima clnicamente relevante entre los De nuevo nos preguntan acerca de distintas variantes de diseo
dos frmacos, que la consideramos de un 10%. del ensayo clnico. La palabra clave para contestar
correctamente esta pregunta est en el enunciado. Necesitan
La respuesta correcta es: 1 un diseo en el que los tratamiento se puedan comparar
administrados solos y en combinacin. El nico que permite
esto es el diseo factorial. En la siguiente tabla se muestra
COMENTARIO: cmo seran los grupos de tratamiento para este enunciado.
La predeterminacin del tamao muestral es tambin un tema
clsico que se ha preguntado en las 3 ltimas convocatorias.
Para contestar esta pregunta lo primero que tenemos que
analizar es el tipo de estudio o diseo que nos plantean. Como
se trata de un ensayo clnico sabemos que comportar una
comparacin de los grupos de tratamiento. As sabemos que
tenemos que tener en cuenta:

- Objetivo del estudio y la variable principal.


- Magnitud del efecto que queremos detectar.
- Variabilidad de la variable principal.
- Errores tipo I y II.
- Poder o potencia estadstica.
- Proporcin de pacientes en ambos grupos. 205
- Prdidas esperadas.
- Tipo de contraste.
-
Con esto fcilmente podemos deducir que la respuesta a la Cul de los siguientes objetivos NO se puede
pregunta es la opcin 1, ya que esto no est en nuestra lista. evaluar en los ensayos clnicos de fase I?:
Este aspecto hace referencia a las dificultades que podemos 1. Seguridad y tolerabilidad del frmaco y bsqueda de la
encontrar en el reclutamiento. Si nuestro Hospital atiende dosis mxima tolerada.
muchas neumonas dicho reclutamiento ser ms fcil. Por 2. Farmacocintica en dosis nica y en dosis mltiple.
ltimo un breve comentario en relacin a la magnitud del 3. Farmacodinmica.
efecto. En condiciones ideales lo mejor sera ser capaces con 4. Dosis ms eficaz para el tratamiento de una patologa
nuestro diseo de detectar cualquier diferencia que pudiera concreta.
haber entre los tratamiento por pequea que esta fuera. En la 5. Biodisponibilidad y bioequivalencia.
prctica hay que llegar a un equilibrio entre esto (hara
necesario grandes tamao muestrales, incrementando los La respuesta correcta es: 4
costes) y las diferencias que son clnicamente relevantes y por
tanto, pueden modificar nuestra actitud. Por eso, la magnitud
del efecto que queremos detectar se hace corresponder con la COMENTARIO:
diferencia que consideramos relevante desde el punto de vista Los ensayos clnicos (EC) en humanos tienen 4 fases. La fase I
clnico. evala la dosis txica del frmaco, la farmacocintica y la
farmacodinamia. La fase II valora la dosis/respuestas, dando
una informacin preliminar sobre la eficacia del frmaco. La
fase III valora la eficacia de un nuevo frmaco. La fase IV har
referencia a la farmacovigilancia, realizando el seguimiento
204 postcomercializacin del frmaco. En consecuencia, la opcin
falsa es la 4.

Cul sera el diseo ms adecuado para evaluar el


efecto sobre la morbimortalidad cardiovascular de un
frmaco antagonista de los receptores de angiotensina 206
(eprosartan) y de la vitamina E y de su combinacin en
pacientes de alto riesgo?:
1. Un estudio aleatorizado y doble-ciego, con diseo
paralelo de tres brazos, de tal forma que los pacientes se Si en los resultados de un ensayo clnico se
asignan aleatoriamente a recibir tratamiento con especifica que se ha realizado un anlisis por intencin de
eprosartan, vitamina E o placebo. tratar qu poblacin se incluye en dicho anlisis?:
2. Un estudio aleatorizado y doble-ciego, con diseo 1. Todos los pacientes aleatorizados, aunque no hayan
curzado en el que los pacientes primero son tratados con completado el estudio.
eprosartan o placebo durante 1 ao y despus son 2. Todos los pacientes que no han abandonado el estudio
tratados con vitamina E o placebo durante otro ao. por acontecimientos adversos.
3. Un estudio de cohortes en el que los pacientes que 3. Todos los pacientes que han completado el seguimiento
tienen una dieta rica o poble en vitamina E se asignan previsto.

42
4. Todos los pacientes que han completado el tratamiento alguno de los grupos encontramos algn valor de p
del estudio. significativo.
5. Todos los pacientes que han recibido ms del 80% del 2. Nos hemos de plantear si el tamao de muestra es
tratamiento y que han completado el seguimiento hasta la suficiente, si pensamos que no lo era continuaremos
ltima visita. aadiendo ms casos a nuestro estudio hasta obtener un
resultado estadsticamente significativo.
3. Si el tamao de muestra era insuficiente, utilizaremos la
La respuesta correcta es: 1
informacin del presente estudio para el diseo de uno
nuevo revisando nuestras hiptesis de trabajo y la
COMENTARIO: predeterminacin del tamao de la muestra.
A la hora de realizar el anlisis de los resultados en un ensayo 4. Acabaremos concluyendo que los dos protocolos son
clnico (EC) podemos realizar dos tipos: -Anlisis por protocolo iguales, al no haberse detectado diferencias significativas.
donde se incluyen slo a aquellos pacientes que han cumplido
los requisitos de protocolo y /o han finalizado el estudio. 5. Nos hemos dado cuenta de la existencia de un sesgo
-Anlisis por intencin de tratar donde se incluyen a todos los que no habamos tenido en cuenta ni en el diseo ni en la
pacientes aunque no hayan finalizado el estudio o hayan ejecucin, por tanto, debemos utilizar un mtodo
cambiado de grupo. Por lo tanto la opcin correcta es la 1. estadstico ms sofisticado que permita la correccin del
mismo para seleccionar el protocolo mejor sin la menor
duda.

La respuesta correcta es: 3


207

COMENTARIO:
El resultado del estudio es no significativa, es decir, que no
Debemos evaluar los resultados de 3 existe suficiente evidencia para rechazar la hiptesis nula. Esto
medicamentos (A, B Y C) en el tratamiento de pacientes puede ser porque el tamao de la muestra es insuficiente.
con dficit congnitivo ligero. Los diseos implementados Aunque el presente estudio no ha encontrado diferencias
en los tres casos han sido muy similares y se ha usado significativas nos puede servir este estudio para realizar uno
lacebo como grupo control en los tres casos. Se han nuevo en el que se pueda predeterminar mejor el tamao
observado los siguientes resultado de significacin muestral.
estadstica y de porcentaje de reduccin absoluta en la
progresin a demencia en las comparaciones contra
placebo: A versos Placebo: 3%, p<0,001; B versus
Placebo: 8%, p=0.041; C versus Placebo 1% p=0,021.
Cul de las siguientes afirmaciones se puede concluir a
209
partir de los presentes resultados?:
1. El valor de p demuestra que el medicamento con un
efecto de mayor magnitud es del A.
2. El valor de significacin nos indica que el medicamento Los resultados que aporta un ensayo clnico con un
con un efecto de menor magnitud es el B. nuevo tratamiento para la hiperplasia benigna de
3. El medicamento B es el que muestra un efecto de mayor prstata, comparado con un alfa-bloqueante, se basan en
magnitud frente a placebo. la variable principal fosfatasa cida. Se observa una
4. El medicamento C es mejor que el B ya que es ms disminucin de la misma con el nuevo tratamiento
significativo. comparativamente con el de referencia de 30% (IC
5. El valor de significacin nos indica que el medicamento 95%=27-32.5%), siendo estadsticamente significativa.
con un efeco de menor magnitud es el A. Recomendara cambiar a este nuevo tratamiento a sus
pacientes?:
1. No, porque el tratamiento elegido como referencia no es
La respuesta correcta es: 3
el adecuado.
2. Si, porque el intervalo de confianza es muy estrecho.
COMENTARIO: 3. Si, ya que los resultados muestran diferencias
En esta pregunta nos hablan de 2 conceptos: porcentaje de estadsticamente significativas.
reduccin absoluta de riesgo y el nivel de significacin 4. No, ya que la variable principal utilizada no es
estadstica. El nivel de significacin estadstica hace referencia a relevante.
la probabilidad de equivocarnos, mientras que la reduccin 5. No, porque la reduccin obtenida de la fosfatasa cida
adherente de riesgo (es reduccin absoluta de riesgo) es una no es suficientemente importante.
medida que implica la magnitud del efecto. Como las 3
corporaciones son significativas, el medicamento que muestra
La respuesta correcta es: 4
un efecto de mayor magnitud ser aquel en la que el porcentaje
de resolucin absoluta sea mayor, opcin 3.
COMENTARIO:
En este estudio se comparan 2 tratamientos y se observan
como con un tratamiento nuevo se observa una reduccin
significativa en las fosfatasa cida para la HBP. Sin embargo,
208 aunque la reduccin es significativa, la variable principal no es
relevante y por tanto no se puede recomendar el cambio de
tratamiento.
Acabamos de finalizar un estudio cuyo objetivo era
evaluar cual de los dos protocolos de actuacin en nuestro
centro hospitalario es mejor. Se ha obtenido un resultado
no estadsticamente significativo (p>0,05) en nuestro
210
contraste de hiptesis. Cul de las siguientes respuestas
es correcta?:
1. Debemos seguir explorando los datos dividiendo
nuestra muestra en diferente subrupos para ver si en

43
Respecto a los sesgos que pueden aparecer en un
ensayo clnico, se pueden tomar distintas medidas para
evitarlos. Seale la respuesta INCORRECTA: 212
1. Para evitar el sesgo de seleccin (diferencias
sistemticas en los dos grupos de comparacin) se debe
ocultar la secuencia de aleatorizacin. Tenemos que evaluar los resultados de un ensayo
2. Para evitar diferencias sistemticas en los cuidados clnico que compara un nuevo antihipertensivo respecto a
recibidos por los dos grupos de comparacin (sesgo de otro considerado desde el punto de vista clnico como un
cointervencin) se debe comparar con un control buen estndar, y donde la reduccin de la presin arterial
histrico. diastlica (TAD) se predefini como la variable principal.
3. Para evitar una distribucin no homognea de los Suponemos que tanto el diseo como la ejecuccin del
principales factores predictores, conocidos o no, de la estudio son correctos. Los resultados indican que el nuevo
variable de resultado, se asigna aleatoriamente a los tratamiento es ms efectivo ya que reduce ms la TAD,
participantes a cada grupo de tratamiento. concretamente en media (intervalo confianza al 95%
4. Para evitar diferencias sistemticas en los abandonos, bilateral) reduce 0,5 (0,2 a 0,7) mmHg ms que el grupo
prdidas o cumplimiento teraputico entre los dos grupos control, con p=0,001. Cul de las siguientes afirmaciones
de comparacin (sesgo de atricin), se realiza un anlisis es correcta?:
por intencin de tratar. 1. El valor de p (0.001) encontrado demuestra con una
5. Para evitar el sesgo de informacin (diferencias probabilidad nula de equivocarnos que el nuevo
sistemticas en la evaluacin de las variables de resultado tratamiento es mejor.
entre los dos grupos de comparacin), se realiza un 2. En base al valor observado de p (0.001) se puede
enmascaramiento de la intervencin. concluir que la magnitud de la reduccin de TAD del nuevo
medicamento en relacin al control es de gran relevancia
La respuesta correcta es: 2 clnica.
3. Si yo acepto que el tratamiento nuevo es el mejor, me
equivocara slo con una probabilidad de 0,001.
COMENTARIO: 4. La reduccin de TAD es mayor con el nuevo
Los sesgos, errores sistemticos nos pueden invalidar el tratamiento, pero la mejora que en promedio ofrece en
resultado de un estudio. Para evitar el sesgo de seleccin, relacin al tratamiento control no sobrepasara 0.7 mmHg
debemos aleatorizar. La opcin 2 es falsa, puesto que cuando en mejor de los casos, teniendo en cuenta un error alfa o
utilizamos un control histrico (grupo de similares tipo I del 5% bilateral.
caractersticas pero de una poca anterior), lgicamente los 5. La estimacin puntual y los intervalos de confianza no
cuidados recibidos no van a ser igual, al comparar grupos de aportan informacin de la magnitud del efecto de la
pocas distintas. En consecuencia, para evitar este sesgo, comparacin entre ambos tratamientos.
debemos comparar grupos concurrentes. El anlisis por
intencin de tratar es el mejor anlisis porque valora a todos los
pacientes independientemente de si han finalizado o no el La respuesta correcta es: 4
estudio o si se han cambiado de grupo.
COMENTARIO:
Se plantea un contraste de hiptesis con las variables: Dos
tipos de tratamiento y 2. Reduccin de TAD. Los resultados
alcanzan la significacin estadstica, para una diferencia entre
211 tratamientos de 0,5. El intervalo de confianza oscila (0,2-0,7),
95% de confianza. Por tanto, en el mejor de los casos para ese
nivel de significacin, la mayor diferencia previsible entre los
Qu es un ensayo clnico secuencial?: tratamientos es 0,7.
1. Un ensayo para cuya realizacin es necesario conocer
los resultados de estudios previos ms bsicos.
2. Un ensayo en que la decisin de reclutar al siguiente
paciente, par de pacientes o bloque de pacientes depende
de la diferencia entre tratamientos obtenido hasta ese 213
momento.
3. Un ensayo en que la fecha de comienzo y finalizacin es
distinta para cada paciente, dependiendo de la Seale cul de los siguientes NO est incluido entre
disponibilidad de sujetos con los criterios de seleccin los 4 principios bsicos de la biotica:
pertinentes. 1. Autonoma o respeto por las personas.
4. Un ensayo con ventajas ticas ya que en caso de 2. Beneficiencia.
empate entre tratamientos es el azar el que decide a que 3. No maleficencia.
grupo se asigna cada nuevo paciente, mientras que en 4. Justicia, entendido como justicia distributiva.
caso de que hasta el momento haya un tratamiento 5. Confidencialidad.
aparentemente superior, es ese precisamente el que
reciben los nuevos pacientes.
5. Un ensayo que est perfectamente incardinado dentro La respuesta correcta es: 5
del programa de desarrollo de un nuevo tratamiento.
COMENTARIO:
La respuesta correcta es: 2 La confidencialidad no es un principio bsico como tal, ya que
est incluido dentro de otro ms amplio, que es el de la
autonoma o respeto por las personas.
COMENTARIO:
Un ensayo clnico secuencial se caracteriza por no conocer a
priori el tamao muestral. Por tanto, es un estudio en el que los
pacientes son incluidos en el estudio secuencialmente. El
ensayo finalmente se detendr cuando el tamao muestral que 214
se ha ido sumando progresivamente sea tal, que consigamos
una significacin estadstica ( p. Ej. P<0,05).

44
COMENTARIO:
Cul de las siguientes tcnicas del Sistema Un ensayo clnico es un estudio en el que se debe determinar a
Espaol de Farmacovigilancia se conoce como "tarjeta priori el tamao de la muestra, el tipo de la muestra a utilizar,
amarilla"?: cuales son las caractersticas de las variables, as como cuantos
1. Ensayos clnicos postcomercializacin. pacientes es previsible perder. Una vez realizado esto, el
2. Estudios de cohortes. estudio se pone en marcha (suponemos que sin sesgos y con n
3. Estudios de casos y controles. apropiado) que determinen si hay diferencias significativas
4. Notificacin espontnea. habitualmente entre dos frmacos.
5. Monitorizacin intensiva intrahospitalaria.

La respuesta correcta es: 4

217
COMENTARIO:
Una vez que se encuentra un frmaco en el mercado (ha
pasado la fase 3), todava puede aparecer efectos adversos bien
infrecuentes, bien que aparecen a lo largo plazo, que deben ser En el momento actual, Espaa y la Regin Europea
detectados en la fase 4 o de farmacovigilancia, la notificacin han sido declaradas libres de polio desde el 21 de junio del
de stos efectos se notifica a la agencia de medicamentos a 2002. Cul de las siguientes afirmaciones es correcta?:
travs de unos formularios conocidos como tarjetas amarillas. 1. En el momento actual en Espaa no se recomienda ya la
vacunacin antipoliomieltica.
2. En el momento actual en Espaa solo se vacuna frente a
la poliomielitis con vacuna atenuada.
3. En el momento actual en Espaa solo se vacuna frente a
215 la poliomielitis con vacuna inactivada.
4. Desde el ao 2002 en Espaa no se vacuna frente a la
poliomielitis.
5. En el momento actual en Espaa se vacuna frente a la
En relacin con el valor predictivo de una prueba poliomielitis con un procedimiento secuencial, primero con
diagnstica, seale la respuesta correcta: vacuna inactivada y luego con vacuna atenuada.
1. El valor predictivo no depende de la prevalencia de
enfermedad.
2. Cuando la prevalencia de la enfermedad es baja es La respuesta correcta es: 3
improbable que el paciente con una prueba con resultado
positivo tenga realmente la enfermedad. COMENTARIO:
3. Cuando la prevalencia de la enfermedad es baja es Existen dos tipos de vacuna frente a la polio. Por una parte, la
probable que el paciente con una prueba con resultado oral o Sabn, por otra la inactiva, parenteral o Salk.
positivo tenga realmente la enfermedad. Tradicionalmente se administraba la Salk a los sujetos
4. Cuando la prevalencia de la enfermedad es alta es inmunocomprometidos, pero debido al riesgo potencial de polio
probable que el paciente con un resultado positivo no por la vacuna Sabn, en la actualidad se vacuna a todos los
tenga la enfermedad. sujetos con la inactiva.
5. Cuando la prevalencia de la enfermedad es baja no hay
aumento de falsos positivos.

La respuesta correcta es: 2


218

COMENTARIO:
El VPP se refiere a la probabilidad que tiene un sujeto, de
habiendo sido positivo en el test que queremos validar, resulta En cuanto a las infecciones hospitalarias de la
ser en efecto un enfermo. Por tratarse de un parmetro de herida quirrgica, seale lo cierto:
validez externa, depende de la prevalencia, si la prevalencia es 1. Son las infecciones hospitalarias ms frecuentes.
baja habr ms sujetos falsos positivos, por lo que en esta 2. Los microorganismos responsables provienen en su
situacin, el valor predictivo positivo disminuir. (VPP= mayor parte del quirfano.
VP/VP+FP). 3. La profilaxis antibitica es eficaz cuando se administra
justo antes de la intervencian.
4. Generalmente se presentan en forma de epidemias.
5. El grado de riesgo de infeccin qirrugica no guarda
relacin con el tipo de procedimiento quirrgico realizado.
216
La respuesta correcta es: 3

De los siguientes, que aspecto debe tenerse en COMENTARIO:


cuenta en ltimo lugar a la hora de interpretar los La infeccin de la herida es una causa frecuente de infeccin en
resultados de un ensayo clnico: los servicios quirrgicos pero no supone la infeccin hospitalaria
1. Las prdidas de pacientes y los motivos de las mismas. ms frecuente. La probabilidad de infeccin de la herida viene
2. La significacin estadstica de los resultados. determinada por el tipo de operacin realizada. En ciruga
3. Un diseo adecuado para evitar posibles sesgos. limpia no es necesaria la profilaxis antibitica por el bajo riesgo
4. Caractersticas de los pacientes incluidos. de infeccin. Sin embargo, en ciruga limpia contaminada y
5. La variable de medida utilizada y a cuanto tiempo se contaminada, s se utiliza la profilaxis antibitica. La profilaxis
evala. antibitica se inicia en la induccin anestsica y no se debe
prolongar ms de 48 horas.
La respuesta correcta es: 2

45
219 contrastada. La indicacin del ganciclovir para citomegalovirus
es tambin fcil. La azidotimidina es ms conocida como
zidovudina o AZT. Es el 1er antiretroviral,inhibidor de la
transcriptasa inversa. La Amantidina y rimantidina son frmacos
En qu tipo de anlisis de evaluacin econmica,
cuya indicacin como antivirales es la infeccin por Influenzae
el anlisis incremental es de eleccin a la hora de analizar
A.
e interpretar los resultados?:
1. Anlisis de minimizacin de costes.
2. Anlisis coste-beneficio.
3. Anlisis coste-efectividad.
4. Anlisis coste-consecuencia.
222
5. Estudios de coste de la enfermedad.

La respuesta correcta es: 3


Cules de las siguientes son manifestaciones
clnicas tpicas de la intoxicacin aguda por un opioide
COMENTARIO: agonista m puro?:
El anlisis incremental es de eleccin a la hora de analizar e 1. Midriasis, activacin generalizada, convulsiones.
interpretar los resultados en el anlisis coste-efectividad 2. Miosis, depresin respiratoria y coma.
(identificar y cuantificar los costes de procedimientos 3. Depresin respiratoria, hipertensin arterial y miosis.
alternativos para alcanzar un objetivo cuyos resultados vienen 4. Midriasis, hipotermia, hipotensin, bradicardias.
expresados por unidad de efecto o efectos por unidad de coste). 5. Depresin respiratoria, hipertermia y taquiarritmias.

La respuesta correcta es: 2

220 COMENTARIO:
Pregunta fcil sobre la intoxicacin por opiceos. Se explica en
clase. La combinacin de Miosis y depresin respiratoria y
neurolgica es clsica.
En un anlisis de evaluacin econmica, los costes
derivados de las horas/das de trabajo perdidos por
absentismo laboral se denominan:
1. Costes mdicos directos.
2. Costes no-mdicos directos.
223
3. Costes indirectos.
4. Costes intangibles.
5. Costes sociales.
Cul de los siguientes antibiticos antibacterianos
no acta inhibiendo la sntesis de la pared celular?:
La respuesta correcta es: 3
1. Cefalosporinas.
2. Carbopenems.
COMENTARIO: 3. Macrlidos.
Los costes derivados de las horas/das de trabajo perdidos por 4. Vancomicina.
absentismo laboral se denominan costes indirectos. 5. Penicilinas.

La respuesta correcta es: 3

221 COMENTARIO:
Es una pregunta fcil sobre el mecanismo de accin de los
antibiticos. Adems preguntan sobre familias importantes de
los mismos: Betalactmicos, Macrlidos, Vancomicina. Se
Una de las afirmaciones referidas a frmacos
explica especficamente en clases, 1 y 2 vuelta y se recoge en
antivricos es FALSA:
el manual.
1. El aciclovir posee accin selectiva contra los
herpesvirus que codifican una timidina cinasa.
2. El aciclovir es especialmente eficaz en las infecciones
por virus del herpes simple, como encefalitis, herpes
diseminado y otros cuadros graves.
224
3. El ganciclovir tiene una actividad significativa sobre
citomegalovirus,
4. La azidotimidina, un anlogo de la timidina, inhibe la
transciptasa inversa del VIH. Cul de las siguientes situaciones NO favorece la
5. La amantidina y la rimantidina, anlogos de los aparicin de interacciones clnicamente importantes?:
nuclesidos, se administran en forma de aerosol para el 1. Cuando se utilizan frmacos con estrecho margen
tratamiento de los nios con bronquiolitis grave por el teraputico.
virus sincitial respiratorio. 2. En pacientes ancianos.
3. Si la unin a protenas plasmticas es escasa.
4. Si existen modificaciones de pH sanguneo u urinario.
La respuesta correcta es: 5
5. Si existe polimedicacin.

COMENTARIO:
La respuesta correcta es: 3
Pregunta sobre frmacos antivirales. El grado de dificultad es
bajo. Se recoge en el Manual y hay una pregunta especfica en
al 1 vuelta (test farma). El Aciclovir es el antiviral ms COMENTARIO:
preguntado en el MIR y su eficacia en Herpes Virus est Pregunta de dificultad media sobre farmacocintica y en

46
concreto interacciones medicamentosas. Las premisas 1, 2 y 5
son fciles de descartar. La dificultad surge de elegir entre la 3
y la 4. Sin embargo las modificaciones del PH sanguneo y
227
urinario puede alterar la eliminacin de forma importante por lo
cual surgen interacciones. Si la unin a protenas plasmticas es
escasa hay menos probabilidad de que otros frmacos influyan
sobre el efecto teraputico. Tambin es menor la posibilidad de Cul de los siguientes microorganismos infecta
que estados patolgicos como la desnutricin o principalmente las clulas del endotelio vascular?:
hipoalbuminemia modifiquen la concentracin libre del frmaco 1. Salmonella Typhi.
que es la activa farmacolgicamente. 2. Richettsia Typhi.
3. Haemophilus Influenzae.
4. Coxiella Burnetti.
5. Streptococcus Agalatiae.

225
La respuesta correcta es: 2

COMENTARIO:
Cul de las siguientes caractersticas NO es propia
Las rickettsias tienen tropismo por el endotelio vascular. Esto
de los aminoglucsidos?:
justifica el exantema y otras manifestaciones graves de estas
1. Algunos frmacos de esta familia presentan buena
infecciones como el edema pulmonar o cerebral.
actividad frente a Mycobacterium tuberculosis.
2. La actividad bactericida depende de las concentraciones
alcanzadas, por lo que pueden administrarse en dosis
nica diaria.
3. Todos los frmacos presentan buena actividad frente a
bacilos Gram negativos. 228
4. La toxicidad producida por los frmacos de este grupo
se relaciona con el mantenimiento de una concentracin
mnima (concentracin previa a una dosis) elevada. En la malaria, la forma de plasmodio transmitida
5. Cuando existe inflamacin menngea resultan muy del mosquito al hombre es el:
tiles en el tratamiento de las infecciones bacterianas 1. Esporozito.
localizadas en el sistema nervioso central, especialmente 2. Gametocito.
las producidas por bacterias Gram negativas. 3. Merozoito.
4. Hipnozoito.
La respuesta correcta es: 5 5. Taquizoito.

COMENTARIO: La respuesta correcta es: 1


Pregunta fcil sobre aminoglucsidos ya que una de las cosas
importantes de las mismas es que no atraviesan la barrera COMENTARIO:
hematoenceflica y que si fuera necesario su uso en infecciones La picadura del mosquito Anopheles inocula esporozoitos del
de SNC, hay que administrarles por va intratecal. Se explica en Plasmodium que se dirigen al hgado del sujeto infectado.
manual y en clase.

229
226

Culde los siguientes tipos de microorganismos


Todas las siguientes afirmaciones referidas al virus NO es un parsito intracelular obligado y puede crecer en
de la inmunodeficiencia humana (VIH), excepto una, son medios de cultivo artificiales?:
correctas. Seale la respuesta INCORRECTA: 1. Chlammydia.
1. La protena CD4 de la superficie de las clulas T es el 2. Mycoplasma.
receptor para el virus. 3. Coxiella.
2. Los pacientes infectados por el virus producen 4. Adenovirus.
anticuerpos frente a las glicoprotenas (gp 120 y gp 41) de 5. Rickettsia.
la envuelta y el antgeno interno grupo especfico (p24).
3. La prueba de Western blot es ms especfica que el
ELISA para diagnosticar la infeccin por el virus. La respuesta correcta es: 2
4. Un problema importante para estudiar los anticuerpos
frente al VIH es su reactividad cruzada con el virus COMENTARIO:
linfotrpico I de clulas T humanas. Mycoplasma es una bacteria que puede tardar hasta 2 semanas
5. Existe una importante diversidad antignica en las en crecer en medios artificiales, por lo que no se utiliza de
glicoprotenas de la envoltura del VIH. manera rutinaria en clnica, pero no es un parsito intracelular
obligado.
La respuesta correcta es: 4

COMENTARIO:
Las pruebas serolgicas para el diagnstico de la infeccin por 230
VIH no presentan habitualmente problemas de reacciones
cruzadas. La variacin antignica de las protenas de la
envoltura es la razn de la dificultad en el desarrollo de una
vacuna frente a este virus.

47
1. Sinucleina.
Cul es el agente causal de infeccin urinaria ms 2. Tau hiperfosforilada.
frecuente en una mujer sin enfermedades generales o 3. Huntingtina.
locales de base?: 4. Ataxina.
1. Proteus Mirabilis. 5. Protenas prinica.
2. Klebsiella Pneumoniae.
3. Corynebacterium Urealiticum.
4. Escherichia Coli. La respuesta correcta es: 2
5. Staphylococcus Saprophyticus.
COMENTARIO:
La respuesta correcta es: 4 En los ovillos neurofibrilares que aparecen tpicamente en la
histologa de la enf. de Alzheimer, se identifican 2 protenas:
Ubiquitina y Taurina hiperfosforilada (opcin 2).
COMENTARIO:
E. Coli es la causa ms frecuente de infeccin urinaria.

234

231

En la esclerosis lateral amiotrfica, la lesin se


localiza en:
Un granuloma epitelioide est constituido por: 1. Corteza entorrinal.
1. Macrfagos tansformados en clulas epitaelioides. 2. Ncleo Caudado.
2. Clulas epiteliales. 3. Sustancia Negra.
3. Linfocitos, histiocitos xantomatosos y clulas 4. Corteza cerebelosa.
epiteliales. 5. Asta anterior de la mdula.
4. Acmulos leucocitarios de apariencia epitelial.
5. Linfocitos y clulas gigantes multinucleadas de tipo
Langhans. La respuesta correcta es: 5

La respuesta correcta es: 1 COMENTARIO:


La esclerosis lateral amiotrfica es la forma ms frecuente de
enfermedad progresiva de la neurona motora. La lesin se
COMENTARIO: puede localizar en asta anterior de la mdula.
En esta pregunta se nos interroga sobre la estructura del
granuloma epiteloide. El granuloma epiteloide es de estructura
tpica de la inflamacin crnica. En ellos aparecen tpicamente
macrfagos transformados en clulas epitelioides (opcin A) y
clulas gigantes multinucleadas tipo Langhans (respuesta 5) 235
rodeadas de una corona de linfocitos.

El estudio microscpico de un ndulo tiroideo


muestra nidos y trabculas de clulas poligonales y
232 fusiformes inmersos en un estroma con sustancia
amiloide. Qu diagnstico realizara?:
1. Carcinoma papilar.
2. Carcinoma folicular.
Cul de los siguientes rasgos morfolgicos NO es 3. Carcinoma anaplsico.
propio de la colitis ulcerosa?: 4. Carcinoma de clulas de Hrthle.
1. Microabscesos crpticos. 5. Carcinoma medular.
2. Formacin de pseudoplipos.
3. Displasia epitelial.
4. Engrosamiento mural. La respuesta correcta es: 5
5. Lesin mucosa crnica.
COMENTARIO:
La respuesta correcta es: 4 Pregunta sencilla en relacin a la anatoma patolgica en el
cncer de tiroides, En la pregunta se describe la existencia de
clulas parafoliculares y sustancia amiloide, hecho que
COMENTARIO: caracteriza al carcinoma medular.
Esta pregunta podras haberla deducido... La colitis ulcerosa
afecta solamente a la mucosa, por lo que difcilmente producir
engrosamiento mural, que es una caracterstica ms propia del
Crohn.
236

233 Es muy importante el conocimiento de las


variaciones anatmicas de la arteria cstica en la ciruga
de la vescula biliar. Cul es el origen ms frecuente de la
arteria cstica?:
En algunas enfermedades neurodegenerativas se 1. Arteria heptica derecha.
produce acmulo intracelular de protenas anormales. Una 2. Arteria heptica izquierda.
de estas enfermedades, la enfermedad de Alzheimer, se 3. Arteria heptica comn.
caracteriza por el depsito intracelular de:

48
4. Tronco celaco. La respuesta correcta es: 3
5. Arteria mesentrica superior.
COMENTARIO:
La respuesta correcta es: 1 El pulmn izquierdo, a diferencia del derecho, tiene ocho
segmentos. Esto se debe a que su lbulo superior se forma
solamente de dos (en vez de tres) y el lbulo de la lngula tiene
COMENTARIO:
solamente uno (al revs que el lbulo medio del pulmn
Pregunta sin importancia que no debe preocuparte. La
derecho, que tiene dos).
estructura de la pregunta invita a elegir una de las tres
primeras opciones, dado su parecido, aparte de que el tronco
celaco y la mesentrica superior estn algo alejadas... En este
caso, es la heptica derecha, conclusin a la que podras haber
llegado por su mayor contigidad anatmica a la vescula que
240
las otras dos.

Con respecto a la embriologa del tiroides y


glndulas paratiroides, una de las siguientes afirmaciones
237 NO es correcta:
1. Las clulas C producen calcitonina y surgen de la 4
bolsa farngea, emigran desde la cresta neural hacia los
lbulos laterales del tiroides.
Donde desemboca el conducto torcico?:
2. El tiroides tiene su origen embriolgico en clulas
1. En la aurcula derecha.
situadas en la lnea media del suelo de la faringe y su
2. En la vena cava superior.
origen es endodrmico.
3. En la vena subclavia izquierda, en su confluencia con la
3. Todos los componentes de la glndula tiroidea adulta
vena yugular interna izquierda.
(clulas foliculares y parafoliculares) tienen origen
4. En la vena yugular derecha.
endodrmico.
5. En el tronco venoso braquioceflico derecho.
4. Las Glndulas Paratiroides Superiores provienen de la
4 bolsa farngea junto con el componente tiroideo lateral
La respuesta correcta es: 3 y las inferiores de la 3 farngea junto con el timo.
5. Teniendo en cuenta la embriologa del tiroides,
cualquier tejido tiroideo en compartimentos laterales del
COMENTARIO: cuello se considera actualmente como metstasis de
Recuerda que el conducto torcico recoge el contenido linftico cncer bien diferenciado de tiroides.
procedente de la cavidad abdominal, drenndolo en la vena
subclavia izquierda. Este elemento anatmico ha sido
preguntado alguna vez en el MIR. La respuesta correcta es: 3

COMENTARIO:
La opcin falsa es la 3, puesto que existen clulas en el tiroides
que son procedentes de la cresta neural.
238

Uno de los siguientes hiatos NO se encuentra en el


diafragma: 241
1. Hiato de Bochdalek.
2. Hiato esofgico.
3. Hiato de Winslow.
Un recin nacido puede experimentar
4. Hiato artico.
sintomatologa transitoria de la enfermedad materna si la
5. Hiato de Mogagni.
madre padece cualquiera de las siguientes patologas
EXCEPTO:
La respuesta correcta es: 3 1. Miastenia gravis.
2. Hipertiroidismo primario.
3. Penfigo vulgar.
COMENTARIO: 4. Herpes gestacional.
El hiato de Winslow no forma parte del diafragma, sino que es 5. Enfermedad celiaca.
la puerta de entrada a la transcavidad de los epiplones,
encontrndose junto al pedculo heptico.
La respuesta correcta es: 5

COMENTARIO:
La pregunta hace referencia a enfermedades que sufridas por la
239 madre pueden debutar clnicamente en el recin nacido. Las
opciones 1, 2, 3 y 4 presentan formas neonatales, la explicacin
fisiopatolgica de las mismas es el paso de anticuerpos de clase
Cuntos bronquios segmentarios tiene cada Ig G producto de estas enfermedades a travs de la placenta
pulmn?: hacia el feto. Para el desarrollo de la Enfermedad Celaca,
1. Diez cada uno. opcin 5, s es necesario que el paciente incluya el gluten en la
2. Doce el derecho y diez el izquierdo. dieta por lo que tpicamente debuta a los 6 meses de vida.
3. Diez el derecho y ocho el izquierdo.
4. Doce cada uno.
5. Diez el derecho y nueve el izquierdo.

49
242
Sobre los fenmenos de rechazo en los trasplantes
alognicos de rganos slidos, seale la respuesta FALSA:
1. Los linfocitos T-CD4 del receptor pueden reconocer las
Los anticuerpos que desaparecen pocos meses molculas HLA-II alognicas expresadas en las clulas
despus de la infeccin permiten detectar una enfermedad presentadoras de antgeno del donante.
actual o muy reciente. Este tipo de anticuerpos pertenece 2. Los linfocitos T-CD4 del receptor pueden reconocer
a la clase: fragmentos de molculas HLA-II y HLA-I alognicas
1. Ig G2. expresadas en las clulas presentadoras de antgeno del
2. Ig E e Ig G3. receptor.
3. Ig A e Ig M. 3. El rechazo hiperagudo se debe a la presencia de
4. Ig D. anticuerpos en el receptor (formados previamente) que
5. Ig G4 e Ig D. frecuentemente van dirigidos contra molculas HLA.
4. En el trasplante heptico el rechazo hiperagudo es
La respuesta correcta es: 3 infrecuente.
5. A menor expresin de molculas B7 sobre las clulas
presentadoras de antgeno del donante, mayor es la tasa
COMENTARIO: de rechazo agudo.
Se nos pregunta sobre la posibilidad de detectar infecciones
recientes mediante la deteccin de Ig especficas frente al
microorganismo culpable de la misma. En la actividad clnica La respuesta correcta es: 5
actual se usa la determinacin de Ig M especfica para el
diagnstico de infecciones recientes o actuales (Respuesta COMENTARIO:
primaria). No es til para diferenciar infecciones recientes de En el trasplante alognico (entre individuos diferentes
crnicas la deteccin de Ig A especfica. Pregunta impugnable. genticamente) se puede producir un tipo de rehcazo
denominado agudo, cuyo mecanismo inmunolgico es celular,
mediado por linfocitos T. La subpoblacin de linfocito T CD4
reconoce especficamente antgenos HLA de clase II mientras
que los T CD8 interaccionan con clase I. La respuesta n 2 es
243 cierta porque las clulas presentadoras de antgenos del
receptor pueden presentar fragmentos de clase I del donante
en sus molculas HLA de clase II que seran reconocidas por sus
Cul de las siguientes citocinas es reconocida por T CD 4. La respuesta n 5 es totalmente falsa porque la
un anticuerpo monocional que se utiliza con xito para el molcula B7 (CD80/CD86) es una molcula de coestimulacin,
tratamiento de la artritis reumatoide?: cuya presencia es totalmente necesaria para la activacin
1. Interleucina 7 (IL-7). completa del linfocito T, de forma que a menor expresin de
2. Interfern gamma (IF-g). esta molcula obtendremos una menor respuesta celular del
3. Factor de necrosis tumoral alfa (TNF-a). linfocito T.
4. Factor de crecimiento transformante beta (TGF-b).
5. Interleucina 4 (IL-4).

La respuesta correcta es: 3 245

COMENTARIO:
Esta es una pregunta que enlaza la Inmunologa bsica con la La deficiencia de adhesin leucocitaria 1 est
clnica, en particular con la reumatologa. En la actualidad causada por mutaciones en el gen que codifica para la
dentro de los frmacos que se usan en la terapia de la artritis beta integrina CD18. Cul de los siguientes es el mtodo
reumatoide se incluyen, los denominados, biolgicos. Entre ms apropiado para diagnosticarla?:
ellos destacan el Infliximab y el Etarnecept, cuya diana 1. Cultivo mixto de linfocitos.
teraputica es el TNF-alfa, que han demostrado un 2. Nefelometra cintica.
enlentecimiento en la progresin de la enfermedad y una 3. Respuesta a mitgenos.
mejora de los sntomas. 4. Microscopa electrnica.
5. Citofluorometra de flujo.

La respuesta correcta es: 5


244
COMENTARIO:
El dficit de adhesin leucocitaria (LAD) se debe a alteraciones
en la molcula CD18, una integrina fundamental para el
correcto desarrollo del proceso de la quimiotaxis. Esta molcula
se expresa como protena de la superficie o de membrana en el
neutrfilo. Para el estudio de los marcadores celulares como el
CD4, CD8, CD19, etc, la tcnica de referencia es la citometra
de flujo, que nos permite de una forma rpida y exacta detectar
deficiencias en esas molculas.

246

50
La enzima 1-alfahidroxilasa, que acta sobre el 25- En relacin con los mecanismos normales de la
hidroxicolecalciferol (25-OH-D) para transformarlo en 1- digestin y absorcin de los alimentos, una de las
25 -dihidroxicolecalciferol (1-25-OH2D) se encuentra en: afirmaciones siguientes es FALSA:
1. El hgado. 1. La lipasa se inactiva en medio cido.
2. En las paratiroides. 2. La absorcin de calcio est facilitada por la vitamina D.
3. En el rin. 3. La Vitamina B12 se absorbe en el leon terminal.
4. En el corazn. 4. Los cidos grasos de cadena media son componentes
5. En el pulmn. constantes de la dieta vegetariana. 5 Las sales biliares
facilitan la absorcin de la grasa.
La respuesta correcta es: 3
La respuesta correcta es: 4
COMENTARIO:
Esta pregunta hace referencia a la fisiologa de la vitamina D. El COMENTARIO:
colecalciferol (vit D3) se produce en la piel por la accin solar y La lipasa pancretica en combinacin con las sales biliares acta
se metaboliza a 25(OH) VITD3 en el hgado. Posteriormente se en la digestin de las grasas; la lipasa se activa con la acidez.
metaboliza en el rin a 1,25 (OH)2 VITD3, llamada calcitriol, Los cidos grasos de cadena media no son componentes de la
que es el metabolito ms activo. dieta vegetariana. La vitamina B12 se absorbe en el ileon
terminal y la absorcin de calcio intestinal est potenciada por
la vitamina D.

247

250
Qu funcin tiene el factor von Willebrand?:
1. Estabiliza la unin de las plaquetas con el colgeno.
2. Interviene directamente en la adhesin entre las En condiciones de presin atmosfrica de 760 mm
plaquetas. Hg y humedad relativa del aire del 100%, qu frmula
3. Estimula la produccin de prostaciclina endotelial. utilizara para calcular la Presin alveolar de oxgeno
4. Inhibe la activacin del factor VIII. (PA02)?:
5. Inhibe la unin de la trombina con las plaquetas. 1. PA02 = Presin arterial de CO2 (PaCO2) - Presin
arterial de O2 (PaO2)/3.
2. PA02 = 100 - 1,25 x Pa02/3.
La respuesta correcta es: 1
3. PA02 = 150 - 1,25 x PaCO2.
4. PA02 = 125 - 1,25 x Pa02/3.
COMENTARIO: 5. PA02 = Pa02 - 1,25 x PaCO2.
El factor vW participa en la adhesin plaquetaria al subendotelio
vascular alterado, uniendo la plaqueta a travs de la GP-Ib de
La respuesta correcta es: 3
su membrana al colgeno subendotelial.

COMENTARIO:
La clave de esta pregunta es aplicar la frmula del gradiente
alveolo-arterial de O2 (D (A-a)O2), es decir, presin Alveolar
248 (PAO2) presin arterial (PaO2); la PaO2 nos la da la
gasometra, luego hay que calcular la PAO2, mediante la
frmula
En el ciclo cardiaco normal cul de las siguientes PAO2 = FiO2 x (Pat PH2O) PCO2/0,8
respuestas es la cierta?:
1. La onda v del pulso venoso coincide con la sstole Si la presin atmosfrica (Pat) es normal, es decir 760 mmHg, y
auricular. la humedad relativa es 100%, que implica que la PH2O es 47, y
2. El cierre de la vlvula artica precede al de la pulmonar, respirando aire ambiente (FiO2 0,21), el resultado de la primera
especialmente durante la inspiracin. parte de la frmula es 150 (0,21 x (760-47). Por otro lado,
3. La fraccin de eyeccin del ventrculo izquierdo es PCO2/0,8 es lo mismo que 1,25 x PCO2. Por eso, la respuesta
superior a 0,85. correcta es la 3.
4. La contraccin auricular activa, puesta de manifiesto
por la onda P del ECG, es diastlica precoz
5. Cuando la frecuencia cardiaca es superior a 100 l.p.m.,
el espacio Q-T del ECG es 0,46 segundos.
251
La respuesta correcta es: 2

COMENTARIO: Un paciente de 46 aos sin antecedentes clnicos


El orden de cierre de las vlvulas semilunares es el mencionado de inters acude al servicio de urgencias por realizar en
en la opcin 2 (primero la vlvula artica y despus la las 8 horas previas dos deposiciones de aspecto melnico.
pulmonar), especialmente durante la inspiracin ya que En las ltimas 48 horas refiere haber tenido molestias
aumenta el retorno venoso a cavidades derechas, retrasndose vagas en hemiabdomen superior. En las anamnesis no
por tanto algo ms el cierre de la pulmonar. describe ingesta previa de frmacos potencialmente
gastroerosivos. Cul le parece la causa ms probabe de la
hemorragia digestiva alta en este paciente?:
1. Desgarro esofgico de Mallory-Weiss.
2. Esofagitis erosiva.
249 3. Varices esofgicas.

51
4. lcera pptica.
5. Neoplasia gstrica. Respecto a la enfermedad de Graves-Basedow NO
es cierto que:
1. Su frecuencia es diez veces menor en los varones.
La respuesta correcta es: 4 2. El 30% de los casos comienza en la infancia.
3. La dermopata es una manifestacin extratiroidea de
COMENTARIO: esta enfermedad y est mediada por la activacin
La causa ms probable de HDA en paciente no alcohlico es la inmunolgica de los fibroblastos.
lcera pptica siendo la duodenal la ms frecuente. 4. El tratamiento con radioyodo puede empeorar la
enfermedad ocular.
5. Las tasas mximas de remisin se alcanzan a los 18-24
meses de tratamiento con antitiroideos.

252 La respuesta correcta es: 2

COMENTARIO:
Un paciente de 67 aos con antecedentes de Pregunta muy sencilla en relacin a la enfermedad de Graves.
insuficiencia cardiaca crnica en tratamiento con Esta enfermedad es caracterstica de mujeres jvenes, siendo
analapril, carvedilol, furosemida y espironolactona acude a muy rara (<5%) en nios (opcin 2 falsa). La manifestacin
urgencias por sensacin de mareo en las ltimas 48 horas. extratiroidea ms frecuente es la oftalmopata, siendo la
Su tensin arterial es de 85/40 mmHg. En la exploracin dermopata bastante infrecuente. El tratamiento se suele
fsica no se objetivan signos de congestin. El ECG realizar con antitiroideos durante un perodo de tiempo
demuestra un bloqueo de rama izquierda a 65 Ipm ya prolongado (18-24m) para alcanzar la remisin de la
conocido previamente. Los valores de creatinina, enfermedad. El radioyodo, que suele ser la 2 opcin
hemoglobina e iones son normales cul de las siguientes teraputica, puede empeorar la enfermedad ocular.
actitudes le parece ms adecuada?:
1. Disminuir dosis de cavedilol.
2. Disminuir dosis de furosemida.
3. Disminuir dosis de enalapril.
4. Disminuir dosis de espironolactona. 255
5. Aadir al tratamiento nitratos transdrmicos.

La respuesta correcta es: 2 La aparicin de un osteosarcoma en una persona


de 65 aos de edad nos hara pensar en la existencia
COMENTARIO: previa de uno de los siguientes antecedentes:
Los diurticos se emplean en la IC para lograre alivio 1. Traumtico.
sintomtico. Si el paciente se encuentra euvolmico estn de 2. Mucopolisacaridosis.
ms y son prescindibles. Ante el mareo del paciente parece 3. Enfermedad de Paget.
razonable retirar primero los diurticos antes que un frmaco 4. Displasia fibrosa.
que suponga mejora pronstica. 5. Corticoterapia.

La respuesta correcta es: 3

253 COMENTARIO:
El osteosarcoma primario aparece fundamentalmente en
pacientes jvenes, en torno a 30 aos. La aparicin en
pacientes de edad avanzada debe hacernos sospechar que ha
Una mujer de 72 aos con enfermedad mitral surgido sobre hueso pagtico. La enfermedad de Paget afecta
consulta por disnea en situacin de insufieciencia cardiaca hasta al 10% de las personas de ms de 80 aos. La
congestiva. En la Rx de trax se observa cardiomegalia, degeneracin sarcomatosa del hueso pagtico ocurre en el 10
derrame pleural bilateral e imagen nodular a nivel de % de los casos o que hace que la enfermedad de Paget sea una
lbulo medio. Cul es la causa ms comn de este causa frecuente de osteosarcomas en ancianos.
ndulo?:
1. Neumona lobar.
2. Derrame cisural.
3. Atelectasia del lbulo medio.
4. Embolismo pulmonar. 256
5. Fstula arterio-venosa.

La respuesta correcta es: 2 El pediatra visita a un nio de 5 aos cuyo padre


acaba de ser diagnosticado de tuberculosis pulmonar
COMENTARIO: bacilfera. La prueba de la tuberculina del nio es
Pregunta fcil cuando se tiene experiencia clnica. Nos describen negativa. Cul es la actitud correcta en este caso?:
una insuficiencia cardaca con derrame pleural bilateral. Una 1. Al ser la prueba de la tuberculina negativa, el nio no
presentacin atpica del derrame es la formacin de imgenes ha sido infectado. Tranquilizar a la familia y no tomar
nodulares, cuyo diagnstico definitivo se obtiene con la ninguna medida.
evolucin radiolgica tras la deplecin de volumen. 2. Iniciar quimioprofilaxis primaria con isoniacida.
3. Iniciar quimioprofilaxis secundaria con isoniacida
porque seguro que el nio se ha infectado y la prueba de
la tuberculina no tiene valor en este caso.
4. No hacer nada de momento y volver a repetir la prueba
254 de la tuberculina 8-10 semanas despus.

52
5. Tratar al nio durante 6 meses con tres frmacos roposo.
(isoniacida, rifampicina y piracinamida). 4. Las manifestaciones oculares (edema periorbitario y
uveitis) se presntan frecuentemente en el subgrupo de
pacientes menores de 20 aos.
La respuesta correcta es: 2
5. Las manifestaciones cutneas se suelen presentar en
forma de eritema erisipeloide localizado en miembros
COMENTARIO: inferiores.
En nios en contacto con un sujeto bacilfero est indicado
iniciar tratamiento con isoniacida aunque el Mantoux sea
La respuesta correcta es: 5
negativo. Pasados 2 3 meses se debe repetir el Mantoux y
actuar en consecuencia.
COMENTARIO:
La fiebre mediterrnea familiar se caracteriza por episodios
recurrentes de fiebre y serositis. Se trata de una enfermedad
autonmica recesiva ligada a un defecto en el cromosoma 16.
257 Provoca algn tipo de dficit relacionado con la susceptibilidad a
los episodios de inflamacin. La clnica articular suele ser mono
u oligoartritis migratoria y no necesariamente se asocia a los
episodios de serositis. La afectacin cutnea consiste en zonas
Mujer de raza blanca, de 48 aos, que manifiesta eritematosas y dolorosas de tumefaccin, habitualmente
en los ltimos tres aos, cinco crisis de vrtigos rotatorios localizadas en parte inferior de las piernas. En caso de
de entre una y tres horas de duracin, con nuseas y presentar afectacin ocular lo tpico son los cuerpos coloide en
vmitos, sudor fro, plenitud de odo. Nistagmo. Acfenos fondo de ojo.
en el odo izquierdo que preceden a las crisis vertiginosas.
Hipoacusia del odo izquierdo que se confirma con la
audiometra de tipo neurosensorial con mayor prdida en
las frecuencias graves. A la exploracin por micro-
otoscopia, se aprecian conductos auditivos externos y
259
tmpanos normales. La Resonancia Magntica cerebral con
contraste es normal. Qu diagnstico considera ms
acertado?:
1. Neuronitis vestibular izquierda. En un estudio sobre la prevencin secundaria de
2. Vrtigo Postural Paroxstico Benigno. accidentes cerebrovasculares con cido acetil saliclico se
3. Neurima Vestibular izquierdo. encontr que la proporcin de eventos en el grupo control
4. Enfermedad de Menire. fue control fue 0,07 y en grupo que recibi el
5. Otosclerosis. antiagregante fue 0,09, lo que supuso una reduccin de
-29% del riesgo relativo. Los lmites del intervalo de
confianza al 95% de dicha reduccin fueron -100% y
La respuesta correcta es: 4
+43%. Cul es la interpretacin de este resultado?:
1. Es igualmente probable que el frmaco doble el riesgo,
COMENTARIO: que lo aumente en un 29% o que lo reduzca en un 43%,
La clnica que presenta la paciente corresponde sin ninguna por lo que es estudio no aclara si el frmaco es eficaz.
duda a una enfermedad de Mnier o hidrops endolinftico 2. Se puede concluir que los pacientes que recibiesen el
idioptico. Esta enfermedad se caracteriza por presentar crisis frmaco tendran un riesgo de un 29% mayor.
recortadas (entre una y varios horas) de vrtigo con 3. El 95% de los estudios iguales a ste mostrara una
caractersticas de perifrico (inicio brusco, sensacin de giro de reduccin del riesgo relativo de -100% a +43%, aunque
obtetos y sntomas vegetativos), plenitud tica, acfeno e seran ms frecuentes los valores ms prximos a -29%
hipoacusia en el odo afecto. La hipoacusia es de origen coclear, que los ms prximos a -100% o a +43%. Por tanto, el
por lo tanto es neurosensorial con mala inteligibilidad y estudio no ha sido concluyente.
reclutamiento. Es caracterstico que las frecuencias graves se 4. Para poder interpretar estos resultados es
vean ms afectadas (curva de la audiometria tonal ascendente). imprescindible conocer el nmero de pacientes que se
La exploracin otoscpica y radiolgica, incluyendo RM, debe incluyeron en el estudio.
ser normal para su diagnstico. En la neuritis vestibular no hay 5. Dada la gran amplitud del intervalo de confianza, lo ms
hipoacusia ni acfeno y la crisis de vrtigo es nica aunque con probable es que en este estudio se haya cometido un error
reagudizaciones en su inicio. El VPPB se desencadena con los de tipo I.
cambios se posicin y no presentan hipoacusia ni acfeno. El
neurinoma se caracteriza en el 90% de los casos por hipoacusia
La respuesta correcta es: 3
perceptiva unilateral y acfero continuo de larga evolucin, sin
crisis vertiginosas. La RM con gatolinio lo diagnostica. En la
otoesclerosis la hipoacusia tiene componente transmisivo en el COMENTARIO:
85% de los casos. Un intervalo de confianza al 95% de una reduccin del RR de
-100% y +43 se interpretad como sigue: El 95% de los
estudios iguales a ste mostrara una reduccin del riesgo
relativo de -100% a +43%, as que, al incluir al 0 el estudio no
ha sido concluyente.
258

Seale la respuesta correcta en relacin con la


260
Fiebre Mediterrnea Familiar:
1. Presenta un patrn de herencia autonmico dominante.
2. Est ligada a una mutacin del gen que codifica el
receptor tipo 1 de factor de necrosis tumoral-alfa. Uno de los tumores citados a continuacin, de
3. Junto a los episodios de fiebre recurrente y dolor conducta benigna, es frecuente en mediastino anterior:
abdominal, la mayora de los pacientes presentan intensas 1. Linfoma.
mialgias migratorias tanto asociadas a la fiebre como en 2. Seminoma.

53
3. Timoma. COMENTARIO:
4. Ganglioneuroma. No olvides que en mediastino anterior son frecuentes los
5. Quiste broncgeno. timomas (opcin 3), linfomas, teratomas y el tumor de tiroides.

La respuesta correcta es: 3

54

You might also like